You are on page 1of 323

Practicas para Resolver

PROBLEMAS

MATEMATICOS
V.Litcineneo, A. Mordkovich

jlfge6ra y
Triqonometria
Editorial Mir Moscu

PROBLEMAS

Praauas

PdfU

ReIolt'tr

MATEMATICOS

H. II. JI nrmmenao,

A.

r_MOP~!.;oDIl'"!
31\JlA~1

no

npJ\I{THHYl\I PEllIEB]IJO MATEMATWIECHJtX

A.TlreGpu
TplTr0.f10MCTPllll

lI1oCl(Ba «I J pocneuieu lie ,)

PROBLEMAS MATEMATICOS
V Litvinenko, A. Mordkovich

Praaicas para Reso/tler

jlCge6ra

crrig onometria

Editorial Mir Moscu

Traducirlo del fl1S0 por el iugen iero Autonio Bal lestcros Elias

lmpreso

ell la

unss

l ln

ucnaucuou

1131,lHC

IS UN 5-0:l-{)OO!i97--'i

©
©

1'13Jl,aTCJ'lbCTlII)

-rt

POCDClI\CIIIICO,

iDS'. Mir, HJ89

truducc iou al espafiol , editorial

Indice

Pr61ogo
PHlMEHA Capitulo §
§

-'

l'A IrTE.

ALGEI.IHA idcJlllcas de expresiones 7 11 1~J


2\)
,,') j~

I. 'I'rausformacloncs

§
§ §

q. Translcrmacloncs
rituileas

1. Dcscomposic.ion 2. Translermociouos 3. Tranalormnclcaos

d~ pol.inomios on Inctorcs . Jr.icnllcns do ex fll'PSJOIlCS racionulcs idcnlic<\s de ex prosioues inaciollill.'s idenlicas do C xpresioncs ex 11()IlCnCI'lh,'~y J"ga·
J as

5. Demostracidu § 6. Comparacton

de dcsigualdmlcs de los valorcs de

expresioncs

numcrlcns

:J3

Cnpitulo II. Soluci6n de ecuactones y dcsigunldades § 7. E IJI! ivnlcncl 0. <1 o ccuacl ones § 8. Ecuaclones racionales § tl. Ecuncioncs quo conucncn \1I11l variable hn]u el signu do 111611ull) § 111. Sis tcmas UC CCII cloncs racionalcs u § 11. Prublemas para In compcslclon de ccuacloncs Y sistemns •k I'CII,I§ 12. ECII acionos j rraci GUllies § 13. Ecu (I cl ones C~pononcia I (!s § 14. ECllllCiOlIC'S lognl'itlllicas § 15. Sistemas do ccunciones expuncnc lulcs § Hi. Dl'sigllnldn1lcs rncionnles § 17. Deslguuklados Irrnclenales
con ciones
'1(\

r,.'!
(;(

0.'1

8:\ III 12;)


('oil

y 11l~:nit"li~,!lS

nl
I·\'t

§ 18. Dcsigunldades cxponenclalcs § 19, Doslgualdndcs lognritmlcus § 20. Ecuacioncs y desigualdades SEGUNDA Capitulo
§ § §

Hili

n:!

panimetros THlGONOlllETnlA iMnliclIs de expreslones

I rt 185

PAInE.

I. 'I'ranalormacienes

L Trausformaciones itlendicas de expresioncs trigononuitricas 2. 'I'ranslonuaclon de las cxpresiones que con tiencn Iuncroncs nometricns inverses 3, Dcmnstraclon lie dcsigualdmk-s Capitulu
(II~

20~) trigo22.')

~:n
2-H

§ 6, Dcsigualdadcs § 7. Ecuacloncs y desigualdadcs


SO luc ioncs

§ §

4, EC\laciollt's
5. Sistemas

II.

Itesolucion

de ccuaclnnes

y dc~igu .... ld.ules

ecuaclones

con par.imetrus

2,;1 21H
:!!11.

;WI

El PIC!SCJltl~ m au unl L'stii dicigido a los estudinntcs de Ins Incultaues fis icoma to II Hl L iCil,~ de Ias Escue] as N ormales Superiores que ostud ian In especialidart N° 2104 «Matemat icas» y «Maternaticns y Iistcn». Ha side osr.rito on correspondcncta COli el programa 011 vigor del curso «Pract icas para resolver problemas mateuniticose. Al escrihir ol IiOL'() los autores han procurado quo on 61 estell rcllejad os los LI pos f unda men tales de problemas escolaros de algebra y trlgonomeuia (en el mnnual hay cerca de 2000 ejempios, problemas, ojercicios). Do ellos, 1700 son ejercicios de tliversa complejidnd para d Irabnio ind ividual (jUII to con los problemas rala Livamen to sancillos, osuindar). La rssolucion de sernejantes problemas ayudard a que e1 ea tudian te CJ 1Ii01'1l 1a cual idnd profeslonal nuis im portan Le del futuro maestro do mntomtiticas: la capacidad de resolver los preblemas do mnteuuitlcns dol CUl'SU 05co101".

hay

0 tros

cuya

solucion

req ulere

un serio

trabajo , en ocaslones

no

au

El llbro que ohccomos al lee lor as 110 5610 Y 110 ta n to U II compand [0 do problemas, si no till libra de pract.icas. Esto so refleja en Ia estrucLura del Iibro. Gada apartado conticne, ell forma iulorrn ativn , ol
ma tarial teorico

:WO) sjemplos detalladamente


de el punto de vista

necesnrio

y una can tidad

analizados,

ut iles a los estud iantes des-

bastan

te gran de (cerca

do

Al trabajar con e1 presen ta material los all tores so han bnsndo ell In serie de manuales para el curso «Prdcl.icas para resolver problemas matem atlcos» oscritus para los sstudiantcs pOI' correspondencin 011 diversos afios. Al cscrihir e1 presente manual Iuoron tambien elllplendos los HInDs de texto y los manuales escolares, Iibros para ]05 maestros, diversos conipendios de problemas de algebra y trigouometria, manuales para los estud ian tes q uo ingresan on los cen tros do ensefianza superior, las variantos de los problemas do matonuiticas parn los exarneues de ingreso a distinLos centres de enseiianza superior, los mntr-riales de las ollmpiadas escol ares de matemriticns.

matodologico.

Los autores

Priuiera

parte.

ALGEBRA

Capitulo

TRANSFORMACIONES

IDENTICAS DE EXPll,ESIONES

§ 1. Dcscomposici6n

de pollnomtos

en facto res

Para resolver muchos problemas algebra icos suele ser precise representar el polinomio dado en forma del producto de dos 0 nuts polinomios 0 Lien en forma del producto del pol inomio pOl' uu rnonomio que contenga no menos do una variable. No obstante. no cada polinoroio permite realizar Ia descomposiclon en factores sohre el campo de mimeros reales, P. ej .• los polinoro iDS x ;1, x2 6x to no pusdon ser descompuestos en fa cto res. Somejantes pollnomios racihen el nornbre do ItO reducibles. Se cousidera que In descomposicion do polinomios en Iactores est[i term inudn si los polinom ios obteuidos SOil no red ucibles.

Durante

In doscomposlcion
procedimisn

cion abroviada,
E.JIWPLO

10.agrupaclon, etc. Exarninemos estes procedimientos.


1.

de di versos

tesis, mediante

Los: sacando

01 r actor COIllUII de los parenampleamlo las f6rmulas de mult ipl ica-

tlo pol inomios


varies

011

rae toros so haec


de nphcaclon

1I~()

ejemplos

rle

I. Descompongamos el poHnomio ell Iactores (a, b) = a2 - 2a3b ~ 2ab3 b2, 3 -7a'J. 2. (a) = a 7a 15. SOI.lTCION. 1) P. ej, unarnos los sumandos extremes en un grupo Y los medics, en otro y en el segundo grupo sacamos de los panintests

+
-

e1 factor comun. Obtenemos:

2ab (a2 b2) = (a~ b2) (! - 2ab). 2) R epresentemos los t01'1ninos segundo y tercero del po I i nom io preiijado de In forma siguieutc: -7a~= -3a2 - !w2, 70 =I~a 2)

f (a,

b) = (a2

+b

-5a.

parentesis los factores comunss: I (a) = (a3 - 3a2) - (4a2 -12a) - (5a = a (a - 3) 2

Entoucss escribimos: Agrupamos los sumandos

f (a) =

a pares

aJ

y en cad a grupo

3a2

4a2

+ :l2a

5a

sncnmos

J~. de los

'15) =
;)

4a (a - 3) -

5 (a -

3) = (a -

(aZ -4a

- 5).

Pl'iltll'l'il

parte. Algebrn, Capitulo I 4a 5.

2-do pracedimiento, Dc 10. ecuacion aZ - 4a - 5 = 0 hal lamos las rakes: a( = -1, ag - 5. Empleaudo In formula de descomposicion en Iactores de nn trinoru io cuadrritico (ax? bx c= a (x - Xl) (.1; - x2)}, ohtenemos: al - 4a - 5 = (a - all X X (a - a';l) = (a 1) (a - 5). Asl, piles, / (al = (a - 3) (a '1) (a - 5). !tJ EM PU,l 2. Descompo ngamos ell Iac tOl'OS

Quorln por doscompoucr en factures el polinomio a'l Esto es posible de rcal izur con dos procadunientos. I-er procetlimiento. Tcnemos: a~ - Lja - 5 = a'l -I- a = a (a 1) - 3 (a 'I) = (a 1) (n - 5),

5a -

5=

(a, b, c) = ab (a

b) -

be (b

.+ c)

-I-

ac (a -

c).

Solucuiu. Aprovechumos que la ex presion en los prirneros parentcsis es 1a surua de las expresiones contenid as en los pardntesis segundo y terccro: a b = (b c) (a - c). Entouces j (a, b, c) = ab «(b -I- c) -I- (a - c)) - be (b -I- c) ac (a - c) =-

ab (b

+- c)

-I- ab (a ~ c) In agrupac

be (b

+
c)

+ ac

(a -

c).

A cont iuuacion

de los parentosis
= (b

, elcctuamos

(a, b, c) = (ab (b

+ (a. -

+ c)
be)

01 Iactor

couuiu.
c)

c) (ab -

c) a (b
3,

+- c)

+ (a. -

be (b

+ e)) (ab +
011

Obteuemos: ac) = (b

ion de los lcrminos y sacamos

-I- (ab (a -

= (a -

c) (b -I- c) (a
l-

+ c)

c)

+ ac (a b (a c)

e» =

b).

gJEMI'LO

Descompougamos

Iactorcs
2 -

f
S()LUC!ON.

(a) = a

5a
.

+ 5.
SaClllOlTIOS

parentesis

01 Iactor
-

Itonliccmos
5a2)
-

In agrupaciou 5) = a2 (a -

y, despues, 5) -

de los

cornu n ;

(a) = (a3

(a. -

(a -

5) =

==
Ernpleando ob tensmos:
EH~MI'!.O

(a -

5) (0.2

scguidauicnto

Ia
(a b)

forJllula
5) (a ell

p2 _
1) (a

1). q'l. = (p -

q) (p

+ q),

f
't,

(a)

+ f).
5b2• t2ab hasta el cuatlratlo
Entonces
3b

Descompougamos

Inctorcs:

f (a,
SULUCION.

perfecto.

Ccmplctumos ObteilcllIos: (2a}2 -

12ab el Iiiuornio 4a2


2 -

4a

+
-

2 (2a) (3b)

(a, b)

(ItaZ -12ab

+ (3b)2.

Db2) -

UU~ -I- 5b'l =


3b 2b) (2a b). 5b) (2a -

= (2a -

~1U)2 -

(2&)2 = (2a -

+ 2b)

= (2a -

§ 1. Dcsl~oml'osicioli
I'JEMI'LO 5_

de poli noiruos
(>11
--

ell LI(;V"rs

Descompongamos / (a)

Inctoros

(t.j

10a2

+ 1GU. +
=

SOLUClON- AdvirLiendo qlle a~ J6g = «(!2)~ LF y comp lctaudo esta sum a hasta e1 cuadrndo perfecto, uhtencmos

f (a)

= (a4

+ 2Ga + 1(9)
2

= (a2
EJ ElIll'LO

+- 13)2 =

2Ga2 - 10a2 (6a)2 = (a~ -

oa -I- B) «(12+

(ia

-I- 13).

u,

Descom ponga mos ell f ac teres

-I- a2b2 -I- b4 - ll. SOLUC\ON. Como a6 - bU = (a3)Z _ (bJ)'l = (aJ - 63) (It~ + bJ) = 2 = (a - b) (a + ab + b2) (a + b) (aZ - ab -I- bl) y a~ + a2b2 -I- oj = = (a~ + 2albz + b~) - a2b2 = (a2 -I- b~)2 - (ab)2 = (a~ + ab + b~) X
/ (a, 0) aO
J

+a

-I- b2) - b2

X (aZ

BJEMPLQ 7,

«a -

ab -I- b2), b) (a 1).

entonces / (a. b) = (a2 ab b) 1) = (a2 ab 02) (a2

+-

+
-

bZ) (a2

au -I-

ab

/)2) (aZ

+
-

Dsscompongamos ell Iactores f (a) = al -I- 9a2 + 27a + 19.


el cundrud 0 perfecto
!\

sion / (al le Ialta 8. Pal' ello, podemos escribir f (0) = (a~ + Ua2 + + 27a + 27) - 8 = (a "1- 3)3 - Z 23 = (a + 3 -- 2) (a -I- ;q~ -I+ (a -I- 3)·2 il) = (a + 1) (a -I- Sa + Hl). I!:mMI'LO~, Demostrcmos (1'10 st a E LV y J (al = 0:1 -I- Oa:1 -12+l3a, +lIa f(a):24*.

SOI.\lCI<m, Es fneil vel' q lie hasto

1II ex pre-

+
3

SOLlH':ION,

= 1) (a 2) de C110tro mimeros na turales sucesivos, per 10 menus '111(1 dl' cllos sc divide per 3, <lsi como dos mimeros son pares, es decir, IIII\J de ellos se divide tambien por 4, POl' 10 tanto, el prorlucto dt! rsto;; cuutro
2 2

f (al =

tcrminos
a4

fI. epresen tum os (ja~ y 'll a~ ell Iornu: dt, (n s 11111 d o IOH iI semejantes: 6aJ = a:J + 5a3 y 'lJa2 = JCL2 + (\fl,2. Ellt()IICUS.

+ 6a) = aa (a + 1) -I= a (a + 1) (a + Sa + 6) = a (a -I-I- 5( ) -I- (lSa


2

+ (a + 5a + (5a + Ga
3) 2

5a~ (a -I- t)

-I-

Oa = (a'

+. un (a + I) (a. + B). Pcro

-to a:J) -I- (;'(1:1 "I",

Demostremos que si I (a) = a2 (a~ -I 1iJ) +49, lifJlldc' a es un mimero impar, f (a) ; 04.. SOLIJCI()N Nolcmos que f (a) = n'l l"a~ -I rj\) :c- ((/~ -I COIllO a es impur, u = 2n - J, doudc It EN. Esl(lJICC:;. I (a) = = / (2n -1) == ((211 _1)2 -I- 7)2 = (411.2 - 4.n -~ -~~l!j (nt ~ It -I- 2)2. La expresi6n nhtenitla se divide pOL' 1l1. P(II- esl a 1'<11.611, para dernostrar que / (a) : 64 es suliciente demostrur que (/12 - n
EJEhlPLO Q.

numerus

se divide

POl'

3·2 ·4. Asi, pues, / (e) : 24.

7r.

sr

2) n es un numero
• Itecnnlemos

+ 2)2

: 4. Analicernos
que

impar.

dos posiblcs

cases: 1)

It CS

1111

numero

par y

(.\ sig no

\<

» signif ica «se d i.... pu ...) Hn rcst»), ille

10

Pnrnera parte. Algebra. Capi tulo I

1) Si Il es PH, n~ (nmhien 10 es y, por 10 tanto, n2 - n + 2 as pal', 0 sell, (/1.2 •.•- n 2) : 2 y, por consiguien to, (n~ - It 2)2 : 4, Jo q tIC signif'ica quo f (a) ; G4. 2) Si It es irup.ir, n~ trunbien 10 es, pew entonces n2 - It es par y n2 - lL -I- 2. 1-a tubi6n. Asl, pues, ell scmejante case f (a) : G4.

EJERCICIOS

+ 1. 4,. a" - 18a + 81. 6. a· + a 'l 7. a~ + 2a 2a - 18. 4lh2 - (u~ + !). a + I,' ttL a' + 5. 11. 4a + 5a + 1 12.,~ - (l + au)c + au. 13. a' + 324. 14. + + 1. 15. + a~-\- I. 16. Za' + aa + 4a~ + + 2. 17. a( + 34 + 41l~ - 60. - 12, 18. (a~ + -I- 3) (a + a + 4) - 1Z. 19. a + a a 1. 20. 2(121J + + 4b c + 2bc 4abc. 21. (ab + ac + be) (a + b + c) - abc. 22. (b - 2d2 + /} - 2{')~ - 2c (a + b)2 + Sabe. (II 23. a~ (a 7)2 - JlJa 2'1. (a + b)5 - (a~ + Pl25. a 1o'J (b - ai + (c - b) + a c (a - e). 26. 8a (1.0 + c) (Za + c) - c (2a - b}, 27. (a + /;+ c)l - (fl + /,J + c~). 2B. + 2U. a4 + i,-I, 30. a" + 5n + 3a - fl. 31. a. (II + 1) (a + 2}(a + 3) + 1. :J2. (a + 1) (a + 3) (fl +- 5) (a + 7) + 15_ ;13, 2 «(12 + 2a1)~ + 5 (aZ + 2<% - 1) + 1) + 2 (a. ~ + 1)2, M. (a - b)c~ - (I). - r)b~ + (I< :15. «t - b)~ + (tl - c);) - (II - c)",
s.
1.
4'

l)~'~(:UIIIl'ollg.lt\ Iactores: eu alz -

I. 2, a~ 2<.1'1 1
3-

1 3. a3
<l:~)2.

(12 -

1.'1 -

l4

1l2b~
2

(l/l~ -

(14

all

C1~

(l

<1

<1(1/;2

a2c

a(~ -

11

112(2

22

/)3

<I~

t),

l'

(112

C)(lJ,

HI),

Ij

1.3.

++ ++ ++ "0. + + +- + + 1. a~ + +++ "2. n + a~ + (2(1 + 2. 4"', a + a + 1. 1i5. Dcmuestren que sl o: ~ N, (al> - 54 + 4tt) : 120. 40. Dcmuestron que si cs un mimero mutuamonte prlrno con 6, (a 1) : 24. 7 . Domuostrcn que' si a EN. (2a + 3a + (1) : 6. 48. (Con que vnlores de HEN Ia expreslon a + 4 es un nurnero prime? a a "H. Delli IICSll'l'n que si II os un numaro par, 12 + 8+ 24 es un numero
38. (l2(}2 39. a4
a~
(1,1 -

:)7.

«(l~ -Ia"

(1)2 ,zp - (0.' - c:)3. :\a2 [,2 - 'labJ - b4• (1/.,2 a2( -I- b2c br;.1 3abc. b~ c~ - 2a2b~ - 2a~c2 - 2/>~{;2. n~ al a~ a 1. ~a3 3112 2a 1. 2,.a[, _. 8a~b~ - Gab~ - [,'. /,2);) 2(1:1/) lO

CI

2-

,oj

(lz

~lItOI·O.

Il '~ 2, Trallsf onnaciones rncion ales identicas II(' l'XPI'CSiOllCS

Ln sustituciou de Hila expreslon nnnl itica pOl' otra id(;nlLr.amenlo Igunl a ella en cierto coniunto , l leva ol uombro do tmmformacion .identica en este coniunto de In expresion darla. AI realizer translormnciones irl~ll t icas de una expresio II C5 posi b Ie In vnriaciun de SIl cam po de UC[ uuciuu. P .eL, red !I,~ it'!Id 0 los lenll i!lOS scmejantcs HI simpl ilicar la expresion
.'t~

+ 3x

[) --I-

~/x - l X,
5

( I)
dll!!:l :;(,10

.definida

SU CRm po de definicion: COli 0, m ieutras que ·do Ja snnpl ificac icn

fun pli.uuos

X>

10. oxprosion

el pol iuonno -

cbteuuIo

est 1I despll(;,'i

x2

+ 3x

(2)

(~st:l.dcfin ida call cualesq uisra valores de z. Lns ex Pi (0:; 11.l1l0~ (I) Y (:L) .son iden ticamou te iguales s610 en el con] unto 1O; 00 [. El campo de definicion do In oxpresien pucdc asurusruo \'Hill!' ·dL1spues do lot slmplrlicacien de In lrncclon. Asi, In f!'al'.cHil'l nigollf',jCO
(.T-l) (.T-[·2)

·eslil {:r -

doCi II iUO COl\ 1), obtencmos

In Iraccinn

=;!= 1,

z =;!= -2,
,.2

VCSPUl-S do

+3" -1-1
"'+2

(1)

·rlefinida con ,'"C =;!= -2. Las expresioncs (a) r (4) SOH id(~nLic-al1lcfllc igualos CII 01 cnnj un to J-oo; -2[ U 1-2; 11 U J1; 00 l. L1I varlaclon del campo de deri II ici0n de In expresicu (IS uun bien -posiblo como resultado de cicrtns otras tvanslormocicucs. POl' 10 que, dospues de eloctuar l a transiormaciuu tie In cxpresiou rhul a , siernpro hay que saber responder a In pregunt» en ({11~ cnujun l o plla es idellLica a la obtenida.
Una axpresion algchraica Ilevn el nombro de racionol. si ella solo contione opcracicnos de su mal', mill Liplicar, restnr, d iv nl ir y ole v ,1cion n Ilna poteucia ontern.
Jo:.lJ'~l\lI'LO'

Simplifiqnemos
Represent.ando

In expresion
ab como

SOLUCWN.

jilnles

2a2

+ au -

2ab-

ab, obtenemos

a,) la SHinn do los t~nnll\{)~ some-

(a,

b)

= 211'-j-'~~'II:

b2

= 2a~ + 2ab ~ ab -lP = (a + b) (2a ~ b),


(o+b)
0+11

= 2a (a

+ bJ

u (a -I- lJ)

Entonces

f(a,

b)=

(211-[,)

~2a-b.

12

Prunera

parte. Algt'br,1. Capi tulo I

La Sillll'liIic(l(·.i'·"1 do que a -I- b


EJ UlI'V 12

n lin parttenrlo '*' pOI.'I)c -I- b so que rcalizndc/J) = I


(J.
Ill{l(io

(a,

2a -

".

~llil

P 1If IIl'lClllUS

..

1<1

- _-

C"-Pl'tSIOIl

I (a) =

b, si a =1= -b, II' -JOa~+ 1G!l


II~

de In condicion

+ I'm-\- 13

SOLUCION 1l0ll!OS (vcnSD

Dvspues

X (a2

(ia +13). l I} I0 l'nut.o, -J' {) OJ' \a:.-

de descompollN en Iactores ol numorador. el cjern . ;), p.ig. D): a~- lOa~+ Hi!) = (a2 Oa

t<l~+ljll-I~L~) (.1!-(;(t+13)

COUlD

todos los vnloi es de (t. EJ I':MPLO:J Sim pit Iiqucmos

a (a2

+ Oa'+ 1:-:1llO so reduce a taro + Gil + 1.'~= (a + 3)2 + > 0),


a'!
11

a~-r-lli+jJ

+ 1.3) X (j + 1'j ='a-- rz


'j

obte-

cI.

COli

!ling-ull
-

(n) = a2

valor real de Ga -I- 13 con

In

ex presion
,
2

1 2,1 I) " ", ' , I {a) '= ('a~-r u(j-l--2.. -,1-.iI-+'1(!~I-,~-1- a'+5a-I-(,
SOLUCWN

(,1-3P+
Z

1211

f\ eal izu n do 1as oporuciones ( a+3+2a(,1+2)+a+1


\(t+l) (a+2Ha+3)

iudica das,
)2.a~-(l(l+')+12a=

ob

tCIICIllOS:

I(a)=

=: (

2"~+ (ia-I-1 1,,1+ 1) Ilt+2) la+3} '( .'1~+311+2 = <J (a~+3tt+2) {Il-l--31

)!!. II" -I- I~<+ !l = 2 )2. «t-I-a)~ __ .,


.i!
..

Asi, pues, i
EJEMI'LO',

(a) = 2 si a =I=- -I, a =I=- -2, a =;6= -3. 'Si;n pi ifiq uenios In expresion 1_ V,-c) 'I

(at b, c)

= \,t-

'I"

-I--

I I il-e)(,-a)

b~

+ te-a)

c~

(I)' c-,

SOLUCI"",

Iled

lIC iondo

Lodas I <IS Irucciones

,11

IlJlII

iIIlQ

(;U IIlUll

denom iliad or, ohteuemus: '( J. ) a,~(u-(')-{'~l(l-c)+c1(II-b) J ,,t , u .' C =:" -.;_...,......:----='~'--~...;_-i---.(a-Uj (It-c) (II-C)

Ad virt ieudn qllo b - c = (a - c) - (a - b), translonnumos cl numerndor de 1<1 sig-llielltc forma: a~ (b - c) b1 (a - c) + c·J (a - b) = a-! (a - c) - a' (a - b) - b~ (a - c) c~ (a - b) =

= (II -

c) (II" -

b~)

-cAsi,

aJ=

+ (a
ostrc

b) (c2

lLZ) =

(a -

c) (a -

b) (a

+b~

(a-b)(b-c)(a-c). pues, si a =I=- b, a*De


III III

\~J J:;~II>LO ~

a3
=

SOLlJCWN

b3
l

-(b

X(b+c).

+ c: = (-b + ;lb'~c + 3bc~ + c


l C)3

(:0111(1

+ b + c = U, a = -b - c. EuloIlCCS, _ + h + c = -(b + + b +
3 3)

+ b"J + c:J = 3abc.


-Ib3

c, b c, / (a, b, c) = 1. os g \IC si a b e = 0,

*-

++

C)3

c3 =

-(:3b

2c

-I- 3bc) =

c3 -Jbc

§ 2, Transformaciones

de expresiones q 11(\ a3

racronalcs

=
b

Pero
3abc.

+c
G.

= ~rt,

De modo

-I- bl

1-

,:I =

-;~[,c \

-ill
a*, U;

*- 0;

EJEMPLO

(U-b + b-c +~)' c I!


II

* 0, Demostremos
entouces

que si a

+b +a=
C_

0, doiulo
\I.
en

(_C_' iJ I, - r _+_a_.+_I_!_).-,
II .1

SOLLIGION.

mora Iruccion
( a-II c

Cousirlcremos del segundo


U.1

el prod ucto
factor:

del pr i Iller

r actor

10. Pl'L=

= 'l + u~-lle+ac-1 Pcro

+ b-c +~) a
+ (a-b)
Hoi

au

_(_' [+ (b-.c/I '1- C-il) _= -b (, a~._c_, = 1 + c (,,-b) -(.,2-1>")


a-I> ai, • --- c a-I> iell _ _.

__ r_ a - 1/ _' __ /I-I!

(c-(a+bli
au

'1-,L -- C

n/J"

C-

(.

(I"

1-/')J)

scgu

plan team

to a

prod lie to q 110 cousidcrnrnos, 0 b lOll Do r 0)'1111\ o.m\!ug-a (>1 prod lie to del l' ri rnor Iactor
fn\(;I'.it)l1

+(;= .- C, 2c~ emos 1 +~.


n '[

P,)

I'

C l ln,

pI 1',1 1'1

d~ll St'g'llIldo

[ur tu r

(!.'i

ig!lld

la tercora

Iraccidu,
2,2
/1//

+
Z

+-=:..::., IIC
los

') I,

pOI'

In

~('g II 11

d;)

en

Lull,l!

qlll'

1'01'

2/1~.
CII

Sumenios,
2/;1 ca

resultados

Ilhh>llltlos
I!~) =
IT':

1+

+"1+

2a

be
-

+1 +
__3
-\-

=3+2
nliC

(-=':'_1' (II,

~+ be

2 (C·1+():'+bJ)

CUIIIO

(l~

+ {;' cJ = 3auc ( vease ul ojom. G, piig, + 3+ 2(aJ+b3+c~) =3+ 2·,1nl,c =U


auc alii:'

l:d),

10 quo

cion

las translormncionos idJl1 \ icas rlo ex pre" in nes rnclonalos acL{H\1l nocO!TI 0 objet ivo, si 110 co mn !Ilrd i (I pam rosol ver prnhlem as en los quo sa 11 ace IIS0 del III 6 l.mlu .Io ilid ucIII

teuinmos qllo demostrar. En los siguientos ejornplos ntonuiticu.

La ajinnacion dependiente de
cualquier n, sf se realizasi

El iudicado

miitodo

sa onuncia
uit

de In forma sigurento: tuimero natural It, es uihda para

dos condiciones:

a) la «[irnuicuin. es valida para n = t; h) de la oalidez de la alirmacion para n = k (con cualquicr ralor


natural de k)
SI!

desprende

iambien. sa oalidez

para n

La dornostracion segun e1 metoda de intlucciou matenuiticu rcalizn asi, Pri mero, 1a alirmacion a demostra r se verif lea fwra II

= If.j- 1.

se I,

Pri rnera parte.

Algobra.

C<lpi tulo I

BsLa parte do Iii domoslrilciou rccibo 01 nombro do base de In induecion. La srgureuto parte (\0 In demostracion Ilevn el nombre de paso de la induccuiu, l!:n ella so dcmuostra In vul idoz de Ia afirmaclou para n = lc '1 ell I" suposlciou do la val idez do In alrrmacidn parn It = k (suposicron de In iuduccion). EJEMPLQ 1. Dcmostrmuos q(leF 22 32 -I- n2 =

I!{II

+ I} (211 -I- 1)
G

+
es cs

+ + ...
ya
COIL

SOLUCION,

1 (1 +1) [2+ 6
r

Para
-.

Su pnugamos
2

n= 1 iii ufirmaclon
quo ella '1)6 (21)
It

varida, corrccta

qlle

"12=
CS

n=k,

dccir 1z + 22+ 32 + ...

+ k = k (k-\-

+ I).

Demostremos

q IJC en

tal ease ella lam liien os correc La

COli

12+

22 _1_ .)z

+ ... + k2+ (1.: + 1)a


2

+ 1, 0 sea, = (k + 1) (fl-I~2){2k-l= le (k+!)(2k;+1k-\-G)

3) .

En ofccto , t~

+-

j2

+ 32 + ,.. +- 1. + (Ie+
..6(k+l)~
=

1)2 = k (It-\- Ih{2k-l-l) _

+(/,;+1)2=

"lk+I)(2"-~IH

(k+ 1) (/(+2) (2k-l-3)

As! se Ira domostrado miruoro natural n.


EJ EMI'W 8.

In valldez

du la aiirmflcion

para cualquior
3

Demos t rcrnos quo 13


COil

+ 23+33 + ' .. -I- n


os valida
COli es correctn Demostremos

=
ya

(n
1

(It

t I))

2•

__(t (1 +1») 2.
2
13+23+3J bien es

SOLUCION

n = 1. la

!\fjrmncioll

que

13 =
uun-

Sn pougmuos

quo

1t=1',

cs dceil',

... j;;J= (k(k:}:1»)2, correctn COil = k + 1,


1l

que eutonces

sea,

P +23+33 En clecto,

+ ... + /c3 + (J£ + 1)3 = ( (k+1)Z(k+2)


+ 33+ ....+ ",1+
(k+
1}s

r.
',(kil)

11 -I- 33

(ft+ 1)3= (

-I-

(k

+ 1)3= (k tlc-l-1)P,tq

(k+1)z \k:+4k+4)

= (k+1}t+2»)

r-l2•

De este modo quoda demostrada la validsz de 10 aflrmac16n para cualquier numero natural n. EJElIfPLO 9 Demostremos que la suma de los cubes de tres mimeros reales sucesivos S8 divide por 9.

§ SOLUCl6N.

2. Tra nsf ormaciones que

de

0:1:

presioncs

]';1

ci on ~J.:s

15

Demostremos (n3

+ (n + '1)3 -I-

(n

+ 2)~)

'._l

(5 )

con cualquier It natural. Ante todocomprobemos :>i In af irm a c inn (5) es val ida toOl! It =1. Teuernos: P 23 -I- 33 = 3{3, pero 3(i: U, por To tanto, COli 1/ = t. la afirmaciou es cierta. Supcngamos que la afirmaci6n (5 )es cierta Cal! It = II:, o sea,

(k3
Demostrornos rcalidad, (k k3 -I- 9k2

+ (k + 1)3 + (Ie --I- 2)3)


+

: 9.

+ 1V + (k + 2Y' -IComo carla sumaudo

(el primer

+ 3k + 3).

que en tal caso tamb isn as cierta can It -~ k -I- 1. En (k -I- 3)3 = (Ic 1)3 -I- {k 2)3 27k 27 = (lt3 (k -I- 1)3 (k 2)3) + U (1(2 -I-

contaner De acuordo can 81 principia de In ill d uccion conclusion de que 18. afirmacion es clerta J;.JEMf'LO 10. Demostrernos que
(32"-1-1 can cua 1quier
SOLUCION,
It

su rnaudo segun I a suposicion de ind uccion , 01 segu udo por el fnctor 9), Ia sum" tam bien se d iv ide por eso IlfUIlCI'O.
lila Call

de la suma ohteiuda

+ +

+
D

so divielo

pOl"

tom 6 ticn llegn rnos n In Ladas las n EN.


(ti)

+ !iOn

(7) : 64

significa

n = 1, 33 40·1 - 67 = 0, pero 0; 04. Esto que COil n. = 1 In alirmacion (6) es cierta. Supongamos que ella es ciertacon n = k; a sea, (3211+1 +401c - 67) : (}Ii. Dcniosuemos que entonces tarnhien es cierta can n = Ie 1. En elccto , tonemes 3'U'+3 +40 (k 1) - 67 = 9 ·32k+1 40k - 27 = IJ (!321; t-l -ISi. Cada sumando se divide pOl' 64, porconsiguientu, sa divide, asimismo , por 64,. Asi, pues, 10. afirmacion COil todas las n EN. EJElI-lPLO 11. Dcmostremos que

natura!.

+
-

+ 40k

67) -

320k

+ 576

!)

(32k+1

+ 4..0k -

67)

+ 64 (U -

loda In sunia (6) as ciorta

5k).

(7)
COli

natural. n: = I Ia afirmncion as val jan, yfl que -1 (j +- 11 -I-I- 6 = 24 y 24 : 24. Supongamos que Ia afirmacion (7) es cierta can n = k, es dccir, (k~ 5k3 11k!'. 6k) ,24. Demostremos que entonces tamhicn as cierta can n = k -I- -1. Efectivamente, tenemos: ik -I- 1)~ -I6 (k 1)3 11 (k 1)2 -I- 6 (Ie 1) = <1£4 f,k3 +-1{Jc2-1- (lie)
It SOLUCION-

cualquier

Con

+ 2Ll

Si ahora demostramos

(k2

+ 1) + 4 (k + 11k).
3

que

WJ, +

11le) ; 6

(8 )

I't iIII era parte. A Igl'l.ll'<l, C;IPI tulo I -con touas t icn,
GSlO

Ius k , qucdani

divide por 24._ Ante nosotros resolverlo de nuevo hucernos

dcmostrado que la expresion prefijada 58 ha surgiuo un nuevo problema. Para usa dol mctorlo do induccion ma te ma-

+ 'l)~ + it (/It + 1) = (In:) + 11m) + 12 + 3m (In + 1). De dos nurnoros nntura les sucesivos m, (m + 1) uno de el los es ohliguLoriamcnLo par. por 10 tanto, (m (m + 1» : 2, mientras que -(3m (lit +- 1}) ; (j Pero , on tal caso , «m:! + 11m) + 12 + 3m (m + + 1» : G. ' De aqui llogamos a In conclusion de que (k..:l + 11k) : G con
(111.
cualqu ier Ic nntural LIl nlirmacion (8) queda rlemostrnda. Asi , plies, la alirmacion (7) os cierta para todas n EN. l1C!llOS til' nul icn r q ua el ojern plo ex ami n ado pucdo ser resuol to si II apl icar el todo de iJl d uccion III II lellla t ica,

Ante todo, coinprobornos si os vtilidu In alirruacicn (8) COil k = 1. es evi.loute: (t 11) : (i. Son In nfil'lllHcion (8) ciortn eon k = = In, es dccir. (m3 --I- 'I lilt) : 0, Domostrernos que eutouccs lnmuien 10 C'S COil It = In 1. En orecto,

me

RmTICICIOS .51. --

5a'Z-a-4 _ ____:_
n"-I

a'+a2-2 53. a,I+8 211' -I- 7f(~ + It ;1,1,1 -1- :ja~ - II " 57.
/1'1

(I.-'-az-12 !I,I+gaz+1:> • ::if! 1 [ill! -3a~b -3/1

+ a21,~ _1_ f,

(/"-1-

3a2

+2

§ 2. Translorrnaciones

de exprosiones

racionales

17

64 65. 66.

(a-b)

(a-c)

+ (b-c)(b-aj"i-(c-a)(c-b)" 1 , 1
r
(e-o) (a-b)

(6-c) (c~a)-

a+lI

b+c

-+

a-c • (13_'''0' +_c __ +e ) : c (1 +e)-.I (12+ac+c~ a~b-bc" a-c c be' a 1 a i _ 8b3 4bZ 8b3 - 4bZ L 6/. a2+2ab+2b~ a~-2ab+2b2 lib1 (a1.+2b2) 4112 Ca'-2b2j' a-b b-c c-a (a-b) (b-e) (e-a) 68. a+ b b+c c+a (a+b) (0+';) (c+a)·

(1

. (a-bJ (b-c)·

c-I-a

+
3

+
3

69.

a b -ab +b c-bc c a-cal a2b-ab:l.+b:lc-bcll.+c2a-cal (a1_b2)3+(b·_cll.)3+ (c2-a"l)3 70. (a-h)3+(b-c)3+(c-o)3


3 J 3

71. Derouestren la identidad: a-c c-a


(a-b) (a-c)+ (6-c)

(o-a)+

a-b «_--o:) (c-/)=

a-a

i2.

Demueslren la identidad:
,,1 (d-h) (a-h) {d-e)+b2 (a-c) (d-c) (b-c) {d-a)+c1. (b-a)

+ b-,,-+
+ +

2 c~o

(d-a) (d-b)=d2• (e-a) (c-b)

73. Demuestren que si

Il, 0, c E R, de la igualdad (a - 1,0)2 (6 - c)'l (c =(a+b-2e)2+(b+c-2a)J+(c+a-2b)2 51" deduce que a=b=c.

_a)l

74. Demuestren que con aE ll(a-i)(a-3) (a-4) (a-6_l+10 es un numero posittvo. 75. Hallen el menor valor de la expresion (a-i) (a-3) ta-4) (a-131+'[!J. 76. Oemuestren que si a+b+c=l1, 11. Dernucstren que si a+b+c=O, l m. It 18. Demuestrenquesi -;+T+-;;-=l 19. Demucstren que SI -b a
b -c a~+b~+c~

117 +&7

+ 7

aJ+bJ+c'J

1l2

+b2+Cl

,1

a~+bLI- c~ a~+ ~3+ c: . 5 2 l~ m -;!2+1)i"+~= L


1

abc 7+m+n='1,
a=(1,

n!

'*

+~/;-+~b c-a

donde

<l

=F

v,

+= c,

c, entonccs

(b~C)i

+ (~~i1)2 + (a~b)Z
a+b +c=O,
a6 (b2+C2)

=0. +b5
(412+("")

80. Demuestren que

si

+ (c~bl+!I~)=
JI) i nduc-

2 Las siguieates identidades han de dernustrarsa scgun el mCtt)do cion matem(hica·,


I.

(a3+bJ+,J)

(a'+b4+c4)

81. 1.2+~.3+ ...

+~(n+.1.)=

It

(n+1~ (n+2)

, En los ejerctcios 81-119 se supona que n EN.


2-0204

18

Prunora

parte. Algebra.

Capitulo

·8;). 86

1 2 1 1 11 +2T+3f + '" +-nl-=l--;r'

'.1!-1-2.21

o,+I<II!=(II+I)I-1.
II -

12 2~ . 87. 1.3+ 3.5


88. 89_ 90. 1 1-3.5+-,':1 1.2.3

+ ... + (2n-l)
2 . .').7+"'+

112

,.(n+1)
= 2 (211+ 1) •
/I

(211+1) (2a-1)
II

(211+1) (211+3)

I~(1I+1) 2 (211 1) (211 -I- 3) •

+234+ - -- +
, ,

j .2·3+23-.+

. _.

II (II

+-

(,,+1) (11+2) 2 "2 - (1~+1) (11+2) 11 (n+l) (11+2) (n+3) 1) (/1-1- 2) = 4


(11 ;2) (311+ 1)

1(1

1)

I !H. 2.12+,1·2"+ !J2. 1 1.2-3.;)

. -I-(1I+1j [
l J 4.5+

II~ = n (,1+1)

+
'I

- ..

+
x-

L
/I

(11-1-1) (11+2)

(11-1--::1)

="""3

((j-

!l3. 1+x+x2
!Ill.

+ -..

1 ) 1,,+1)(11-1-2) (11-1-3) x"+l-l _I_x" = l' doude

x =fo.

1.

7 -I- 77 -I--

77H- -. -

r 777 ... 7= .____,


u. cifJ':lS

7 (tonH-gn-lO) 81

05.

(11-1-1)(11-1-2)

.(II-I,.)=2".1·3·5·
tIl

... ·(2n-l).
(

(}G. t

--2

+-,1 --, 'I -+ ... .:


1 In" Iormulus I para

+-211- 1--2II =-+'1 II


las sumas: 1

+ ... +-21II •

Dcduzcau :l. !J7- S n = 1 . <:I

+ 3 5 + . -. -1...

(211- 1) (21t -I- 1) '.

no.

111 08. Sn=n+(,.7+·' I 1


1

'+(311-2)

(311-1-1)'

Sn=175+5_9+

+ (/1Il-3)

(/m-l-l)' t

100. S" =T(j+ lOt. Sn = P-2z-l-32_fl.~


Dernuesl.reu 102. x-l-2x:+3x2

1 G.11+ ...

+ ...-I- (_1)n-lIl3.
n=
••. x-(n-l-li

+ (511-4) (5n-l-1)'

las idcntrdades:

+."

.+l1X

xlHl+nxn+-~ (l--x)~

, donde x

i= 1.

103• a+1-1_a-l-3+(I+7+ 2 4 8
..
"",

+a+2"-1=
2n

(a-1) (2"-1) +n. 211.

§ 3. Transforrnaciones 1 2 " 1+x+l+x2+1+x~+"'+

de

ex presioncs

i rrncinonlos

1\1

2"
L

10".

l;r;' =1=
105

-I-x
?U~l

2"

--J--t~
x-

2""
.t2

1 ..
')11

n st

Jondc

1.
.1:3

.. ,-x

x~ -.--2+__ +__ 8"" + 1 1 x

lOG.

(x-+.1'<

=~ -1 (x111+2 __ 1_) -2,1-1X211

r+( x
-3; J

_l_

x-

-:I:

I-x"

1 J:-"~ tlfllldc ··_-_._-- ... .,"-I-x


X~'

;~

)ll+_." . +( x"+ 1511-1)

l-x-

I"

1):1*1_

=.

Dcmuestren
107. (em_I) lOll. (2~n+3

10. validc:z: de las atlnnactones:


: 35. 108. (4" : 17.

+ sn·3"+')

; g.
; II.

110. ((pn+3n·~_1_3")

111. (33'U~-811-9) : G4.


113. t2n+6·3H'+5·'''+I) 11". (7n+2+8"-'U,) ! 57. 11G. (21\·~.3n+51!-1j): 119. (1I"-1-3Itr'+GII~ -

112. (3,n+Z+5·2lnH):
115. (11"+~+122'tH)

ID_
: 133. 23_

: 37.
25.
:

117. (!J~IH'+2"N-I-2"+I): 105;j, ; 21.

118. (32'1+~ .5~1L_:-J:rIlH .22")

71!~-21t)

§ i~. 'I'rnnslormaclcnes
lrraclonales

idenlic[ls

de exprosiones

bien Ia elevacion de esta a potencia cionaria, Ilevan el nornbre de irracionales con relacicn de la variable
0

Las expresiones

algehrnicas

ell Ins quo hay extrnccnin

de 1<1ruiz Iraca dicha v ariaraclonal

ble. n
dud z"

> 1,

Recordernos

In definicion de raiz aritmet.ica. S! a .?:.'l: 0 y n E JY, s610 hay un Utl mero no nega ti vo x tal que se realize Ia igual-

a. Esto

nurnero

palencia del mimero De 10 dicho so desprende

denom iua rafz aritmetica no nega t ivo (t y so designa COil


x
5C

tonto que las igualdades


las.

quo la igualdad 4U = -7 0 bien

V 4U =

Y .:'10 =

'Va.

de n-esima Cit

7 os ciertu,

±7 son iucicr-

Si n es lin nurnero impel)" mayor que 1 ya < 0, por semejante numero negati ... x que z" = a. ·o Si n, k, mEN, a>O y b~O, en tonces: in,

IVa so cnt

iondo

'V'ab='Va:.'Vb.
Est" propledad ");f~-~ b11 -

Iactores, p.ej.,

11 8· 27·125
1!r-

5e

propaga

to ]'- al Jprod uc3'--de =cualq u ier = 'V 8· V 27· { 125 2· 3·5 --'-~~O.

11(1111(11'0

de

'Vb'

si b:;bO.

2*

20
OUSEnVACION,

Primora parte, Algebra. Capitulo I Si a<:(}


y b<O

las propledades
"/-

18 y 2" tornau In

forma:

'~'-b=T1/-,a-,"I'-jbl'l Jay l' "V


3",
4(1,

~-~
b

-';1m-

11/--

(va)It=';/al"
It!" ,--

P_!}]., p.ej,

I"

eJ' a= Hh, a. P .., v-

50, mvamh=1/a\
dudes 1°_511 tambien
S8

'VV- 12ra= r a. (a4= Va2., iVa =V-a.


3

(VaZ)3=Y(azp=i!ilB,

ODSERVACION" Si los Indices

do las ralces son numeros impares, las realizan para a < 0, b < 0 y para ab < O.

pre pie-

Recordemo.s otra Importante propiedad de la rail aritmettca: un numero par, es decir, n = Zk, t iene lugar 10. identidad

2V aV< =

si n es

=la!.

p.oj., l!"O/3-2)z=IV'3-21=2-Va. Ilecordemos Ia definicion de potencia 1) Si a 0, aO = 1.

con cxponente

rac lonnl ,

2)

Si a~,;:O, a"

3) 8i a>O,

ct-r

=+
a

= y' a'" (II.

111"-

In

son numeros

naturales.

71.::;:.2).

(r es un posi ttvu racional).


1

4) sr a<O, rnEZ, a-m=am. Enumeremos las propiedades fundamentalcs exponantes arb itrar ios racionales: 1a, ar• a' = a"'p. 2&. (ar)'=ar
-.
T •

de las potencias

con

3•

(aby= ar·b

douda

a> 0, b
I.

EJEl\lPLO

> 0, r Y S SOil mim eros arhitrarios Slmpl liiqusmos In expresion


simplifiquemos 1145 cad a uno
=3

racionales.

A
SOLUCION

<1(32+ V 45- 1(98) (l! 72- 11 500Primero

V8).
de los radicales

dados:

V 32 = Y 16·2 = 4 liZ,

= V9.5
V2,

tIl, V98 = V 49·2=

7 li2,

V72=1/36.2=6

Y500=V100.5=101/S,

llS=V4.2=21/2.

§ 3. Tra nsf or runci

0 !les

d (J

presioncs

j rrn

ci 0 n n lr-s

21

Despues de esto Ia ex presion prefijuda toroa Ia forma:

A=(4 VZ+3 V5-7


= (3

112) (6 V2-10 V5- 2V2) V 5-3 )/2) (4 V2-10V5j.

Despuas, ohtenemcs:
A = 12

V 10-24-150-1-30 = 6 (7 1110-29).
Simpl ifiquernos

V 10= 42 V 10-174

EJE;.rPLO 2.

Ia ex presion

A=

V 2 -I- -v 3· V 2 -I- V 2 + V 3· 1/2+ xV2-V2+V2+V3.


Multipliquemos primero 3.

~/ 2 +

f 2+ V 3 X
tercero y cuarto:

SOLUClO:-I".

los Iactores

It/ 2 - 11' 2 + V 2 -I- V

llr 2 + 11 2 + V 2 + V 3 ".~
2

4-

(V 2 -+- V 2 + V:3)
=

V 4-

(2

+ 1/2

-I-

V 3) =
factor:

V 2- \/2+ VS.
obtenldo
por 81 segundo

Multipl iquamos

el resultado

V 2- liz + V3. V z+ -VZ+ V3= V 4- (1/2+


= V4-(2+V3)=

1/3)2=

V2+V3.

V 2- V3. V 2+ -vs = -v 4- (V 3)2 = -v 4-3=


EJEMPLO SOLUCION. 3.

Multipliquemos

este resultado

por el primer factor:

1. Asi, pues, A = 1-

Simplifiquemos De acuerdo
COli

10. expresien
la pro piedad 2-

A=
y,

V (2-

i7)~.
obte.,1 =

neinos A""",
=

Simplifiquemos la expresi6n A = V 27 - 10 ·Vz. Esta clare que la expresi6n SEl simplificnru si rnsultn que bajo e1 signo de la rale tenemos el cuadrado perfecto de la dileEJEM.PLO~. SOLUCION.

V -(2-17) = V V7 -2.

V 12- V7' I.

Pero

y'7

<a

5a de las raices por 10 tanto

rencia entre ciertos dos numeros, Representemos 10 como el producto doble de dos ruimsros, la suma de cuyos cuadrados sen igual a 27, es decir, 10 =2

V2

yz

·VZ·5.

22

I'111(I{,1'1I

parte.

AIgdH'<l. Cap] tulo I

Do ostc modo, ... = Y 2 -21/2 . .5 2S = 1 2-5)z= IV2 Y como -5 <:: 0, A =;:; I;;JE:MI'LO s. Simp l if iquomos In ux p rosi on 11 = V 9V3-11

-VZ

-VZ.

-+

11cv

51

VZ.
y

SOLUCJr)N

rad icando

11(1111111'05.

TOliPIlH1S D V3 = 3 'V'3+ 6~r3 11 yi=!J l!2+2 f2=3(V3)z.-VZ+<VZ)l. De I1sLa 10 I'!! hl ,

Il aznnnndo como en el antorior ejempio, escri bamos el en Ior rua del cu 00 perfecto de I a d ilereucta ell tre rlos ciortos
=

n/3)3 +3· V3. (l/2)2

Y(v
A

J}3_ 3 (V3)~ 1/2+ 3

EJ BMI'LO

= ~'t ~/ ;) - ~/ 2)3 = 3 G. L rberemnu os de In


= ",'1 I 2-1

-v

-v 2.

V3

(V2)Z-{V2)3=
idad
Cll

Irracional

01

II U mcrndor

rlu

Ia

f rae cion

SOLlIC!\;N

don

POI' 01

M u iL ipi icando el numerudor Y dcnorn inador do Ia Irac«cund rado II 0 perfecto» de In su rna de los n umeros y 1,

ohteuemos:

VZ

.'1=
EJ'~Jl-I]'LO

'j

(, 2-1)

(;--2)"+:V2+!

«I

2)2+;,2+1)

V4-~~ 2+1 =0/4+;\/2+1. d 2)3-13 r


Oil

1. Lil)~ll'eJllOnOs L\

cit) 111 i r+ac ioun lu lad


/'

01 donominador

tic Ia frll('ci,'.IIl

1+1r2-V 3' SOLUC16N Ltoenimouos de }/3 on el denotuiuador. COil este fin, mult.ipl icamos el numorador y danonunnclor de Ia Irnccion pOL" l a ex presion ad jun La al denom inador:
.

3_

A = (-t

+ J"-z- r
l iborarnos

3 (l

+ yrz+ "'''3)
3 (l

_
21/2

3 (t
(1

3) (l + 112+\'3) -

+ 1'2+
ell

+ 1'2)2 -

-I- 1'2+

1'~3)

1/"3)

Ahnra,

HOS

lie

iii

el denoruiundor:
3 ({Z+2+ I/G) = -~-~-'--;.__~
3Vi

A=
£J EM!'LO 8.

3 (1+

2,.f'Z'lf2

y'z+ 1:"1hl'Z _
la su rua de

Ca lculomos
ILlgillllOS

20

+ V 392 -I- r 20 - V ~192.


igualdad, Y eh-vcObtenemos: 3U2 >:

SOLUC!ON.

ruos al culm los dos miembros (20+

A,=V20+V3B2-1-V20-11392
esln

1/ ;i~2) +3
x

(V 20+ V 392)zy 20- V 392+3 ~/20+ V (V 20- V 302)2 +(20- V 3n) = AJ,

;.1. 'l'ranslormacionos

de exprcsiones

in:rCll>ILI"':;

~;1
2(1

+ V 20-'V

hien

4.0+ 3 } 20+
3U2} = .13, don de

V 20 + V J\)2 + V 20+ d' + +

Ii ::HJ2 V 20-

V ;j~lZ> Of

-I--

V ;1\12+
,I.

Ii i)\:l~.".."

De osta forma, OU\'CIICIHOS: 40 3 3v' 202;$U;-3j2.;1 - ,.j" 4.0 + ",13- GJ1 - 40 = a . Pero kl - SA - 40 = (A;I - ..;.... 12) (tiA2 - lb,l) --I (lOll - 40) = A 2 (A - 4) 4A (A - II) -I- to (A - 4) = VI - /1) X x (A2+4A-I-10). COIllO A 2 4A -I- 10 = (,12 4A -I- 4) 6 = (.4 2)2

+ G;.\ = ,13,

con A

*=
0,

Ia

igual

dad

(A -

4)

(112

4. V20+V392-1o, 'I'ransforrnomos

4..4

10)

+
so

+
a

'<;I)!U

reai

iz

AS1, plies,
EJEIHPLO

V20- V3U2=4 1<1. ox-presion

f (a) =
u In Iunun
(l2

!1I10

no

CUll

+6n -I- \) ticno los sigll!ls do ln 1',111. V rlpl


~! 2-4a a
2 2

-I- q-!-

Va

Ilirid

11111.

Los Plll1~OS al = y a~ = 2 divirleu la recta uumerrcu On los Intervalos J-oo; -3[, r-;~; y [2; 00[. AIlUlic(:!J)J08 In cxpreston 2[ preiijnda en cada uno de estes intervalos. Call a < -3, tanemos: I a - 2 I = -a 2, I a -I- J I .= -0 - 3, 0 sea, f (a) = -a 2 - a - 3 = -2a - 1. Con -3::;;;;; a < 2, tencmos: I a - 2 I = -u 2, I a :1 I = =a 3 y, entonces, f (a) = -a 2 a :3 = 5. POl' fill, con a ~ 2, I.CIlCtllOS: I a - 2 I = a - 2, I n ~t I = = a -I- 3, os doeir, f (a) = a - 2 + a 3 = 20 -I- L

SOL{)CION. Como ~!a~- 4a + <'J = V (n - L)~V + lin + !J = 'V (a + ;:$) = I a + 3 I. I (a) = I a -.2 I -I-

-a

I iL + :~ I.

I a -:2 1

+++ +

+ +

-211-1,
Asi, pues, f(a)=
c:.1~MPLO 10.

si 11 < -3, 5, si -:h:~4<2, 2a +1, si

Sirup l if iquemos

In expresion

.- a+b2 I(a, b)=l / --,;-+2

V-' I" a+b~ a-V-/}--2lra,


-

.-

dondo a~O,
SOLUCION.

b>O.

(a,

b)=

V O/i:W
I l/a:+I, I-I ~.rb

V (Ya;I,,;
I

l/~~b

Z4

Primora

parte.

Algebra.

Capi tulo I
pOI'

Va + b.
Ahern

Como a~O,
Esto

u>O, Va+b>O
sig nifico quo

y,

10 I

tan to,

IVa+b!=

I f (a,o I)· = y~+ b - V;;}Ira

.
2)

-b<O.

homos

do anal izar

dos

casus:

'l>Va-u;;?O,
=

-v~con-

En el primer caso , tencmos; sigu ien te,

I Va-vi

Va -

y,

par

En 01 segundo caso

Iva-vI

,-

=-

eVa-b)

,-

y. por To tanto,
b '

f
Asl, plies,
EJEJIIPLQ

= (ct, b)

{-a+b+ I/o-/.! Vb
2

= 2 jfiib

'V t.

st
si

Va > b,
,/Ci < b v
r-

(r7.,

b) = [ 2 'Vab

--r-'

11.

Simp lifiq uomos


-,

I(

a,

IJ

3/1,)_(41a-2a

V- V:1T.'1i2 3 2b-1/nb2~ J b+ ),3V-Z 3,- V·ab aU +"iu.~I"':a-v v-;: • l' (lz_ y b

In oxproslon

a.

SOLUCI6N.

samente, simplifiquemos parsntesis. Tonemos,


1) a
31"': 3 y (l-2a l/ b

Realicemos las translcrmaciones por operaciones, Preciconsecuttvaments las Iracciones entre

'O'+- l" a"/P = V- JrJ,..a'~(~, 1.1'-2


=

Vn2-Vab

ra~t'a-~"b)2

~V- J,a-v b
J,-l' (llb

JV

(V-a -V-b) .
ab

va (Va--Vb)
b I.
(Lu.

3,-~;-;:o Y Ilb+l'

b2)

A con lin uaciun , 2) Aliora, 3)


Y, Asi,
3.r~.-

al,2

~ra_rrb

}!

3·-

3,i'

(31': y a(1-

Vy b

3ri':)

=V

3"

Va (VaV~;1/ a
pues,
2

Vb)
= I.
=

+;1 ab = V a
1.

2,

por fin, 4)

f (a, f.,)

§ 3. Translorrnaclcnes
Ii:JEMP!_O

do expresloues

irrncionulcs

!2_

Simpl if iquemos 1/~+\/(l+1

1£1 expresion

(a) = (

+ l/~-"\/ -I a

):

(I + l,r ~~:[.
y'

SOLUCI6N. Liquidemosln irracionalidad en el denom iuador, primero en la primera y, a continuacion, en In segundo Iraccion. Teuemos,

V-a - va:+=t -(Va+ya-H)(j'a-ya+l)


2). =
i

- - a-(,.-\-I)
IF~+ (;=t
a-f_a-f)

a-,/~

ya+Va-1.
3)
1

Ya-Va-t

Va+Va-t -----'=~.:...._~~==tV
(1-

~ra-t)

(V~+ Va-i)

Asi, pues,

V;)+Va+l
icn,

A contiuuac

4) 1+' / a+1 = If;;=!

V a-t

+ a:t=1
j/

Jfa-l

y, a continuacion

5) (11 a
Asi, pues,

+ 1+ V 1.t--1):

1'0:=1 + 118 =
ya-1

-v ((-

1.

t (a) = V a-

20
Simpl ifiquen lZ0. V7+'J '128. "12!).

Primor«

parte.

Algebra.

Capitulo

bs cx presiones: 127. V3----Z-l--:''''=Z.

vfa.

CV5+21/G-I'

1''-5-2 1/1j).1~3.

}/"2+ 11
1/'l7 -4
Liherense
4 ,'_

2+

J.1'3 I :H- 1/3 _.


jj2.
I,

/ 2-1 3 1/2- Y2-1!3


)/3.

1:10.

V!.}/'2+2

I/G.

'131. '133.

117+ 1-""'2.88,
!}

'V2.8-1G

'V +
.

I.' 5, 13!,. de 1:1 irt'ndollo.lida.d


UlLi.
J'

Ii-3+-:;-;'Vr=, S=_==V7=1S=+=):::§r§48.
un ol dcnom lnador de 10. fraccion:

"135.

1. ,5-,,'2
1

1 J,r' ,,'15-1.7

137.
139.

Vlr5+1'~

)/5-

,I

Jl3

,.138.
-3

.r ,r 1-1- r 2+ l' 3

a", l'

-'I-h> G-f-,' 9
4 __

3~

r-

1',1), 142.

1'14+
11 I

1/21 1

+ ),1"15+
'. If. .3.

tfl>1
f-

14. t. ~ ,_
I

2+" 4+v- 8+2


T

2+ 1-'-0 2 Y 2+21
1-

Y 1 2+ ~ :3

3- 1 (j-2

J-

Compruebcn

las sigUJl'lltes

igualdades:
j=-.

l2

'145.

/20+12 \/3 2+ 1/3

147.

5-2 }'25)
J5U.

+ '~25=4,

15[, /5 r'2+7Demuestrcn
I

V 5 ,,'2-7=2,
irlllicall\lo
01+

las idenlidade:s,
2
"J

el domluio

tic su determinacion:

tsz.

211
:;

J !

1
=ll.

113-3.-J

a3_aJ

"

J. -

012-1.\01+3

3. Trnnsformaclouos

dt.'

ex!,rc,~iOIlC5

iUllcLlf)1I11

I'l~

27

153.
151i.

155.

15C. Dcm uestren Ias ide lit ill (Ides:


' 51

a~+2(l-3+((l+t) lia~-O 1/0+3 15i. =--'--=~--=-~:"":'....:..!.-'-I== = I' a~-2(l-3+ (a-1) 1 122-9 1 a-3 158. .. / .r:

a > 3.
1'211 -1- <I
4

I a+V

.r a~ -11

11iD. (

(x~-I-l)
-

=
160. Hi!. Hl2.

31'-( x2 'I' x'

~/--/~=1 l / 1-1-1 L (x2_1) 1 1-7 " 7+


1

l/ r-: -a-+' la-l /r a 4 -a2-

,a

i--) x~ -1

r~=

,~l

(I;:;:~2.

'

51 X

",

.» 1.

Simpl if iquon las cxpresioncs:

Hili.
165. 2a

V H-{- (1/ f- V·+r :(~ ~- f/' ~)-1(V + V1+ l(l/i--V ~


)2), dOlldca>II, 1-'>\1

1GB.
IG7.

28

Pr iruera parte. Algebra. Capi tulo I

172.

173.

175. (

II

aJ_aG+l

-I-

-1 a3+aG +1
! I

2i-2
a3_n3+1
:1 I

, -I

-4

! a3

4. Translormaclones

do oxprt'siones exponcnciales y JogJritlllirll'1

29

§ 4. 'I'ranslormaclones idiSnticas de exprcslones exponenclales y logaritmicas ,

Hecordemos los datos fundamentales acerca de los lognritmos necesarios para resolver los problemas de este parralo, Sea a un rnimero posttivo distinto de 1. EI numero z recibe al nombre de logarltmo de N en base a, sl aX N.

P .ej .• log\! 16 = 4, ya que 2-1 16, =


<V3)-8=

~'t . En genoral,
=

logV38T

= --3,

ya

que

log"a"=r.

De Ia definicion de Iogari trno so dcsprende que, pruuero , 1;] anotacion x log, N y 40: = N expresan una mismn dependencia entre los rnimeros a, z, N; segundo, el mimero N debe SCI positivo; tercero, 51 a> 0, a '4= 1. N > 0, entonces (I)

En esencia, l a identidad (1) es Ia anotaci6n mateuuitica de Ia definicion de logarilmo; ademas, ls dan el nombre de tdentidtui logarltmica fundamental. Para todo mimero positivo N y cualquier mimero positive a, distlnto de 1, 5610 existe UD nunierc real x tal que x = log(l N. En particular, de aqui sa deduce que si NI = N 2. loga N', = log" N 2. donde > 0, N! > 0. Recordemos las propiedades fundaruentalcs do los l()gnritmos: Si!N1 ·N2 > 0, nntonces

n,

to. loga (N,.N2) = logolN1) +log" IN2i.


2". log" Si,
~1
2

= log"INtI- log"

IN ~I.

ell par t.lcular,

N,>O,

Nz>O,

tenernos: loga (N ·N2)= -logo.

= +1. +2; ... ), IogaNJl.=lllogalNI.


4°. Si N>O,

3(\. Si N > 0,

s;

log" N,

+ logo.N2'

INd=Np log"
J

INzl""7.""N~

y oh-

N,/

= logaN, -

~L

E R. log, NJJ-= ~L log", N; si N =r= O. /.L::-::' 2m (m b*1, logaN=


110gb N

==

b>O.

Esta ident.idad suele SCI' Hamada [ormula va base. En particular, con N = b, de log"b = 5°. Si
1 -1--'
O&b
(1

ago" de transicuin. a una nueella sa dosproude q lie

N> 0,

~1

E R,

logo

N = log" flNIl.

30
EXDIll

ilIOIl illS algunos


t

ejern p los.
'I,

E,rF.~!J'LO SULUCl()N

C',il J Clth;>1l10S
IlSO

,()I--i- log,2;,
~

Il acrcndn potenci a a una potcncia

de que 49 los exponeutos

= 72 y de que al elevar
se mul tiplican.

una ohtenemos:

7 EJ expoucute
I 2- 2 I\si,
S('

2-2..

IOjl,25

pucde

transformer

del modo 5

stguieu

te:

iog1 Z~ -c- 2-log1


pees, I
ht~t

5 = IOgl 49 -log7
=(
49 _101:1 -,-

= log7 49 . 5
de la ide
11 ticlarl

2 --;! lo~,25
~

_.

PI'I'O

(1) so

dud lice qlle

a=

+.
,19 ._,

De modo

que 1lJ

I ~_!_ ltlBt'la

= U.S .

I~J!::l\Il'L() 2
COil

SOLUC16N Tl3JlCnlOS

los numeros 10 y 2 (es decir, can la base dada y e1 mirnero cuyo logarjtruo es conocido ), etnpleando las nparacionas de multiplica10 cion, division y olovaciun a potencia. Como 5 = 2' 2 log 5 =

Calculamos log 25, st log 2 = a. log 25 = 2 log 5. Expresemos

el

rnimero

5-

2 Jog 2' = 2 (log 10 -log 2) = 2 (1 - a). EJI:;MPJ,o 3 Cnlculernos loga 18 sl log;) 12 = a. S()LUCWN, l-er procedunienio, Como en 01 anterior Iiquemos tog318: IOlh 18
=

10

ajGID

plo

si III pli-

log,l (3~·2)

=2

+ log:) 2.

EsLo signHicll q ue dehemos cnlcular lOth 2, sabientlo que log-'j 12 = = a. Expresemos el numero 2 con los mimeros 3 y 12 (la base dada y 01 numero, euyo lognritmo es conocido} haciendo usa de las operaCiOllCS de III III l.i plical', d iv id ir y elev [I r n po tencia.
/ TOlLeTllO~; 2 = ~ 12

T1
T

en tonces

log;) 2 -"-\Og3!/

. lit
8 2

= 2" (logJ 12 -logJ


a-'[ + -2a+3 = -2- .

3)

= T (a-1).

ASI,P"CS, ogJ I =

2-do procedimienlo. Tenemos: log, 18 = 2 log3 2. Introducimos Iii untucuin log32 = x, entonces log3 18 = 2 x. A conununcuin, log3 12 log, (3.22) = 1 2 Iog3 2 = 1 -I- 2x. Do ncuonlo COil ('I jlbllltl'1\(JlilJllto iog3 12= a, par 10 tnuto,

+ + +

1+2x=a,

dr- duurlu

x=

-:!-.

II-I

DC oste modo, logJ 18 = 2 -I-.r


Em;'Il'LO'SOLUCION_

(I-I + -2-

c• ...l-0 ---T- .

Calculemos log~o 16 si lOg1428 Emplcando las formulas 5" y 30,

= a, obtCIH>TllOS:

JOg"9 1G= loglr~o VIG = log7 4 = 21og1 2.. A continuacion, I


og 14
I ntrod ucimos

In tWO l.acion tenernos:


iog128 log, H _ -

log, 2 =.:2:,

en Lances log

~9

Hi

2x_

28 _
--

log, (21,7) log, {l- 7)

2 log, 2+ log, 7 _ log) 2+ log, i -

-I+T '
::-0 red Il11,1 11 amos

2x+ I

Como ce
11

SCgUIl I:I so I·IICIOO


.
pUBS, 5.

cl planteamien

d () 1<.I

., CCIl:tI~IOJl

to Iog.; 28 = (', 01 problcm« 2z+ 1 = a, '111 doudo r+ 1 ,_


\J

x = 2-a

a-t

Aai,

I og~o 1 G = 2 x=-""

2(a-j) 2-11

. 10g6

EJEJlIPW SO~UCION.

Calc ulomos log 1:1. 60 si

30 = u, log 15 24 = b.

Iu troduc imos la 1Illotacion: fiO 2+.x-l-y 1ogl2) = 2+x A cUlltilluaci()IJ, tcueruos: a=lo
gG iug.6

log'.l3

X,

Log2 S = y.

Entonccs,

30=]ogz30=lo!l"2(2.3.5)=
lug~(2·3)

I+r+"
I-I-x

b = 102ft

g 15

= IOlh 24 = \og2 (8.3)


log, 15

log~ (3·5)

= 3 +x

.r -1- s"

As}, pues, el prOhlClUft se reduce 1+.x+U 1+ =a, de ecuacioues: .x { .x+3 = b. Con nate sistema
x+y

n resolver el sig uicruo

sistema

hallamos:
b+3-nb
ab-t

X=

,y-

_2a-b-2+ab
lib-I

Entoucos

ogh

60

2ab-l-2a-1
ab+b+1

32

Prunera parte. Algebra. Capitulo I


EJEHCICIOS Calculcn:

182.

3) logalog, Log: lu;

c) lug log 183.


a) (~~)

y~'·ill.
'IT

b) -log21og3V:/3;
10gSI ~

1010\2~ 3

; b) (2

7)
,

Ta

184. a) 3S10g.5+lOl-10g2_3108,36;
1

b) 81Iol< • .:l 185. log

271v:., 3 ~

310g, 9.

,1) ( 2-log! 'J' 3 .1oglTJ 3'


J

2 _ 2 VIO:., 3.

186.logs7.1og75dogs'.-1-1; 187.
3) 2 Log,

b) log.2.1og.31og&4.1oga5.log16.1ogs7. b) 3 Vlog,

s_

Slog, 2;

Calcul en: :188. log 12~0 1B!}. logwllO


1~O. 1\11. -192. J93.

91

loga 16 = 11. logJ 5 si loge 2 = a, logo 5 = b. log35 28 51 log14 j = a, iOgl~ 5 = b. iog SI log" 27 = b, II > 0, a+- L

SI log~5 SI log 12 27

lug 2

= =

0,3010.
u.

·194. log-r, 3.38 si log 2 n., log 13 b. 195. log\! 300 si log3 20 = II, log3 15 = b. 196. IOg~~5 GO 91 log,~ 5 = a, logu 11 = b. 197. loge ab si log" /I = [I, log" n q, loge n: ros positives drstiutos do 1.

V3

V;-

r, dondo

II,

b, c, n son n urne-

.1/;;

-lfl8. logau

~/b si logau

n. donda a, b

50[\

numeros positivos, con La parti6. donde a, b, c son numeros

cularrdad do que ab.p L 1!l1J. log"/I( It si loga II = 2, logb n = 3 loge n posi ti vos distintos de L Demucstren las ideutldades:

b)

Iogab

10&(1

It

=1

+ logo b;

§ 5. Domostracivu

tit)

doslgunhlarles

:n

205. log at

=T

(log a+ lo~ Ii) ';1 a~ +bZ = 7f1b.


(l

206. log -4-

a+2b

1 ='"2 (log

+ log b)

Sl

112

+ 4u~ = 12ab.

SiWIJlifiquen

las exprcsioues: b) 10gb rr-1.

207. (logal!+ 10gb 0.+ 2) (toga b -logotl

208,
\o~,••

~1_-~i~o~g~~b~__
(log" b+logba+1.)
(l

\og<l

209. (b --rog;;-'a ')Og'b-/ \Q[:ab{a+bJ log,r,V«'


210. tJ,2 (2111oli,b+3b
v~ ).

log, •• b

loga b-log,r-I"b 214.


........ ___,,.::.bl--..,....-: log" b -log a b ;;< /.j

y"

10gb (a31,-I:).

i 215. 2 logo /, ({Joga 4 I' .~iIl>t, I!>L § 5. Dcmostrnelfin En antran el presents

al)

--

-1-

IOg'I, .J

de deslgualdarlos
parrafo seniti
trntndas

las destgualdades, se considcra

euya que

validez ha de ser establecida


en este. Si semejante

en el conjunto preiijado
eonjun to no se indica,

de valores que

las variables pueden tornar eualesquiera valores reales, t. Demostracion de destgualdadcs con ayuda de la deli ni d 61\. Como sabemos, por definicion se supone que a> b si a - b es un mimero posit.ivo. POl' esta razun, para demostrar, segun este procedimtonto, la desigualdad f (a, b, ... , k) > q (a, b, ... , Ie) en cl conjunto prefijado de valores de las variables a, b, ... , k se com pone la diferencia (a. b, . , ., lc) - q (a, b, .. Ie) y se domuestra que es positiva con valores prefijados de a, b , ... , Ii: (de modo aualogo este procedimlento sa aplrca para demostrar Ins igual d ades del t ipo

'f

t «:s, t >». I<-g·

PrilL1CI'<l EJEJl-I('LO 1

parte.

Algebea.

Capitulo

Dernostrumos que st a#O,

1)#0,

eu l.onces
(1) calcu-

~~- b;?--V
DEMOSTHAGION,

ab

(dcsi gualdad

do C<luchy).
a+b -r - ~V-y au

Compongamos

lomas su sig no. 'I'enornos:


( 1r-: y r-)2a- b 2

"t h_ -v au

In diferencia
=
(l-

~ab+b

= (y;;-;

YbF_

Con

todos

los valorcs

no nllgnt.ivos

de a y b 13. exprosion
si,

I)() ('8 !lPgativa.


a~
b-

pUGS, In d ilcrcncia
prccisamente, lugar
COl!

que

-2-> yab.

a+b

-v ab
._

Ella

so anuln no

y s610 si a= U. Asi,
y

es negatlva sig no
de

esto

significn
s610

El

igualtlad

tiene

EJEMJ'LO

a= I). 2. Deruost remos

que si ab ;» 0,
a U 2 -,+-;>. I 11 (2)

l1BMOSTllACION.

Tuncmos:

r.omo igllrl

ab ;» 0,

('l~l;i'.!
,1

>- 0,
--

COil

lu par ticulnr
(t

idad

de que el siguo p ues,

de

(~+ ~)-2
EJEMl'LO

ld nd ~61(, ticue

1UgHI' con
0

b. Asi,

In difel'clIci:l
(2) q uedn do....

no os uegntiva,
3,

sea,

la dl'sigulIldnd

most.l'nda.

Dernostremos

que
2

a2
DEMOSTHACI(iN

+ 4bz + 3c + 14.>
Annlicernos Ia

2a
(2a

+ 'l2b + Gc.
+ 12b + 6c),

(3)

dilerencia

(all

+ 4b +2

3ell

+ 14.) -

Despuds de reagrupar (a2


-

los tel'minos

de esta rliforencia,
2 -

obtencmos:

2a

+ 1)

.. (!ll)z +

12b

= (a La ultima

1)2 -I-

+ \J) + (3c 6c + 3) + 1 = (2b - 3)Z + 3 (c _1)l! + 1.


do a, b, c.

exprcslon es posil.iva COIl cu a lesquicra valores La desigualdad (3) quoda demostradu. EJEMl'1.0 '. Demostrernos que si a b + c> O.

aO

+ 0 +c
3

3~

3abc.

(4)

§ 5. Demoslrncion
DEMOSTllAC]ON.

lie d osiguald adcs

35

Consrdercmos
u"J

en Ia que Ia suma

Obtenemos: a3

In d ilorenc ia a·1
hastn
2 J

b3

se complota
2

el cllbo de In snrn a.
J2

+ bl +-

cl

::labe,

+b +e
3

3-

3abe = a3

Iactnres,

3ab2

Despues

de

obtenemos:

+ 3a b -I- 3(1b -I- b + c 3(t b 3abe = (a + b)3 - 3ab (a + b + c) + c dsscomponer l a suma de cubos (a + u)3 + c3
3•

P.lt

(a+ b)3+ e3- 3ab «(1 b+ e) = «a b) c) «a+ 1))2_ - (n + b) c eZ) - Bab (a b c) = (a b c) (a2 2(!(; + +b2_ ac -oc+ c'l_ :3ab) = (a +b +c) (a~ 1)2+ ;:2-

++
Z

++ ++
2 _

-a/;-

bc- ac),= ; (a
=

+ b + c) (2a + 21P+ 2c
c) «a-

+-

2ab- 21x- 2ac) =

(a+b+

b)2+ {a- c)Z+ (b-cF).

d e acucrd o COli el pl antearniento ,a b c ~ 0, lu ohtenida no es negative. De aqui se despreudo In val idez de In desigualdad (4). Seiialemos que el signo de igualdnd ell In desigualdnd (4) tiene lugar cnando a -I- b c = 0, aSI como al SCI' Como, expresi6n

++

2. !I'ictodo slnletlco de dernostractdu de dcslgualdades. La esouc i a de este matodo consiste en que con una serio de transformaciones, In desigual dnd a demostrar 58 deduce de ciertas desigualdarlos conocidas (de relerencia), siguiontcs Como estas a) ult.imas so puerlon b) d esigualdndcs:

b = c.

a2~O;

a1

utll lznr, p.ej.,

las

IJ

y ab,

duntl c

<l;?-(),

1/>0; c)
a>O

:+~~O,

dourlo

uv>O;
que si a~O,

d) ax2+b.r+(">O,
b~O,

dcudi-

y 02-4.oC<0.

EJBl\lPLO 5.

Domostrernos

c;;;?O, d;? 0, eu tonces

(5)
DE.MOSTHACION.

Tomemos
Q+b

como
c-l-d

desigualdad

de referouciu

la de

Cauchy:

-2-+-2- ........ l/o1+b'


2
::P

c-f-d

22'

Como,

511voz

a+b

rt: :r-~ V/ Ill;

-2-~

c+d

Vcd,
t"

c+d V,I ab . 11 cd= / -y-'-Z-> 1a-l-b y V

~' a c .

bd

JG

l-r imera

parte.

Algebra.

Capitulo

ESLO

sig nificu,

(1+ Ir ,-j-d -.-+~


2

:l

;:;;:. V abed, •

1,,--

a+"

' 1;+ II+c -+ d A SI, pucs, ---t,,--;;;'" hl-'::-d ave. AJ analizar lu demostraciou l lugamos H Iii conclusion de quo 01 signo do iguald.id ell la desigualdad (5) s010 pucdc toner lugar si,
y s610 si a=b,
F.JEMI'LO

P oro, --Q'--- =

-Y+'-Z-

c+r1

a+b+c+d
4.

c=d

at

C~d,

es decir, cuando a=b=c=d. tlonde nEN, Ins siguientes


(11-2)+3-...
2

c. Dcmostremos
do Cauchy:

que

(ati)n>nl.
touieinos
-1).:6'

n>1.
desi-

DEbIOSTllACIllN

Como de referencia (,,-li+2


:l

guaidades

n+l_",Li---:-Y, 2 ~y n,

"'1/(' .;d3y

'V (ll-<:o)'

'J

2· .. " ,2+(rI-1) ;;;;:: , Z

'V2 • (n- I): --y-~y ), 1+"


obtensuios:

1/-1-n,

M ul tiplicando

ostas n desigualdades,
1) (n-2)
-=

(ntt r~V(n(n~

... 2·1) (1·2.3

... (n-1)n)=

V nlnl

V (111)2

Asi , puos, (

1\~1-1)">nl
ue
SCI'

= nl
(6) de las

Como, do acuertlo
desiguul dades

con el planteamienlo,
s610 pucde

n =fo 1 l a primora
estricta.

do Cauchy

Pero , entonces,

incluso despues de Ia mul tiplicucicu las desrgualdades de relereucia, In dcsigualdud (G) obtenida debe ser tnuibidn estricta, De forma
11.+1 que ( _.-2- )" :::.">
EJEMPLQ 7
ri ,

quo es 10 que deseahamos

demostrar.

Demostromos

que si a

> 0,

> 0,

> 0,

entonces (7)

DEMosrnAcrON

Como do referenciu

tcmemos

las siguientos

dssi-

gualdades:

§ 5. Dernostracidn

de deaigualdmlos

37

cuando

(estas dosigualdades se conviortan en igualdades en aqucllos cases a=b, a=c y b=c, rospectlvamente). Surmindclas, obtenemos .!!..+~+~+-=-+~+~26 6 bien b+c a+b 2G.
b a
C
(1

>"

abc

+~t+

-s-:

A continuacirin, nas:

realicemos

una serie

de sencillas

trnnslormncio-

Sacando

a+b+-c

de entre

parentesis,

ohtonemos:

(a+b+c)

(!
1

+! ++»9.
+7<
t

EI signo de igualdad s610 tiene lugar cuando a = b = c. Ii:JEMl'LO 8. Demostremos quo S1 n E N, n> 1, entonces

T+Q+""1'O+'"
DEMOSTnAClON_

1.

(8)

Tcncmos:

Sumomos cstas (n - 1) desigual clades y

0 btencmos:

T--rg-+1if+ ... -I--;a<T'2+


= 2-1 -I- '3-2 -I- 4-3 -I1·2, 2,·3 3·4

111

1
2.3 •.•

+-

+3T+'"
=
1l

n-(1I-1) (n-f) n

(1 -~)2 +

-I- (n-1)7I

-I-

(~-_!_) 2 3

-I- (~-~) 3
1+1 4 2-

+ ... + (_1 _!_) = 1-J.. < 1. __ n-1 n


+1

A'SL, pues,

1_._ +16 ' ...

nr <1 .
!J1l-

3. !'t'ictodo de demostraci6n de desigualdades a 1.. lnvcrsa. \ EJE!I1l'hQ e. Dcmostremos que si a;::: 0, b;;:::O; c>O, d~O,
touces

(f)
OEMOSTHACION.

la desigunldacl (9) no es clsrta, Y(a + c) (b -I- d) < Vab + Vcd.

Supongamos

que can ciertcs


0

sea, se cumple

valores de a, b, c, d

Ia dssigualdad

·38

Pr

11II

ora p nrto. AIgeb ra, Colpil ulo I

Como los dos III iembros de est.a dcsigual d ad no son ncga ti vos, al elevarios 0.1 cnadrado, oulcnemo.s; ((1 do dondC' be
que nuestra
EJEMl'LO

+ c) (b + el) < ab + cd+


abed
1

2 Vabcd,
be

+ tu] <: 2~/


10

y, a con t inucoion,

ad

< V (be) . (ad).

Pero es to con tr a d ice Ia d es igual d ad J e en uch y . Es to sign if ica su posicion no es ciarta y, por 10 tan to, es val ida Ia d esigualdad (9).
Dcmostremos a-I-b-l-c
3
DEMOS'fIlACII)"

que si -. /
~

a> 0, b > 0, c > 0,


.

entonc

es (10)

fl~+b2-1-e~ 3

desigualdud

Su pongamos q lIC co 1I ciertos valores de a, b , c III (lO) no cs c icrt a , es decir, so cumple Ia desigual dnd

Al clev ar 1\\ cuu.l ;:1(1 sus dos miom bros, o ( >'l-l-I,-I-c).


3
Z ..

0 h tenouios:

>

a2+b~+e1

y, H COil Li II lIHI..'!iJJI , (a+ b cF >;3 (al -I- b:!+ca), :~ (a2 -1-[;2 c~)- {rL-I- b -I- c)'!. < 0, :~ (aZ b'l. -l- CZ) _ (aZ /;2 CZ 2ab ~ac 2fJc) < 0, 2a'! 2//1.-1- LCZ - 2a/, - 2ac - 2hc 0, (ab)2 (b - c)Z

<

+ + (a-c)2 <0

+++
<

La ul Li mn dcsigualdad

no es cierta,

ya que la suma

de cuadrados

no puede ser IIIl nurnero ncgativo. De forma que nuestra suposicion t.ampoco es ciertu, por 10 que sera val itla In desigualdad (10).
mos
OI!SgllvACION. Scan CII In conaidcracuin

dados
las

/I

ufuncros

positivos

cj ,

siguicntcs

varl abies:

a~, ... , a". Introduzca-

If .,,= I·

-+-.:..... + (J,
(Jl

1 ..If

i- --,UN 1"ia
Ill'oiIO
.,.

• anncuicn,

(! II

G n =1 A"

! 111 • '/2'
"I

+ a ~-~'
est.is

.•

. (1)1 -

nuxl ia

rclonal.
.

. , . -I- fI"

It

met! ,<I a rJ Lmelll'a,


-merl ia cuudrut icn de los uumcros al> a ex isle la siguientc
'!n"';;;

Qn.=

1/ (I;;+II~-I~,".+0:,
V

l, . ",

an.

Entre

ill'

iit Lies

dcpendencln:

tt ; ~ Gn"';;;

QII'

(..)

§ 5. Demostracton

de desigualdades

3~1

Al~un05 do los cases particulates de estu dcpentlencia ya han SIU!)dcmosu ados. ASI, en i05 cjemplos 1 y 5 (pag. 34 y 35) Iuoron dcmostrados las deslgualdades G2 :;;;;; A y G~ ~ A~: de In desigualdad demostrada en I'L ejornplo 7 (;lag. 36) sa desprenJe In dependencia II 3 ~ 113; por fin, en el cjem plo 10 (pilg. ::l8) rue demostrada la desigualdad A 3 ~ QJ.

4. Demostracion
lila

de deslgunklndcs q ue si

segun
It

01 r1\(Hoilo de inducdon

tematlea.
l;:mMPLO u.

Demostrernos
2"

E N, n ;> 3,
(tt)
3

>

2n + 1.

Can n: = 3 In desigualdad (11) es ciorta: 2 > Supongamos ~ue (11) so realiza con tt = fc (k > 3), es decir, supongamos que 2 > 2lc 1 y dernostremos quo, en tal caso , 1£1 desigualdad (11) so cumpla, asirnismo , call 12 = Ie I, Q sea, demostremos quo 2ft+! > 2k -I- :~. En olecto, tencmos: 2" H = 2 ·21<> 2 (2k 1) = tll( 2= = (2k -I- 3) (2k - 1). Asi, plies, 2h H > (21£ + :j) C!.lt: -- I). Pero '2.k - 1 > 0 co II ell nlq II ier valor na turn I de Ii, POI' 1:.0 11:-;ignienLo, COil mayor razdn 21<+1 > 21£ + 3.

> 2·3 + 1.

D£MOSTnACI6N.

+-

Ilegar a la concl usiuu de que Ia desigualdnd (1 I ) e,~ v.i l Uli1 n 3. EJEIIWLO 12. Dcmostremos que si It EN, entouces

Do ac uordo con 01 pri nci pie do

nrl ncci bn ma temii l ic a, potlumos


CO!l

>=

loti a

1+2-1-3 T+
DEMOSTHACION.

... +
qlle cntru

1 2"-1

>2'
Jlllt'lIllJru

/l

(12) de

La exprcsion dosde

In dosi gualdad
son numeros Supougamos

(12). es In suma
naturales numerrcu

ric Irncc iunes,


2" -

ell cl jli'IHI('1' ell yos de

1 haste

nomi n adores '1. Cun n = 'I so reduce


('.011

a In dosigunldud decir,

cierta que In elesigualdad

'I

> {.
(12) se real iz a
u

11"

os

valida con

Demostremos
It

que, en tal CHSO, In desigualdad = Ie i, 0 son,

(12)
k-I-I

(~S lllllll.iiull

8'1+1 = 1 + 2'+ 3+ ... En decLo,


t ... + Zk+1-1 )
SI<+1= (

:t'<.1-1

> -2-' ,

1 +2+3+

1
PI,

21<-1

t)
t

+ (1 21'+

1 2/'+1

+ ..

= Sk

+ 1\, donde

='1h

1 1 + 21<+1 + "".+ ~"H_l'

40 La exprcsurn
cua I os os mayor
1
Ph =

Prtmora par to. AIgcbra. Capitulo I P" es Ia suma de 2h fracctones,


que
1 1 21L+\'

cada

una do las

A 51, pues, ' -I= 1


2hf'l-1

Th-l-2il+1

+ ...
L

>

1
2h+l

-I-

1
2h+1

+ ...

• -,

2h+l

2h

1 • 21tH

="2'

Do forma que 811 >

~. Ph>

~ • Pero , entonccs
0

~ :>. k 1 S 11+1='-'1>+11<>2+"2=-2-' k+f

sea,

/,+1>-2-'

k+{

De acuerdo COli <,-1 principio de In inrluccion matemat.ica llegamos a Ia concl usiuu d l' q lie 1u desigunldad (12) es vul id a para tad a It E N.
EJERCICIOS Dcmucstrou las ; iguitlltcS dcsigunldades: a~ i ZIG. --~217. Si a> 0, b;;;;"", 11a~ +"b;;;;.,
218. Si a>(), b>(I,

:I.+a'

2'

a4b-l-ab'.

219.

st a+b ~

0, a

~+ 1~_;;;;. l/b. }lro:+ y 11 a


Il, 11 =1= 0,

V+"'R3~ 7+ T'

It

220. 5i lI.+b ~ O. aI, 1.It+b) ~ a~+b~. 221. al+2b1+2ab +b 10 > O. 222. 1 +211.'> a2-1-2aJ•

~8 a a2ZIj. Si a ;;;;'-1, a +1 ;;;;. Z+a. a 225. a':l.+b'J+c~+3;;' 2 (il+b+c).


II. J

223.

s:

a4=-

2,

!+It >

226. Si a+lI ~

n, aL~b3 ~ (

227. a~ 230.81

b' ;;;:. ab 228. a' b~ ;;;.a3b 229. (a+b)4 ~a4+b". donde ab;;;' O. a<b<c,
231. a8-ab+a2-a+1=O.

(.lib)

3.

+ abs.

a2b+b~c+c2a<a':l.c+b2a+clb.

232. Si a~n, b>ll, c~ll, 233. Si m, n, k son lIurncros


a ~ 0, b;;';' 0, c ~ 0, 235.5. a;;'I.I, b~U, .::;;;;oU, 236. 5i a;;';'l1, b:;;;'U, c~lI, 237. s: u;;;:.n, b;;;;'ll, c;;;;'\1, 234.

a+b+c~l/,1ij+-llbc+-Vac.
!I110 turales,

si

mil

+ 1Il/' + nl« ~
~ 8abe.

3mnk.

(a+b)

(b+c)

(a+c)

a+b+c=1, (1-(t)(l-;)(i-c)~8abc. (a+f)(b+1)(c+a)(b+c)p1tia~c. d;;'ll, a"+b4+c~+d~~4,!IJrd.

§ 5. Dcmcstrncionjde

desigualdades

239.

st

+ ...+ Yala + VU a +··· +- {a.an+


ll

til;;;;'

0,

til!

> 0,
2.

"',

an

> 0,

If ala.+

";;;-2-(at+az 241). log23+loga2>

Tl-1

Z3

+'

..+(1,,).
2 l

\/ a)a3 +... + r'a"-l(j,,~,::~

a~+2 l/a~+1
Vi3. Si a>O,

>2.
b>O,

242.

Va +a+1
be

c. +a+2

;;;'2. ab

2115. Si a>O, 1.1>0, 1+a+b'<

c>O.--I--b +-~a-I-b+c. a C Z44. Si a;;;<:: 0, 0 ~ 0, c;;;,. 0, rlO(a+b) + be (b+c)+ac (a+cl d. Gal'c" a+b _ " b
l+a

ac

+ H-b

2iJG. Si all a~. "', an son nurneros posit ivos, con 1I1·(lz·... ·an=1, (i+al) (1+az) (I+an)~2".

la partrculur hl.ul riP !I'I!.'

247. Si Tl=2, 3,4, 2A8. Si n=2, 249. Si n=2, 3,4,

''', "',

l'1+V2+
" 2

+Jf;'>II.

n! ~ n

3,4, "', _+1 1 n


-I 2

++2 + ... -I-+-> -21 .


11-,' ",It

250. Sia>O,o>O,

,1 ~lab.

~+b
"'51 ~
.

S' !

-"'0 a_,

&---(1
p,

a+', 2

~ ~

--./a'+b V 2
4 ,-

252. Si

a>

0, /1> II, b>O,

y';t+ 1ia-I-

l/ii>

1/ a+/;. ~liab.

253. Si (1)0,
2M.

2 l(al>

lfa~+b2 >-V a~+b3. 255. aZ+b2+c'>ab+ac+bc. 256. Si a> 0, b> 0, hi;;+ Vb)8 >1I3,1/> (a-I- b)1. . a+b+c .Z57. 51 a;;;' e, 0, c >= 0, 3 ;;;. abc.

I>

b>

v~I

258. Si abc=fo.O, ab+ac+l7c=/:O,

-;-+T+--;:
2GO. Si a >0, IJ >0, c> 0, d >0, 1 1
'"

1 ~]"

4 .~

a/,,'d,

a+T+7+T
suporic que "-EN.

261~. Si x>-I.

11~2,

(t+X)n>I+III.
261 - 2G8
SC

* Ell los ejercicios

Pruncra parte. Algebra. Capitulo I


202. Si II;;;;' 5, 2" > ,,2. 2(i;J, 5i 11,111, 2" > I!U. 2(Yi. '(I.I ~ S· 2GJ. 1

+"'+ .+0.
r»:

11

I ai'
1

n;;'2,

1 n<.t+'"""T-+~r-+-"
,.2

+ I a 1 +- ... + I an
2

l-

1'3

.+-r-:.
111
I'

200.

S. 11;;'
S'
I

2, Z ll'ii

> 1 +1;-2+ .

,;,+ ... + .;-.


3 .
11

:w-

I •.

1 j 1 "? 2'-1-1 ---1-"c: 1l/l-

+- .... + -21n
1 2"-1

13 >2" '1

.11 2G8. 81 II;;;':

2,1-1-""2+3+"'+
de los valores

<It.

§ 6. Compnraclon
numdricas

de las expreslones

Si son dildos rl as n umeros rcalos, ell La mayoria de los casus cual de cllos es mnyor sc ve dH iumed iato, p.ej., 8> 3, IT> V5". Es facil ostablccer que < 100U. gil c lecto, 5" < 2, mientras quo 1000 > 2, 0 sen, 5' < ·100U. Sea ahora a = V 3, b = "V 2. Los dos II UIllCl"OS pertenecen .11 i11tervalo ] t ; 2[, pcro emil de eHos es mayor 00 esta clare hasta el

i/5

11

11

Hcernns Jus sig n ien Los razonumlentos.


quo dad

momon to. Para

es Lablecer

al signo

rle deslgualdnd

V3>V 2.
al soxto

Supongnmos

quo

ell tre ellos,

a>

b,

sen,

reu-

Elovaudo los dos m iembros gratia, obtenernos 3)G >

\t

(V 2:)(;,

do Iii ultimll desiguales decir , tl> 8.

Como \J > 8 es una desiguul dad cicrt a, l a III b ieH 10 sora In Llcsigun ldad equivaleute a ella a> b. Si hubieseruos supuosto que a < b, hnln-iamos olitcn ido: a <

Asi, pues,

a> b ~ (VB)O > eVZ)G ~


< (vJ:n°

9 > 8.

< b <;::;>

(V3)G

¢;>

110 e ier ln , 10 tuisnio 10 es a post bil idnd . a > b.


EJEMl'LO \,

< b y,

9 < 8. Como 9 < 8 os tina desigunl dad ya que a =F b, ~LOS queda una sola
a

Comparomos 2) a-=log\
1) Supougamos

los nurneros 3,b=log3t,j;

y b si I) a=Y2l3+ 3) a=loga3,b=log32j VliO.


hacienda Entonces,

V6",

b=V13+yf17;
4) las
It= SOLIICWN

Vi) + "V;)O+
II

T VclU, b=

ito+ V:W+
que a

desig

n I dades numerlcas,

obtonemos

> b.

uso de

sucesi vaments:

(V 20 + iTI)2 > (V 13+


i+~/'15G>

V 17)2,

32+ 2 ViSG

> 30+2
V156>32.

V221,

V221, (1+V15G)2>221,

§ 6. Com p araci 01\ de las

ex presiones

1l1·IIlH'rIr:,] 3

Asi, plieS, a> b <:;:> V"15G > 32. PeroY 15G moras inic i ales estan, [lsi m ismo, I iga dos con el gual.la«, es decir, a <, /J.

<:. ::\2,
IllI~!110

0 ,'it''1,
::dg!1PJ

los ruid e dcsi-

:3<log! '1=0, b=lugJ1,1:>logJ1=0. "2 "1 Es dccir , a < D. b;» 0, asi , piles, a < b. 3) 'I'encmos: a=log23>log22=1, b=logJ2<log3:1=1. De modo que a> 1, b < 1, 10 que sig nifica que a> b. 4) Prirnero est imemos cada uno de los sumarnlos de la SU!lHl a con U 11[1 precision has ta 1. Tenemos; 2< < 3, 5 < :30 < 7 < 1/50 < 8, es dscir, 14 < ~/5 30 1/ 50 <:. 17" 1\S1, Pues,

Z) Tenernus; a=logl

14<a<17.

-1-·Y

V))

e.

Estimomos ahara carla uno de los terminos de Iii SUIIW /) tambien COl! una precision hasta L 'l'cnomos: :.1 < l' JO < <'1, '" < <1/2U<5, 7<VGO<8, 0 sell, 14< VlO-I-Y2U-J-)/f.i,U< 17. Dc forma qllO 14 < h < 17.
Vemos lJuo las est imac io nes obt cnidas no perm uou eOmp,UHI' los rnimeros a y b. Por esta razon, au meutemos La prccrsrou de las cst imncio nos. 0 sea, estimemos .a y b COIl una proe.isi on basla U,1. Tonemos

2,2<)/5<2,3
+5,1

3,1<1[10<3,2
y -I- 1,4 <1/2U

< V3U < 5,5


7<V50<7,1 14,H < a < '11i,O

< 4.,5 < 1:>,;"1


15,~d, 10

7,7 <{GO 15,2 <!J


ell tanto que

< 7,8

Asi,

pues,

II

E lJfl,(j;
qUI< a

que quiere decir


H.n;:~tPLO2.

Dispo ngn mas ell orrl en d e crcc inucnt o los 1111 \) l'OS III a = iog2 3, b = logo \), c = logo 17. SQLUCION. Comparemos los numeros a y b. Esto cs posi blo de raali znr co n dns proeecI im iantos. l-er procedimienio. Tenemos log2 2 log~ :3 < lOK~ Ii, 0 sell, 1 < a < 2; logu G < log~ \J < IogG 3G, es decir , t < b < 2. Los II Iimeres a y b pertunecen a I in tervalo 11; 2[. Cr.ll1lPHI'UlllOS cads 11HO de ullos eon el punta medio del iI! tnrva l o , (I S1.'1I, co 11 el

< b.

H,91,

b E )15,2,

<

ninnuro Supongamos
3

que Jog2 3>

T
'{

en toncos

tondremos

sucosi va-

mente: 3> 22 ¢::> 32> 23 ¢:::> g> S.


Pero, como

a> ~ <::::>

g>

8,

a> ~

es una desigualtl,ul

ciortu.

44 Supongamos

Primera parte. Algebra. Capitulo I

que b >2'

entonces 6
2
3
¢:::;-

logo 9 > La ultima


I':

4~

9>

92>

(}3 ~

81 > 216.
. :3 2
¢:>

doslguuldad

es incierta,
.. incterta. 3

peru, ya quo b>

8t

>

3 > 21 v, b > T

.. cs asunismo 3 a> """"2

DC
que

IIlOC 10

que, b

<T . 3
quo

ASl, pues,

, b < 2""'

10

signific"

a> b.

2-do proceduniento, a-b


=

Examinemos

la dilerencia

a- b. Tenemos:

log23 -lag6 0 = logz 3_ -

3-

Ijog2 = og1 6

Es decir, a> b. Comparornos los nurueros que ef'ecto, ;

_ JOSh3·1tlg,,6-21og; loth 6

logz 3 [log16 -2) log! G .

>a

c. 1\15.:;arriha
O!I'O

homos est ablecldo En

< a < 2, m mimero c log5 17 < 10gb 2S = 2.


3

tambien Por

yace en estes Iimites, Ia do ,

T = log~ 5 = log~ Comparemos

V-- < log's 17. 125

J +'; 2 [ , es duclr , con


Supongamos que de las desigualdadcs,

los numeros

aye

en cl p un to modio del intcrvalo Entonces,


7

a>
~

+.

ul n umero

+.

emp leando Ins propiedades

obtcnernos sucosi vnmcn te


7

log~3>T
Pero, en

3 > 24 ~

34> 2' <:=> 81 > 128. 10 que sigriifica que a <"4 .


7

renlitlad , 81 tIlJO

< 128,

Supongamos

7 c> T'

log, 17>
La ultima

+
1:5

Eutoncos
7

<=> 17> 5/;-<;:;:> 174> 5'.


ciarta,

dosigualdad

10 que quicre decir que nuastrn

suposlcion tillnbicn 10 es:

c>

T'

§ 6. Comparucieu

de las expresioncs 7

lIU1n~ric(l'

Asi.

puos,

a<T'

c>T

y,

por

10 tanto,

a <::'c. Du uunlo

que li-c;«

<c.
EJERCIClOS
los nurneros
(l

Comparcn

y b:

269. a =~/5 b=~FG; 1

270. a= 111.7, b= 1(26+ I/G. 272. a=G. b= If:'lI!;3

271.a=1+-;_. b=Z(jI2-1). 12
273. a=

"'i+tif

\/2

V 9- ViS,

b=

l5

,/"'2.

274. a= V79+t!26, b= V84.-V28. 275. a= 1/"3+ V23+ V53, b= j/"I3+ I'C33 276. a) 0=log4 2, 11=10go.oU&O,25; 1 b) a = log, 5, b = Jog! 25

+ 1/43.

16

277. a) a = log, 26, b) a

b = loge 17;

= logt V3, b =logt VZ.


2' '3

278. a) a = log~3, b = log, 8; b) a = log~ 10, b = IOg16 72D. 279. a = log, 14, b = log, f8.
280. a = log~o 811, b = log5o 640. _ 281 . a- Jog5+log 2 I/"7
I

/:)-1
-

og

5+V7 2

282. a=3 (log 1-10g


283. a

5), 1

b = Z (-}- log

g-+

log

s) .
C,

1 =-1--+ og. n

284. Dispongan

b=2, log •. G 11: en ardell de crecirnleuto

/:)=log83, c=

V2, d=log\5.
4'

los numeros 11, /1,

d si a

= log.

I,

Capitulo
SOLUCION DE ECUACIONES

II
Y DESIGUALDADES

§ 7. Equivalencia

de ccuaciones

t.iencu rakes. P.ej., Ins ecuaciunes log x = 0 y = f SOIl cq uivalantes [carlu Illla de ell us t icue un a sol U ru IZ x = 1); SOli equ ivalen tes his eCH<\C iones 2"(.>::-1) = 1 i' = 3.' (cad a una de el las t ieno dos raices: 0 y 1). Si cada una de las ruices de In ecuncion I (x) = g (x) es simultaneamonte n\l,. de la ecuacidn II (x) = gl (x), oLtenida medinnte ciertas LranfifOl'lnrlclones de In pri mcrn, In ocuacion 11 (x) = gl (x) l levn 01 nombre de corolario de In ccuucion I (x) = g (z). Asi, puos, In ocuuc io n (x -1) (x - 2) = 0 es cl corolario de Iu ecuncivn x - 1. = 0 (pcro estu no as corolario de In ecuaciun (3: - 1) (x - 2) = 0). Si cadu \!Jla de dos ccuuciones es corol ario de otra de el las, semeruutos ecunclonos SOli oqu ivaleutes. Val'ios ecuaciones COil una varicble rcciban 01 nomhre de cOnjwtlo

conjuutos
no

Dos

ecunc ionos

reci hell el nomhre de equiualentes cuando los do sus rnices coiucidou, en part.icular, si ambas ecuaciouos

-v~

VX'

valores do la vnr iabl c, cada UIIO de los cuales sat.isfnca, par 10 monos, 1I1Hl de las ecuacio nes preli jad as. Las ecuuciones que Iorruun un conjunto so cscriben en una columna con ayudn de corchetss, p.ej.,

de ocunciunos

si so pl anton

01 problema

de hul lar

to dos

aquel

los

r 2x + 1
l4x-3=x

3:1:+ 5

(par cierto , que COlI mayor Irecuenc in , las ccunciones que forman u n coujun to ."Q escri hen en linea: 2:r 1 = ;.Ix !J; 4.x - 3 = ;rz, dlvidiendolns con 01 sig no « ; »). La solucuui de un conjunto de acuac iories consisto en Ia union de los conjuntns tie rakes de Ius ec uaciones q uc const i Lnyen eI conjuuto dado. St 1\1 ronl lznr 111111 sorie de Lt'nJ\sformac.ioncs In ecuncto n I (x) = = g (x) S8 redujo n 10. ecuacion j, (x) = gl (x) (0 bien a un conjunto de ecuacio nos), c iortns rnlces de III cunl (0 del c ual) no son raices de

§ 7. Equtvalcncia

de ccuaciunes

.\.7

(X) = g (X), dichas ruices de J n ccuacion j I (xl' = It} l-l) 10(; i Len el nombre de rakes eztratias de 10. ccuacion f (x) = g (x) P.cj., al clevar al cuadrado ambos miembros de l'J"."... -.i:, obt.enemos Ja ecuacion x = XZ que Liene dos raltes: 0 y 1 1'·,1valor x = 0 sat.istace Ia ecuacion = -x, mientras que x .0,···1 uo la sa tisface, es decir, para dlcha ec u acion es una rai I, ex LIn i!;) La ecuacion (x ~ 1)2 = X ~ i tiene dos rakes: 1 y 2. Si ambos miembros de esa ecuacion so dividen por x -" I, outc·

VI"

x - 1 = i que s610 t iene Unit raiz: :1: = 2. En casos, susie decirse que durante In trnnsformacio» de In scuacion inicial tuvo lugar In perdtda de rakes (en nuestru ejemplo x = t es «la raiz per di da»), Por regla , III resolver eeuncioues se realh.lln diversas t ra nslnrmaciones, como resul tado de Ins cual es la ecuacion dada se reduce n otra mas senc il la (0 a un co njuuto de ocuaciones). Per ('Sld r.a:t'illl, es de im porta nciu co nOCOt C lI.lles de lns Lrunsiunuacronos red 111:011 III ecu ncien dud a a l a oq niv alent C', (.IUi! cs, a 18 C'C 1,(<lC ion COf() Iario y cua les, a 1<1 p~l"didn de raicos. TEonEIIlA t. a ambos miembros de la ecuacioti f [r) = f! (x) se

nemos In ecuaclon

acmejnntes

adicioua una misnui expresion. (p (x), dc/ill.ida COli. toda .:t del cantpo definicion de la ecuacion f (x) = g (x), se obiiene la ecuacuui I (x) (P (x) = g (xl {P (x) equiualente a la dada. L'.o]., In ecuncion ;3xz -I- 2x - 5 = 7x -1 es cquivulcute a ecuac itm :1.1,2 2x - 5 (-7x 1) = 7:1.: - t (-7.r 1). que 1<1 expresion {P (.r) = ~7x 1, con todos los valorcs do z, sido clC'Ji uid a en el en mpo de dor in icron de In ecuncion :b:2 2x

s,

de

+
la

+
L

Pcro lit ee unci x" = 1 no os eq IILVI'lI on 10 a x· I La violucion de Ia equivalenc ia so produio dobido a que In l'-,prl'si()ll fP (x) = x-rue delinirla no con toda x del campo de ddl!llci!lu do In ecuncion x· = 1, sino que s610 con los valor-as or 0 L;1 n.rlic:.i':'ll de ia expresion Ip (x) = a ambos miembros de In ecn;lci(JII ;1;2 =1 ha eonducido a cstrechur el campo de definicion dlJ In rcuncion, 10 que pudo acarrear In petdidn de raicos, Ell el CIlSO dado, s: = _.j es Ia ra1-z de In ecuacicn x2 = 1, peco no es raiz de .{.1l .+

= 7x -

yo. hn
-

on

-+ Vz -", + 1/7.
>-

fx

lJ H~' q 110 compren dor con clad lint! q lie en cl teorcma I ~(II<) .~(' trnLn de una transforrn ac icn: Ia adici6n a ambos micmbros ill' In ecuacion de una misma exprcsion. En 10 que atafio n In posterior reducci6n de terminos semejanLes (si ella es posible), (.5[11 us [lila nueva transforrnacion de 13 ec uacion, La red lice irJll lit' t.erm inns inlcl al, P. ei., si a ambos miembros de x2
afiadimos

+ VX' = 1 + Vx.

semejantes

puede cond ucir a una ecuaciou


x, obtcndremos
z',

+ log

x-log x = log x -i-log

(p (x) = -log

In ecuoclon ;r.~ + 2x + equi valente il In in lcinl , yi1([IIC J,"\

+ 2x + log

que

Il(J

0/:1

oq ni v n I r. n to a Ia x = log 1: - 1

48

Prunora

parte,

Algehra.

Capitulo

II

expreshin 'F (.r:) = -log x eslii dnli nida COli tod a x del campo de definicion de in ecuacion in ici al , Poro , si en In ultima ecuacicn reul iznuios b rerl ucc io n de tcrmiJ)os sernejantes, obtcnemos In ecuaciun x2 2.r = -'1 quo no BS equivalente a l a inicial. La eliminaci6u en los dos rmornbros de In ecuaciun iriiciul de l a expresi6n log x no.') lIcv6 n la ampllnciun del campo de definicion de l a ecuacion y, COUlO resultado , pud iemn nparccor raices cxu-afius. ESlO es 10 quo ha ocun+do ell nuestro casu: 01 valor x = -1 os 10. l'Ult. de Ia ecuacion x2 2.e = -I, peru no 10 es de x2 2x log x = log x - 'I. COHOLAHlO. Las ecuue iones f {x}-I-<p (x) = g (x) y I (x) = g (x) qJ (x) son equiualenies. TEOIlEMA 2 Si ambos niiembros de la ecuacion: / (x) = g (x) se multtplican 0 divulen. por la misma expresion: 'P (x), dejinuia can todas los ualores de .7: del campo de definicion de la ecuacuin dada y en diclio campo de de/intct61l no se reduce a cera en ningiir: lugar, se obtiene fa

ecuacuin.

f (x) . {P (x) = g (x),

qJ (x) (0

bien

f (xl = g (x). ) tp (x) IF (xl

1Ia sido def'In ida por do qu ler en 01 onrnpo de definicion de Ia ecuacion dada (x 0) yell nrngtin lugar de dicho campo no so roduco a cern. Si ambos m iernbros do In ecuacion x - 2 = 0 so mult.iplrcan por (P (x) = X 3, obtonuruos 111 ecuncion (x - 2) (x 3) = 0, no

equivalente a la inicial, Asi, p.ei., si ambos miombros do In ocuuciun x - <1. == z so div iden per (Il (x) = 2, obtonomos In ocuacion = x equivalente 1\ In dada, ya que la ox presion (P (x) =

-vX' +

(yx- + 2) yX' - 2 = VX' + 2

>

de definicion de 10. ecuacicn iniclnl , lu expresion (p (z) = X 3· S9 anuln , a unque ella ha sido definida can to das l as x del campo de definicion de In ecuac io n x - 2 = O. Como es fUcil ver , en e1 caso dado In mul t ipl icac ion de ambas partes de In ecuaciou par Ia ex presien rp (x) = X ha conducido a In aparicion de la ruiz extrafia

equivalenle

II

Iii dada, ya que

COil

x = -3,

perteneciente

al campo

z:

ana logo , si los dos miernbros do In ecuuciun x - 4 = = x (V'x - 2) se t1tvitlen por 10. exprestdu rp (x) = 2, obtenemos In ocunci6n 2 = x no eq nivalcute a In dada, yo. que COil x = 4 10. oxpresion rp (x) = 2 se anula , a pesnr de haber side De modo

-3.

+ ;)

Vi +

yx- -

definida

can Ladas las

x del campo

Vx -

de definicion

do 10. ecuacien
Oil

iui-

cial. de ambns partes de In ocuacion por una misma exprosion. La posterior simplificnci6n tie la fraccidn (si ella es posihle), es una nueva transLlamamos s610 se trata In atencion del lector de una translormacion, acorca de que In multipllcacion el teorema 2 (0 division)

7. Equivaluncln

de ecunciones
dl1

formaei6n ecuacion prlmora

.lOti

do la ecuacion.

P.ej., por

ill multi pl icar ambos nuernbros


cp(,X)=2.l;

la In

-!-x=3

In expresion
LICO

reulizamos
2

translorrnacion

q ne cond

a In ocuacion 2x ~x+1 i

+ 2x =

= 6 x. IJa

.. . .. .. siguron t e simp ITicaciun d e I o r raccron 2.tfz+11 I ·2,x


U

por 2 z os
una ecua_

una nueva translorrnactcn: ella nos lleva a In ecuacion z = Gx. Semejante simplificaci6n puede tambidn oonducir cion que no es equivaleut.e a l a inicial. x!-5x+6

+ ! + 2x~ =
par
.

Asi, si ambos miembros 2 , ohtenemos

de

x-2

=0

se mul trphcan
(x-2)

<p

(x) = x-

.. (x~-5x+6) la eeuac iou ,x-2

=0

aquiva-

Iente

a Ia dada, ya que Ia expresion 'P (z) = X - 2 osui dclinidn con todos los valores de x del carnpo de tlcHuicitill de lu ecuaciun linda (x 2) Y Oil ningun lugnr de d icho campo 1\0 so nuul a, Pnro , si ell el primer miemhro de In ecuacion obtenida realizamos In simpl ificacion por x - 2, obtenemos Ia ocuaci un x2 - 5x G = 0 nc eq ui vaIente a In inlcial: el valor x = 2 es In snluciou tic itt ill Lima ocuncidu, pero 110 sn tislnco In CCtWciOIl inic inI, cs dec ir, pant ella os UJl(\ 1'<11 z extratla. La cuesucn radica en que nl simpl.ilicnr In Irncciun se produjo In ampliacicn del campo de definicion y, como ya liemos indicado, esto puede acarrsar la aparicion de rakes extruiias. conOLARlO. St ambos miembros de una ecuacion se multtpltcoii {o

*"

dividen} por un mismo ruimero distinto equiualenie a fa inicial,

de cera, obtenemos

una eClwci6n

a In dada.

xt

P. ej., al m lI11i p 1iC:1l' am bos miom bros de la ccuaciou


3

.z.-J- 1 =
te

por G,

ohtonamos

In eCWleiUII

3.1:+3= 2T+G

equivalon

TEOREMA 3 S i ambos niiembros de La ecuaci6n f (x) = g [z], donde (x) .g (x) ~ 0 con todos los valores de x del campo de delinicion. de la ecuaciiin, Sf! eleuan a una misma palencia natural n, obtenemos la ecuacuin. U (x»11 = (g (x»n equiualente a la dada. Si, p.ej., ambos miembros de In ecuacion 2x - -1 = ~fx -"\ se eleven al cuadrado , ohtenemos In ccuaciun (2x - 1)2 = (f.r - J):l equivalontc a. In dada, ya quo con t o das Ius x Jel campo do Jcfinid6n de 1a ecune ion chula (x ;;:;: 1) los dos m iembros de 65tH no son Ilrga-

Livas.
Pero , si elevnmos nl cuadrado ambos mlambros de I~l cuncion c

x-

G = V;,

ohtenernos

l a cual no podernos

af irmar que es equivalonte

la ecuacion

(x -

6p = (VX)2, respecto
a ln pref ijndn,

de

ya que

~-ozo~

50

Pnmora

parte.

con ciortos vaiores de x del campo de definicion de 1a ecuacion inlcial (x ~ 0) su primer miembro toma valor negative (p.ei., con x = 2 tenemos: x - 6 = -4 < 0), en tanto quo 01 segundo m iembro siem pre es no negattva. En decto, In ecuaciun (x - 6)1l = sa trunsforma a la forma x~ ~ 13x 36 = 0, de do nde Xl = 9, X2 = 4_ Pero x = 4 es raiz ex trafia para Ia ecuaci6n inic ial. l ndiqucmos que en el teorema 3 s610 se hahla acerca do una transIorrnacton: Ia elevacion de ambos miernbros de In ecuacion a una misma potsncra natural. La eliminaci6n posterior del siguo de ru dicacion (si osto 125 posihle) os uno. nueva transformacicn de Ia ecuaci6n. La el imtnacion dol signo de rad icacion puede conducir a Ia ampl iacion del campo de de£inici6n de III ecuac ien y, POl' co nsigulente, a una ecuacion no equivalente a Ia inici al.

Algebra.

Capitulo

II

<V::tya

1. El teoromo 3 s610 es v6.licloparu las ecuaciones sabre ul de numefos rcales, 2. Si n os un nurnero impar, en Ia enuuciaci cn del tcorema 3 es poslblc omitir In cond icion. f (x)· g (xl;;;' 0 para tod a x del campo de definicion de
campo
OBSE.flVACIONES,

In ecuacieu.

Al resolver ecuuc ioncs tamhien hay que omplear transiormnciones no indicadas en los tsoremas 1, 2 y 3, es decir, aquella.s que pusden conducir ala nparicion de raices oxtrafius 0 bien, Incluso a In perdida de raices, La causa de In aparictdn de raices extrafias 0 de Ia perdida de estas puede n SOr las transformaciones real izudas con ayuda de formulas que hacen varier el campo de definicion de la ecuaci6n. P.ej., tales son las formulas:

VnJI=Vn.
r-r-r-r-

,-

-v-b,1- /a. T=lr;;'


l;;
X

(V-)2 =a, a
I!

I ogq (.ry ) = 1oga

+ log,.
~,

y,

10" b
e rr

b, ate.

!g .r . c Lg .c = 1,

.:c 21g?
5!'11:&=

1 +tg·""2

•x

I g (x

+ y) .

.= ....,...l....::,g..,.x_+'-t'""""g~/J:.... 1-tg,r·tg!J'

Ell todos los cusus cuundo lil~ traJjsformuciones reulizudus ducen n una ecuacion que es 511 coro larro , pero 110 llllY segllriu,ld
que
('5 [<IS

CUI\-

de
n

de In resol uc ion. gJEMr'LO \ Rt-solvamos

nos h a II au us ell 1n ecuacion inicial: 2) demostrundo In equi valencia de las trunslormnc io nes real izadas de 10. ecuaci6n eo todas las etapas
In ecuaci6nV 2x

de las sol uriones h al ludas. Si esta no 51} ha efeotuado , In sol ucion no puede consirlerarse, en tal caso , acabada. ~Cilmo so ver ifican las sol ucio nes halladas? Es posilrle indicar dos pror ed imientos principales: 1) ponierulo cadn una de las soluc io-

ec UilCWIlCS

SOll

cqui valenl.es,

os necosar ia I a ver ificaciu

+ 5 = 8 - V x -1_

§ SOLVCION.

7. Equivalencia ambos

de seuaciones

SJ

Elevemos

Obtanemos:

=58-x.

2x+5=(8-V'x-1f
elevamos
X2 -

rnicmhros de In ecunciou ;.11 cund r.id». y, n cnntiuuecion , I(q!x 1=

mas adelante,

De nuevo

ul cuadrrulo:

256 (z -

1) = (,58 -

x)Z

y,

372x -I- 3620 = 0,

Xl = 10, x~ = 362. analizamos las transfnrrnucioues reallzadas. potlcmos solo afirmar que cada nueva ec uacion f ue el corol ario de la an terror. (N u hay segurltlatl en Ia equivalencia de las ecuaciones obtan id as durante Ia resoluci6n.) Pero esto signilica que en el proceso de resoluciuu pudieron aparecer raices extrafias, por 10 que las raices halladus hall de ser verificad as. VEIUF!CACION. En nuestro caso las raices lialladus se pucden verilicar co n Iac ilidad poniendol as en la ocunciun inicinl . Veril iqucmos

de dontle
Si

x,=10. Tenoruos: =8-1'10-1=5.

V2xt+5=V2.10+5=5

3-l/x,-

-l =-=

Asi, puos, COIl x =' 10 los dos miembros de In ocuacicn Illici.d tomnn iguales valoros numdrlcos, 0 son, z = "10 es 111 I'::tL~ do I" CC uacitin II ada.
Vsr ifiquemos

en lnnto

x2=362. 'I'enomos: V2x2+5=V2·362+5=27, title 8- V X2 -1 = 8-1/362-1 = -11.

Con x = 3tl2 los micmLros prlmero y segundo de l a ecuaciun in ici [II ternan d Hercntcs val ores nurnericos, es docir, .:!.' '-"0 ;-!(,2 os II na ru iz ex trafia.

A51, pues,
SOLUCION.

nuestra

ccuaci6n solo

t ienc

linn raiz:

EJEMPLO 2.

Resolvamos
Elevemos

In ocuacion li3x+ 1 = 3
ambos miernbros

+ Y x-1.

= !U.

de la ecuacron
.1-

III cundrnrl«:

3x y,
(I

+ 1= (,1 11 +
7.

-1) ~
I)

con tinuaciou, 6
VOl. 7)2

V x--l = 2x-

Una = (.2x --

mtls real iz amus In ol evacion al cundrndn: ,Ili Lr y, desp ues, 4x2 - 64x 85 = 0, de uond e ha!l

(lInOS:

cT:2

1Il-3'I'19 =--2--

VlmIF(CACI6N. Esta claro, que 10 verifiencion do 1~\5 rn ices hal lmlus, poniendol as en 111 ecuacion iuicial , esta liguda con co nsldernblcs rlificultades de calculo, Por ell 0 , clegimos 0 tro proccd im iento de verificacicn.

52

P ri mora

pa r teo A Igeh ra. Capitulo

II

s610 satislacer
11 ad

£1 campo de definicion de 101 ecuucion dada es cl siguiento: x ;;::. 1. En 61 la prluiern ol evacion al cuadrado es una transforrnaciou equivalcnre do t{\ ecuucion. La scgunda clcvacion al cuadrndo Iuo apl icada a Ia ecunclon 6 -1 = 2x - 7. A esta ecuacidn puedeu

2x -

tales

val orcs
sea,

V.x

7 ;;;:~0,

> 3,5.
sea,

de x quo sat islagun


Es f!lcll

a In

desigualq lie

establscer

1a

desigualdad
1G-3

1t1-1-~1/19 >3,5
,::;.., .,)
XI = 3. "':>

os real, mientras
X

que
2

In desigualdad
una , raiz

Y19

r-

es

flOsa. a

10-3

)119 es

ex t rana Y
EJEM:PLO

1G

+ 32 lri9

,. ,a 1 umca
(2.1:

, .. rmz de Ia ecuacion

d a d a. (2x-1).

log (x2

Tx

+ ;3) -log

nc~ol vn mcs Iu ocunc ion

SOLUCfl)N

(x2 7x-3) -log 'I'rnus] o \'\11 C mos In oc I I uc il)11 a 1(1 [0 rma·

+ 1) = log

y,

U COil Ij nunclon
:tz

z2-7.T+3 2.r.+1

:r~+7:r-3 4x.-1

tie

donde

ha llames:

x1=O.

=""
. ;)

COlliO cudu una de las ocuuciones obtculdas en una u 0 tra etapa de In resol uc ion s610 es 01 corolario de 1£1an terror, 110 pudo tener Iugar la perdida de falces, pero pudieron aparecer raicos extrafias, con lit part icularidad de que s610 a cuenta de In amplfacion del calli po do <.I efin ici un de J a ccuacion iulcial. Por est.a rn7,611 , en el prssente CllSO, us poslbl o ciectuar In vcr if icacion con ayurlo del campo de deliuicion de In ccuacidn dada quo S(' prefija con el stguiente sistema de desigualdades: V[';IlH'IC,\CI<JN

I
Ni y, por
X=

x~-7x+3>O 2x+1 >0 x2+7x-3> 0 2x-1 > 0,


in ultrmu As), Igualdad pues, del sistema no

0,

III

X'l'=

satisfacen
rakes

tiene raices.

10 tanto,

son

extratins.

1£1ecuachin

§ 7. Equivalencla

de ecuaciones

53

EJERCrCIOS Dcmuestren 285. que 13s slgulentes lf2~x= ecuaciones no tienen rakes:

1/,1:-1+

l' x~5· 'Jf8x.


J 4)

2813. ~/ :r~- ili4 = li x-8+ 287, log. (XZ ~ i) 288. 2


!Q!:zx (x+2)

289. r ",-1+1 2-x=x~5. 1 1 s 2~O. x .:1:2-16 15+ ",'-113

•r r=:";

+ log:) (x -1) + log~ (l -x ,--_ + 3 logz\x+3l = \. - [1. • ,-

= I<~·

20 I. log (1fl-x2) = 1/;+ 2!J2. Zlo!{. (x-3) = 2-.:-5. 2!l3. lix~_j-l+ Ilx~+-t=1.
JaS

}f x+ 2.

2911. r4+x~+1=I()gI2. "3 siguiontcs


CCI1!'1L'il'lles?

c: Son cq UivIlIC'I\lCS lmo.

205. xl+l = )i'i y xz+ t


1'2-1

= (;;: y
y

XZ

-1

+ V l-x=jli-I-li
y

+ If 1-:1:=

)';+ it-x.
1-x.
Y x:+l=()
X

2ft7. x3+x=O

$3+%=(\.
.t

298. ,1:2+1=0

2(ln. 2x2+~+S

~"z+3

3.t~+2z-1

x+3

2 • 1-2 +3

z--

=x

3 2+2.r.

,-.

SOO.

2xz+2x-l-3
x-\-2

g,t2-1-2x-l x -I- 2 y (lf

Y 2x2

+ 2x + 3 = 3x'

-j- Zx - I.

301.
302.

Vx+2=

yi'i+1

x+2)2=(J-,rZJ:+1)Z.
y x-4

(V"'x-2)2 = (V2x -1-1)2


(;

303. 2 ,ri--7xt=2

+ V;:)
yx:

Jl'i-H = 2x-j2

y 21(%-7x =2x

'VZX+1. + 2 Yi.
de ccuacioncs ?

3010. 2 1/:;-7x2 = 2,1: -I- 2 y -7x~=2x. l Sou cquivalentcs las siguientes ccuaclones (exphqucn In respuesta):

y conjuntos

305. (x-~) (.x +3) =0 y x-4=0;


3013. (x-4)

x+3=1I.
.1:+ .1:~3 =Il. =0.

(x+

Z~3

) =0

y x-'.=fl; Y x-!i=!l;

307. (x-4) 308. ~i.x -2 309. Y2-x


310. (x-3)

(x+

x~4

) =0
y y

x-l- T~4

y x+3=O
V x-l-3=D
loy {2-x)=O

V x-2=O; V x-l-3=U_
Y2-x=O;
y x-3=(l;

Yx-!-3=O.
log(2-x)=D.

312.

311. (2-x) log (x-3) =0 y 2-x=O; (x2 -2x -3) (x-l-l) 2


.:1:-3
f)

log(x-3)=D.
.. -3=fl; %+1=0.

Y x -2

Pr Imora pa rto. Algehr a. Capitulo


X~-[jI+() .r'-Gx+8 ~._\) (2 -1)=0 :r~-5I+6=O Y [ Itcsuelvan raicos extrafias 3[4. Z-:r xH ~.t.. Zx.-n -1 _"

II

313.

las sigulontes UC\WCiOllC5 y ch'ctii.cII expliq11cn In causa do su apar iciou.

Ia vcrificacion.

Si hay

315. x+2
3t6.

2-x +_4 __ (\ x+1 i-·x x-1 - . :r 25 t 13 2",-1+1,;1;2_1 U-t-2x' x-1

+2=

4
2X_X2 •

317. __;_ __
x -1

2_=2_

.l:+4 x-I'

318. 1+1/2.r+7=x-3. 3Ul. ;-2 = Y x-Ii. 320. li22-x-l·~1U-x=2. l 2x-7 r-r-. ,/,-r-:~21. 1- x+3-\-l' 3x-2=7. 322. r 3x-2=2 J x+2-2. 323. J/2x+ 1 lr x-3 = 2 Vi. 32>'1. log (5!.-x3) = 3 log x. 325. log (x-2) + log (x-3) = 1-log 5. 32G. log V5x-/d-Iog l' x+1=2+Iogfl,18. 327. Ilog(3X-;) __ t_ 328. log (2x-5) 0,5. og (3x~ +5) 2' log (XZ - 8) 329.log:r(2x2-1x+12J=2. 330. log", (2x2 - !<J:'+ 3i = 2.

§ 8. Eeuuciones

rationales parrafo so estud ian las ecuaciones del tipo P (x)


=

En el presents
=

0, = 0, donde P (x) y Q (x) son pol inom ios, <lsi COUlO las ecuaciones del tipo f (x) = g (x), donde f (x) y g (x) son expresionos racionales, Recordemos algunos conceptos de algebra. 1. Todo poltnomio de grado n sabre uri campo de ntimeros complejos ilene n raices compleias. 2. Six = a es la raiz de polinomio P (x), P (x) se divide entre el binomio x - a sin resto, 3. Scan niimeros enteros todos los coejicientes del polinotnia P (x) con La pariicularidad de que ei mayor es igual a 1. S i semejanie polinomic tiene como su raiz un ruimero racionai, esle es un ruimero entero, 4. Sean iuuneros enteros todos los coejicientes del polinomio P (x) = = aox'i a1x"-l (In' Sl la raiz del polinomio es el numero entero b, este es el divisor del termino independlente an ~condici611 necesaria pnra Ia ox ist.encia de Ia raiz enteral.

~~:?

+ ... +

§ 8.

ECUIlCionE.'3

raclouales

55

Seiialemos que al resolver ecuaciones entaras ractonales s610 sa raaliznn transformaciones equi valen tes, por esta rascn las ralces halladas no sa verifican; no hay necesidad de irulicar ost o en cada caso concreto. Pero , 011 resolver ecuaciones raci 011 ales f raccio nar j as, sa efectua In multiplicaci6n de ambos miernbros de In ecuacion pur una misma exprssion (eliminacion de denominadores), 10 que puedo conducir a Ia aparicion de raices ext raiias. Par ella, al resol ver ceuaclones racionales Iraccionarlas as precise ejecut ar In vortficacion. Durante Ill. rasolucion de ecuaciones recionales los metodos princlpales son los siguientes: 1) descomposicien en Iactoros: 2) introduccion de nuevas variables (auxiharee). El metodo de descomposicion en f<tctores consiste en to siguienta:

seo

f (x) = t. (X)'f2
solucion

(x). _ .•

'/n

(z],
(1)

ontonees cualquier as In solucion

de la ecuacidn

f (x)
del conjunto

=
In

0
(X) =

de ecuaciones ... ;

t, (x)

= 0;

12

(x) = 0;

o.
(1).

(2)

La afirmaci6n inversa es incierta: no toda soluci6n del conjunto de


ecuaeiones (2) es 10.sol ucion de 10 eeuaci6n Asi, p.ej., la eeuaci6n

(3)

sa reduce

0.1 conjunto

de

ecuaciones:
.:t

x'-3x+2~=O.· .. x

x+2 \l

-1

+2=. 0
Xl

(4)
= 1, x+2
Xl

Las soluctcnes del conj lin to (4) son los valores: ,0 t x3= • x~= -T-

= 2,
con

Pero , con x= 1 no queda def inida In. expresion

%1-1

:c =
(3).

o.

1 ., x1-3x+2 a expreslOn x

Asi, pues, los valores x = 1, x = 0 no son rakes de Ia ecuaci6n

En general, al resol ver Ia ecuaclon (1) segun e1 mstodo de descomposicion en factores de las raices halladas de las ecuaciones del conluuto (2) satisfacen la scuacion (1) aquallos, y solo aquellos, vnlores de z que pertenecen al campo de definicion de ln ecuacion

(i),

8JEMPLO

1.

Rssolvamos

In ecuacion

x3

+ 2x + 3r + 6 = O.
2

56
SOl,..UCWN_

Pruncra

parte.

Algebra.

Capitulo

Il

Descom pongnrnos en Iactorss el primer miemhro de In

ccuacion.

(.r

+ 2) (x + 3)
2

'I'enerncs:

= O.

x~ (z

+ 2) + 3 (x + 2)
= 0;
X Z

0 y,

mas

adel an te

La ultima

ecuaciun

+2

OS cquivalente

al conjunto

de ecuaciones:

3 = O.
,!)

~ ±i
= o.

Dcspues de resolver este conjunto , obtenemos: Xl = -2, x2 = Estas son las rakes de In ecuacion dada. EJt:;IIII'LO 2_ Hesolvarnos Ia ecuac ion X4 x3 3x2 2x 2=

VI.

++

SOLUCI(>N

ccuucion
1I0S

iulruc tuosos. Por ello, in ten tames rapresen tar alguno de los lermide In ecuucion Oil forma de In suma LIe varies sumandos de modo
quo permi ta obtener una dcscom posicicn

una agrupaci6n

Los intentos

de realizar analoga a

In hecha

en el primer miembro en 01 ejemplo

us In 1 son

que In agrupaclcn,

[osa» Oil factol"eS, sea ejecutable. Hagamos 3x2 = x~ 2x2• 4 -I- x:l 2) 2 En tonces chtenernos: (x x (2x 2x 2) = 0 y. U conttnuaclon, x2 (.:t·t -1- x 1) 2 (Xli x 1) = 0, (x2 X + 1) (Xli 2) = Nos quedu por rssol ver e1 conjunto de scuaclones: x"l. x 1=
=

0;

Xli

+ +2

o.

.+ +
±
3

+ +

«venta-

O.

De este conjunto
EJEMPLO 3 SOLtJCWN.

hal lnmos:

:t1..2 = -

i
x

Gall este fin,

r II ten

Lomas hallnr in rafz entera de Ia ecuactdn prclijada. escrihunos los d iv isores del termino ill dcpondien te:

Resolvamos

In ecuacion

+ 4x

ra ;
2-

++ ++
.

xJ.fo

± iV2.

24 = O.

a = ±1; ±2;
mos

±3; ±4;

±6; ±12; ±24.

Despues de esto, comenznmos las prucbas. En l ugar de x pougaOIl la ecuacidn dada el valor a = 1. Obtenemos: 13 4.12 -

Do modo que x = 1 no es In raiz de l a ee uaoion , Continuamos las pruobas: a=-1 : (_1)3 1_·(_1}2 - 24 =1= 0, a = 2 : 23 +4·22~ - 24 = O. AS1, plies, Xl = 2 es l a ruiz de In ecuaci6n. La ecuaci6n dada es de tercer grado , ]0 que quiere decir que ella tieno dos raices mas. Hagamos uso do que el pol inomio x3 4x2 - 2/1 sa divide entre x - 2 sin resto. efectuemos dicha division

24

* O.

.rJ+hZ
.ra~2:r:3

r-

6x" Gx:l-12x 12x-24 12x-24

§ 8. Ecuaeion •. rncionales -s

57

De este modo, x3 a 10 cual l a ccuacion


Esta

+ 4.x

2-

toma

21< = (x l a forma:

2) (x2

+ UX +

·+12i, d chi d o

(x las (IUe, de hecho, ya


S8

2) (x2

+ Gx + 12)

= O.

nI conjuu to de ocuaciones [uuu de x - 2 = 0; XZ (ix -I- 12 = O. Do Ia segunda ecuaei6n del conjunto hallamos: X2,3 = -3 + i ASI, pues, la ecuacion dada t iene las siguicntcs rain's: Xl = 2, Xz = -3 t X3 = -3 - i

ecuac icn ea equivalente

ha resuelto):

V3.

+ 13,
+

onSERVAC[ON. La ecuacicn x~ el metodo de descomposicion la surna - 2,x2 Ox:!, obtenemos


Call

-+ 4:z;~ ell

YJ.

x3
.:r;2

2,1:2

+ Ox! -

24 = 0 pucde resolverse Iact.ores. Ileprcsontandc h2 sucesivnmento: 2!j = 0 etc,

en

de ncuerdo [OI'!!IU de

(x-2}+G

{x-2)(r+2)=O,

Resolvamos Ia scuacion XS - Ux3 8 = O. SO/,lICION. Apliq ueruos 01 muLo!!o do Intrnd lIeciu1I dil 1I11:l nucv <l variable. Hag-amos y = x3• E ntonces, In ecuacion prclijarlu torna In forma: .II'}. - 9y 8 = 0, de dondo hall amos: .Ill = 1, .112 = 8. A COIlEJEMPLO 4.

tinuacion, x3 = 1;

el problema x3 = 8.

so reduce

D.

resolver

eI conjunto

de ecuaciones:

1<1 primera ecuncio n, Tnnemos: x3_·1 =() V, rlespues, f 0 ue lor [ 1 I3 ( x- 1) ( x~ +x + 1) =, .1 nonne x1- - l', x?,.J= -T±t -2-'
Ilesolvamos
Do manera
mus:

analogn,

de la segund a ecuacion
X~;,D

del couj unto hal ln-

x4 = 2;
EJEMPLO~.

-1

±i

VI.
= o.
linda

Resol vamos

In ecuaci6n

(XZ
SOLUCWN.

+ x + It):!
Hagumos
.II .111.

-I- 8x (.r~ -I- x

torn a 10. forma:

x2

-I- x

+ 4.

+ It) + lSx~
= O.
con

Ento nces In ecuacrou

+ 8.ry + 15x2
como

ResoIvamos

esta

ecuacicn

cuadratlca

rcl ac ion

a y:

.11',2

-4x ± VHh2_15x~.

De modo que .Ill = -3x, Y2 = -5x. Asi, pues, el prohleru a s(' red [lee a la res 01 ucion del sig uiente conjun to de ecuaciones: x'J. -I- x -I- 4. = -::Ix; x'J.

+x

-I- 4. = -~'X.
=

De este conjunto hallamos:

Xl•2

-2;

Xl•4

-3

± ~"!1.

58
EJEMPLO

Primern parte.

Algebra. Capi t ulo II

11asol V emos In ecuaokin 21 a;s x2 - Sx - 1 = D. SOLuemN Las ecuaciones ell las que e1 primer miembro es un poHnomio can coeficientes enteros y termino independienta igual a 1 6 -1. se trunslorman con Iaclltdad en ecuaciones reducidas mediante Ia division term iuo a terroino por .:r a la mayor potencia (os racil vet" que semejants division no conduce a In perdido do raicss, ya que x = 0 no as raiz de 10.ecuacion, donde e1 tarmino independiens. te as dlstinto de cero) y In posterior obtencmos:
1 5 + -----z~ z

sustituci6n
i
x3

de ~I par y. En
XI

nuestro ejemplo,

21

= O.

Haciendo i = y, llegamos a In ecuacion 21 y - 5y2 - y3 =' 0 x y, a continuuc ion, y3 5yZ - y - 21 = O. Hullando, segun 81 metodo de pruebas como en el ejeruplo 3, 1[1rniz entera do Ia ocuaciou Yl = -3 Y dividieudo 81 polinomio y3 5yZ - y - 21 pal" y ohtenernos el trinomio de segundo grade y2 2y - 7 con las raices de segundo g ratio y? 2y - 7 con Ins raices Y2.J = -1 ± 2

+ at Yi.

Pue~to que x
EJEMI'LO 7 SOLUCION

= y'

XI

= -"3'

a•3

1±2Yt
7

Resol vamos In ecuaci6n 4_x3 '10x2 - 14x - 5 = O. Aqui vamos a emplear un procedirniento mas para tranalormar una ecuacion no roducida en reducidn (e1 objetivo de tal transformacion esta claro: Ia ecuacion reducidn tiene como raices rnciouales s610 mimeros enteros y nosotros tenemos procedimientos para hallar raices enteras). Multipliquemos ambos miembros de In ecuacion Inicial por un mimero tal que el coaliciente de x3 sea el cuba de cierto mimoro entero. En nuestro ejemplo, tal factor puede sor el uumero 2. Multipliquemos ambos miembras de Ia ecuacion por 2:

8x3 Ahora, hacienda

20x? 5y2

+ 28x

-10

= O.

y = 2xt Ia ecuaclen y3 _

+ 14y -10
ojemplos, 2

LOUla la forma: =

O.
las raicas de Ia
:t= ~,

Como
ccuacion

011

los

anterioros Yl = 1,

reducida:

Y2,]=

± i V6, Siendo
Xl

hallamos

las raices

de In ecuacion

inidal

son lag siguien tes:

=. ~ ,

§ 8. II: cuacl
EJf5.M!'W 8.

0 n es

rae lonalss

Resolvamos

Ia

ecuacion

(5 )
tlene una interosante pecul inridad: y sl t6rmillo indepeud .onta y el cuadrado de Ia raaon entre el segundo coeficiento y 01 pcnulumo 8011 iguales entre si. Las ecuaciones con .seroejante peculiaridad recihenel nornhre de reciprocas, En sste ejemplo vamos a mostrar el procedimienl.o para resolver una ecuacion reciproca de cuarto grade. Dividamos ambos miembros de Ia ecuaci6n por x2 (esLo no COIIduce a la perdida de mites, ya que el valor de x = 0 no es wit, de Ia ecuacion dada). Obtenemos:

1<1razon entre su primer coeficiente

SOLUCION.

La ecuacion

dada

3x~- 2x + q - ...!_ + _g = 0 2
x
.:r

y, a

con linuaci6n,
3

(xz + x~ ) en tonces

2(

IIagamos .
,:c2. x x

+; = y2_ 4. Sustituyendo x2 + 4 por y2 - 4, obtenernos:


.

.z: + -;:= y,

x + : )+ 4 (2x +-;-.) = yZ .
2

O.

(6) [al! to,


yy

y, pOL' 10

011

la

ecuacion (6) z
4) - 2y

3 (y2_

+ 4 = 0,

+ 2_ X

par

de doudo

VI =2,

Y2

= -3'
el problema. se reduce

Ahora

a resolver

el conjunto
4 -3-'. -

de eeuaclones:

2 2 x+~~x =2; x+-=

De oste conjuuto
Estas son

hallamos:

Xj•2

=1

±. f, xM
(5).
2

=-

3 ±.~J-

l-_r~

las raices
9.

de 1<1ecuaci6n

(t) EIEIIlPLO

R eselvamos Ia

eCUo.CLOn

..

:z;

Oxz + (.:;:;+3)2

27 .

SOLUCW!'f. EI primer mismbrc es una SLIwa do cuadrados. Esto lleva a Ia idea de afiadir a ambos micmbros de l a ecuaci6n una expresion tal, con 1a que el primer miembro se convierta en ol cuadrado perlecto de In suma. Asi, pues, nd ic iorrando a ambos m iombros do In

.• ocuacron

1a oxpresiou - 2 x x+3 .. 3x 1
(X-

b tenemos:
6~

x··1-3 .

_ax

.)2

=27-

x+3

GO y, a con t inuncion,

Prirrrera

parte.

Algebrn.

Capftulo

II

Ahora, y2

hnguruus

Y=

x+ 3'

XZ

ElltO!lCUS

In ecuaclon

toma

Ill. Iorrua: de
eCHU-

Gy -- 27 = 0 Y. de aqui, Yl = -9, !J~ = 3. El problema se ha roducido a In resolution del conjunto clones


x2 .1'-1-3 = -9; x+3
.1:2

=3.
=
g. 3 Y3 -7 ± L -2-'

O e 1a prunera
seg undn,
ccuaclou
.rM 2 =T

" ccuacion
-I- -2-'

Ira II 11III OS
y,

Z,.2

! (C

IU

in

COlt" icivll

iu ic.ial.

.r +:3

*"

V5

'I'odos Ius val orcs hal lndos


POl'

snt islucen

co II siguicu te,

SUII

las raices tie In

EJlmCrCIOS
Ilosuclvau las sigllielllcs ecuacloucs scgun d mCLodo tic dcscomposictou Iactorcs : 331. ;:4--1 =0, 332. x~-r.4=O. 3:33. x~-I- W=O. 334. 308+1 =0 335 r:l+x-2=O, .. 330..• 3-4x2+x+G=O. 337. x~ ~lxl 23x 15 O. 33M. (;r - 1)3 (2x 3P = 27x3 8. 339. 2x' - 2iJ;3 71x~- 105x 50 = 0.
(,IL

3Il2.

340 . .z!l 3/,.1. x~ x5

31l3. (x

344.3 ;H5.

(x+ :~ )-7 (1++)=0.


(I-I-x)
(1-:1:)

+ .sx + 213; - 24 = 0. 4x~ + 4.t + 4x - 4 = O. + 4x4 ~ Gx 24x2 - 27x - 108 = O. + 1) (x + 2) + (x + 2) (x + 1) = 2.


3 4;.;2 3lXl
2 Z

+ -+

{3+xj (2+x)
(3-.r)(2-x)

= -35
•.

variable au xil iar: 349. x8 - 15x~ -- 16 ~ O. 35U. (x2 - ;iJ (x 2) (.1 351. (xa ~ 2x -- 5)' - 2 (x~ - 2,,; -

28 x+3 -15' 347. 2x~ - xJ + 5x2 - :r 3 = O. 348. 2.t~ - 4.l;l 13x'l. - 6x i5 = O. Itesuel van las siguientes ccuacioncs segun el metodo tie introdncci6n de 3;..!-1

3'-6. x-2 1'-1

f- :.+2

=~+

x-I,

.:c+r.

U1Hl

+ tv --

3)

3!J2. 35-\.

1'1

-x-+
;rZ

+1

3) -

1. 4

x ;cz+1 ==2,lJ.
x"'-x-2

353.

:3 1 +3O+rz=3-x-x~.

0;

J:2_X

+",C

,-x

x'l-:r+2=

1.

§ 9_ Ecuacioues 355 . . 1

de uria vartahla

bajo el signa do mo,lull)

(it

xZ-3x+3

2 () +--,;---,;:---:--;x:ol-3x+'. .J;2-3.l+5· xl_.:z;2·

356 z~_xa __
358. (z (.:z;

8_=2

357.

J;

(x -

as!.!.

aGu. taz 361. (x 362. (x Rcsucl van las ecuacionos: aG3. -iOTl - 3xz - 2.x t = O. 364. 4xJ - ax - t = O. 365. 38x~ 7x2 - Bx - 1 = O. 366. 4r' 6x" 4x 1 = (l367. 16x" - 28x" 4x 3 = O. 3G8. tOOx:! - 120x" 47.x - 6 = 0_ 3Gr!. 6x~ - 13x2 -I- 9x - 2 = O. 370. 4xJ 6x~ 5", -\- Gn = 0, 37!. 3xJ - 2xa x - 10 = O. 372. 32",3 - 24xl - 12.r = n, 373. 4zJ 2x1 - 8.1' 3 = O.

1) (x _ 2) (x - 3) = 15. 1) x (x 1) (x 2) 2!!. 1) [z 2) (x 3) (x 4) = 3. 7)a (4x 3) (x 1) 1" 5. 4,5)4 (x - 5,5)' = 1. 3)4 (x 5)' = 111.

++

+ += + + + + += + + ++

+-

371,. 2

(xa+ ;.") -7

(x-I-+

-j-

!)=-=II,

375. 4x2-1-i2x-l-E-I-_!_=47. :Ii x~

x'-4x-Ll x~-Gx-!)

§ 9. Ecuacloues que contlcucn una variable ha]o el siguo de modulo Al resolver ecuacionos quo cOIILioJ)CH una variuule LJ'ljO 01 signo de modulo se ut il izan con In mayor Irecuencia los siguiontns metedos: 1) supresidn del modulo por definicion; 2) elevacion de ambos miembros de la ecuacion al cuadrado; 3) metoda de par ticion en interval os. EJEl\1PLO L Resolvamos Ia ecuacion 12x SOLUCIIlN

3 j = 5. Como por dalinicicn

(1)

I-er

procedimiento,

(x)

={~

f (x) si f (x»O,
j (x)

si

f (x)

< D.

(j2

Primern

parte.

Algebra, al siguieute

Capitulo

II de des sistemas

mixtos:

in ecuacion

(1) es equiv alente

conjunto

2X-3~O . { 2.r-3= 5 '


do,

I-

2x-3 <0
(2x- 3)

= 5..

2-do procedunienio. Como ambos miembros de la ecuacion (1} no son Ilcgativos, ella es equivalente a In siguiente: I 2x - 3 I~= 25. Perot I f (x) 12 = (f (X»)2, Por sllo , In ecuacion ,(i) es equivalente a In ecuacion (2x - 3)2 = 25, de donde obtenemos: Xl = 4,

Del primer sistema X'Z = -1.

de este coniunto hallamos xl = ~" dol segun-

Xa

=-1.

EJEMPLO

2_

Resolvamos

la ecuaci6n

I 2x - 3 I = x

+ 1.

(2)

SOLUCl6N Como In anterior, esta ecuncion puede ssr resuel ta de dos procedunrentos. Al rcsolverla segun 01 primer procedimiento obtendrernos el siguieute conjunto de sistemas mixtos, equivalentes a 10

ecuacion

(2)"

2X-3.>0 { 2x-3=x+1

'

2x-3 <0 { -(2.0;-3)= x+1,


2 con el segundo

de dondo hal lamos x\=4, x2=T.


Al resolver Ia scuacicn

hay que tone!' on cueutn que In expresiuu x 1 on el segundo mieuihro de (2), segun cl sentirlo de la eCU<lCiUII, debe ser positive: x 1 ~ 0, Dob ido a esta condicion In clevacion al cuadrado de ambos miernbros de In ecuacion conducirti a otra ecuaclon equivalents a la inicial. 0 son, In ecuac ion (2) es aquivalentc al sistema mix to:

de acuordo

procediwienLo,

+>

x+ 1;;:'0 { (2x - 3)2 = (x + 1)2,


proporciona
Xi

4\1(" a1 rcsolvcrl
EJEMPLO 3.

o,

IIl)S

= 4,

X2

= ~, (3)

Hasolvuruos

13 ecuaci6n
=

I 2x - 3 I
SOLUC!I'.oN

Ix

+7

al cuadrado» (segundo procedimiento) es aqui e1 mas convenlente. En eiecto , al aplicar dicho proccdimientc. obteneroos una ecuacirin
equivalcnte a la (3):

Es faciI eorciorarse

de que 01 procedimiento

«elsvacidn

§ 9. Ecu(lCioncs
EJEMPLO ~_

do una. variable Ia ecuacion

hajo el signo de ruo.lulo

G:l

Resolvamos

I3 - x I- Ix

+ 2 I = 5.

SOLUCION, En estecaso as preierible el matodo de «paruciou (HI in tervalos» (tercer proccdin:dento). Marquemos en Ia recta numerica el valor dez' COil el que 3 ~ .C = = 0 y el valor de x can el que x 2 = O. Con eIlo, In rec t o numeriea se dividira en los intervalos I~oo; -2I, [-2; 31, 13; =L Hosol vamos In ecuacicn (4) en cada uno de los indicados intcrvalos, os clecir, 131 coniunto de sistemas mixtes, eq ui valente a (4):

-00 <x<-2 3-x+x+2=O;

{-2~x~3 3-x-x-2=5;

{3< x<oo
>3
-5=5.

-3+x-;r-2=5,

a bien
l,' {

J-oo; -2[,

La solucion del primer sistema de este conjunto es 81 rayo del segundo sistema hallamos que x = -2J y 01 tercero, no tiene soluc ion. Unificnndo las soluciones de estos tres sistemas obtenemce 1a soluci6n de In ecuacion (4): 1-00; -2/. EJEMPLO 5. Resolvamos In scuacion [x anterior

I + Ix

I=

x-

3.

(5)

SOLUCION_La ecuacion (S) es muy parecidn a In resllciln:ell~ ol ojempl o , es decir, a prirnern vista puade parecer que su mas oonvsniente reso!uci6n se Il evarla a cabo segun el metoda de «pmtici6n en Intervalos», Pero, de 1£.1 ecuaciou (5) esta claro que z - :~> > 0,( 0 sea, x> 3 y, entonccs, asi mismo, x - 2 > 0 Y J. -- t > O. Asi, piles, la ecuaci6n (5) cs cquivalente at sistema ruixto
X-2+X-l=X-3 { x> 3

q ue

es equi vnlontc

_ {:r=11

x>.\

'11lP

W)

tlene sol ucionss. De modo que Ia ecuacion no tiene rakes.


EJERCICIOS R esucl van ins ecuaci ones: B81i. I :r I ;r.3 = O. 385. (x - I) () 3BG 4:.:-8 387 71:+4_= , .. I :.:-21 a: _. 5 :!:

I - 1) = 13:r-~)1
;,:

0,5, '

". 388. 7 3!IU. I x2

3\\2. I x2
fx

+x

4:r = I lix - 7 I. 38!1. I 3:r - 5 I = 5 - 31:. 3x 3 I = 2. 31B. I 2.r - .r! 3 I= 2,

-1

! = 2x - L 3\13. I x~ - ,x - :3 I

= -:r

I
II

301i. 2 l.r~

5~6.

+ lP

+ 22
-

21x

5 I =x -

+1I+

1. ,13')5. ;rz + 3 I :r I -I- 2 = 0 1 = O. ' an. .t; + 2.1'7- 3 I )'

1.. ;1 " .. l'.

Priruora

parte.

Algebra.

Capitulo

II

3!)1). I :c I Ir 1 , = 1. 39!J. I.r. -I- f I + I x -I- 2 I = 2. ~UU. I s: - 1 I - I x - 2 I = 1. '101. I x - 2 I -I- I I~ - x I = 3. 4U2. I s: -1 I r x - 2 I = 1. qOJ. 1 x - 2 , x -3 I -I- I 2z -8 I 40'1. I 2x -I- 1 I _- I J - s: I = I x - <'0 [. 405, I J; - I I I 1 - 2.r I = 2 I'" [, 406. I x \ - 2 1 x -I- 11 3\ x -I- 2 I ~07. I:r; -I- t I - I a 1 3 J ,1: - 1 \ - Z Ix - 2 I = I :r; -I- 2 I. 408. ! s: \ - 2 Ix -I- 1 I 3 1 :t: 2 I = O. foU!1. I s: I 2 I s: -I- 1 I - 3 Ix - 3 J = O. ~IO. I x~ - 9 1 I x - 2 I = 5,' 41 I. 1 z2 - 1 I x 1 = O. 112. I z? - 4 I - I\).1'2 J = 5. ld3. I ",2 9 I -I- I x2 - 4 I = 5. 411. I x - xli. - 1 1 = I 2x - 3 - x~ I 415. I %2 2r I - I 2 - x I = I x~ - x [,

+,

=
=

D.
O.

++
1.

4113. 113-2xl-ll

=21 z }. lil7,

1~~-4x1+3 s: +1 :.:-51 mclonales

§ 10. Sistemas

de ecuaciones

1. Nociones Iundameutnlcs. Var ias cell ado lies bles z, y forman un sistema si se pluutca 01 problema

del sis tema. esol vel' un sistema de ecuaciones significa hallar todas sus scluciones, En particular, 01 conjunto de soluciones de un sistema puads ser vacio y, entonces, S9 dice que el sistema no tiene soluclones o que es un sistema incompatible. Varies sistemas de ecuaciones con dos variables x, y forman un conjunto de sistemas sl se plantae el problema de buscar todos los pares (x, y), cada uno de los cuales satisfuga, por 10 menos, uno de los sistemas dodos. Cada uno de tales pares so denornina soluci6n del coniunto de sistemas. EI proceso de resolucion de los sistemas de ecuacionas consiste , POI' regia, en ol paso succsivo, mediante ciertas transformaciones, del sistema dado a otro mas «ccmodo»: a contlnuacidn, a otro aiin mas «comedo», etc. Si como resultado de ciertas transformacione.s del sistema

de todos aqucllos paros (x; y) que satisfacon nes dadas. Cad a uno de d ichos paras llova 01 nombre

dos variade Ia Inisqueda cada una de las ecuacioCOil

de solucuin.

II (x, y)

= gl (x, y)

1
pasamos at sistema

': (~'. !J? ~ .g2. {~'.!f?


In (x,
y) = gn

(1)

': I

(x, y)

(x, y) = g~ (x, y) f~(x, y)=gl(x, y)

..
y)

f~ (x.

= g;, (x,

..

...
y)

(2)

§ 1.0. Sistemas

de ccuacioncs

racionales

f)J

y, si con ello , cada soluciou del sistema (1) es, sirnul tancnmento, Ia sol ucion del sistema (2), esLe reeibe el nornhre de corolario del sistema (1). El eorolario de lin sistema de ecuadones puude ser tambuin una sola acuacion. P.ej., In ecuacion 3x - 2y = 3 es 81 corol ario del sistema

2X+Y= 5 { x-3y=-2
rio do un sistema de -ecuaclones
(como Ia suma de las ecuaciones del sistema). En general, 81 corol ade ecuaciones puedo ssr otro sistema con un numero

tan

to manor como mayor. Asi, el sistema

2.r--t~ y = 5 x - Jy = - 2 { 3x- 2j/ =3

as el corolario del sistema A su vez, el sistema

2X+Y=5 { x- 3 y = - 2 . 2X+V=5 { x-3y= - 2


x-3y = - 2 { 3x-2y=3.

as e1 corolario del sistema

2X+U=

Dos sistemas de ecuaciones rsciben 91 nombre de equiualentes si los conjuntos de sus sol uciones colnciden. Esla claro , que dos sistemas son equivalentcs si, y s6lo si, e1 segundo cs el corolario del primero y el primsro, del segundo. Ell particular, de aqui se desprende que sl en un sistema de ecuaeiones se ad icionu una scuacion mas, corolnr io del sistema dado, cl nuevo sistema es eq IU v alanto al iuicinl. Si omi timos ciorta ecuacion del sistema, i;) rest ante ecuacion (0 bien sistema de ecuacioncs) sed coroIario del sistema inicial. Si en el pianteamienLo no S8 ha prelijado 501>1'0 quP. conjuuto ha de resolverse el sistema de ecuaciones racionnles, se supone que es necesario resolverlo sobre un conjunto de numeros eomplejos. Aducimos dos teoremas que 58 utilizan durante la resolucion de sistemas

t;

11 (x, y) = gl (x, y) es equualente (es el coral aria) a La ecuaci6n I~(z, y) = g~ (x. Y) mieniras que la ecuaci/ui fa (x, y) = g2 (x, y) es equicalenie (es el caralano) de la eciuicuin (x, y) = (x, V), los sistemas
TEORE1.rA
I

de ecuacicnes, I. si La ecuacion

g;

/~ (;:, Y) = ': (It Y) { f (x, y) =g.(x, y) 2


son equiualentes
5-020~

(el segundo

sistema

es el corolario

del pnllll'ro)

Pr imer a
'!'EOHfo:IIIA 2

par to.

Algebra.

Capitulo

II

ecuaciones

f1

Sf la ecu.acion. I (r , y) = g (;r, y) es el corolario (:t:, I)) = g 1 (z , y) Y f.J. (r, !J) = {J z (x, y), el. sislema

de las

/12(x,
l/(x,
1..'$

y)=g2(X, Y)=C(x, y)

y)

et corolorto

del sistema

II (x, {12(x,
II
en tanto que et sisienia

y) = e. (x, y)=g~(x,

y) V), es cquioalentc

(3)

i, (x,
f

(x, y) = s, (X, Y) y) = g2 (;z;, y) (x, y) = g (x, Y)

al sis-

lema (3).

En part ic ulnr , coroinr ios dol sistcmn

(3) serfiu tales sistemas:

Si
j
2

lIO

hay tales pares


_
f)

(x,

y),

COIl

los

qne

ambos

ecuaciones

(X; Y) Y g2 [z , y) se red ucou


y)

s imul tancamcn te n coro , In ecuaci6n

_-=-1 _
/1(X,

En tonces [\1 SIS tema (3) serti eq ui va lcnte el sigu ion to sistema:

g'1.P:' !J

es eq u ivalen to a In ecuacion

j'2 (x. y) = g2 (x, y).

fd:r;, { f (.r, 2
A
SU VC'Z,

y) = gdx, 1
y) -

y) 1

g2(X,!J}

SI!

corolarlo es cl slstoma:

Asi , Pues,

j J egamos

a Ia sig uicu to co lid usiou: si


ex presioucs

flares (x, y), con los que ambas

110 existeti icles 12 (r , y) y g2 (x, y) se retluceti

§ 10, Sistemas

de ecw:wione3 r acionales

G7

simuluineamente

acero,

el sistema

I! {

(x, y)
'2 (I, y)

= gl (.T,
gl(XIY) g2 [z , y)

y)

'1(.r,y)

(7)

es el corolario del sistema (3), Sial resol vcr el sis (em II

que es corola rio del inic in I, Ias sol uc iones h a l lnr] as tid 1111 CVI) si stnma obi iga Loriarnente deben ser veri f icad as (p .ej. po niendo ell ('I sisl em a inicia l Ios va lores obtenidos de Ins variables). A co n t rnu ac run, ~Cn'1I1 utrles las slguientes alirrnacio nes:
1, EI sistema (4) es eq u iva lcntc n1 (3). 2, Si 110 existon tales pares (z , .11), COil los de la ecuacion II (z, y) = g! (z , y) so anulan sistema (5) es equivnlente al (:~).
(1'1.1(,

10 hem as t.rausf orruarl

(l

e Ij

1111

sist euin ,

muhos

Illi('lIliJJ'os

simultiillCilJllClllc,

('I

El sistema (0) es equivalents <:II(3) sobre uu campo de numeros renles si PaJ:1l t oda x, y del cam po do defiJi ici()Jl del SJ ~r.l'llnl Ci) se
cumpte sa red ucon
(1 cero am bas m iern UI'OS (3), el sistema (7) as cquivalsnte l ndiquemos un resultado mas 1 y 2. TEOllEMA 3 el conjuruo de

a.

la dcsigualdad 12 (x, y) -s, (X', y) ;;:;.:O. <1, Si no sxtsten talss pares (x, y). con los qua snnuluincumonte de In segundo "I (3).

ecuscion

d oj slst ern n
(COfeI\\IlS

que se dsspreudo

do lus

s,

ecuaciones

- t«

(z, Y) = R" u (r, y)

f 22 (:c, y) "'" g 22 (x, y)

_ !2h. (x,
el conjunta de sistemas
es equioalente

!J) = Ifz/! (z, y)

a la ecuacuin 11 (x, y)

gl

(x, y)

bien es

51<

corolnrio ;

Ifdx, 122

y)

= gs(x,
(x,

y)
y)

tIll! (x, y) = g21 /,(X, y)=g.(x, {


(x, y)
y)
=

y) y)
y') . (z , .11)

1422 (X,
.,

{f: .:
es equicalente al sistema

= ~1'

f~l' (z, .11) "'"'

f.[~ll

(3) (0 bien. es

su. coroiarios

63

Primcra

partu.

Algebra.

C;JI'i ttl 10 II

Ell purl icu l ar, el cornlarto

del

si<;L0111<1

II (x, { u.!».

y) =li, (x, y)

Y)·/?2(X,

y)· ...

·f2l,(x,
1/);

y)=O

cs el cnujuu to de sistemas: { fl(x,y)=gj(X,

f 122 (x,
EJ EMPLO I

it

(x, y)

=«, (I,
y) =

121 (X,
y) ;

y) = 0 { II (;1:, y) =B, (x, !1)

0
el s istcma . XU-6
{

1211 (x, y) =

o.

Il oso 1\"amos

=-

1;3 s:

xy+24=
reules.
outro u" G)

y
del sistema

;t"

(8)

sobre

un

C.UIIJlIlIlo

do

numcros

SOLUGI()N

obtenemos

Mult ipticando cl sistema:

81 las ecuuciones

(8),

.xy-6=--a
{ (xy

+ 21t) (xy

r "z" = ~} ~ xy

(0)
,

ces, tomando

que es el coiolario del itdcial. Cun ayudu UC seuc il las Lr-a nslorrnn ci ouos la segundn eCl\iICLUII del sistema (<J) so reduce IT la ecunctou = 8, OS decir, (II Gorolado tic 111:scgllnda ecuncion dol sistema (9). Ento n-

xu

ell COl\5llieraciuIl
{

el tcoremn 1, ol sistema

XY

-6

=~

xy=8

(10)

sera el corolarlo'ilel sistornn (Q). Sustraigamos ahora la primera ecuacion del sistema (IO) de In seguud a. Oblencmos el sistema:
Xy=s {

6=8-XI./=8 {

y~

x'

y, a continuac

ron ,

~~ =2.
2, el sistema

(11)
(11) es el corolar io del

A co nsecueucia (to).

dol Lcorcmn

§ '10. Sistemas

de ecuaclonos

rncionales

(jg

Despucs obtcncmos

rle multipl icar cntr e SI I,1SeC\IOci()I1CS el sistema p:y=8 lyl = ·16,

(k! -rstcma

(11), (12)

que cs el corolario del :;isLOJlI a ("I1 ). De 1;.1 scgund n ec IFllCJ(JU del sistema (12) 11311aOlOS Yl = 2, Y2 = -2 (nos l irn itames a 1a1O Ices real cs) r,\ y de la primera ecuacion Xl = 4, X2 = -4, respectivarneute.
Asi, pues, el sistema (12) t.iene Ias siguientcs soluciones:
y

(11; 2)

VERIFICACION. Como el sistema (12) es, en Iin de cuent.as, e! coroI ario del (S), las sol uciones hulla rl as del sistema han de ver: Iicarse, 10 q ue pued e rea Iizurse ponicmlo Ins sol uciuncs oh tenid as rlel sistem a. (12) en 01 (8). Esta verificncien muestra que amhas solucioues dol sistema (12) 10 son tamhien del (8). Asl, pues, las soluciones del sistema (8): (4; 2), (-4; -2).

(-4;-2).

EJE;\!PLO 2.

Resolvamos el sistema
Sumando 2: las

XY+XZ= -4 yz yx = -1 { zx+zy= -9.

xy Xl, yz = -1. Unisndo esta ecuacion II los ccuactones del sistema prefijado, ohtcnemos till s istema eq U lvalen te "I dado segun
el teo rema . z.. y

SOLOtION.

l.res ecuacioncs

del sistema

oL tenernos

1
sistema:

xy+rz= yz+yx= zx+zy=


z.

+ x. +.

yz = - 7
~4

-1 -9.

Sust ituyumus lu segundo ccuEicion de este sistema por In d ilcrencia de las dos primeras, Ia t.ercera ecuacicn, poria diIf'.('encia de In primera y la tercera, y Ia cuarta, por Ia diferencia de Ia pr imeru s la cuarta y, a dernas, omi times Ia primera ecuaci6n. Ob tcnemos el

YZ.=- ... 3 xz= -6


X!J=

que, ell v irtud del teorema 2 y 1a alirmaei on I, es 0(111 valen [6 III 1 dado. 1\j ult iplicando entre si las tres ecucciones, hallamos: (xyzr = = Be. Aiiadiendo esto. a las ecuacioues del anterior sisternu. llcgnolos

70 al sistema eqnlva

Prunera lente.

parte.

Algebra_

Cn pi tulo

rr

(X!/Z)2 -= 3Q yZ= -3 xZ= -G \

xy=2
Vi!Z,

(aqui se utili z« de nuevo el tcoremu 2). a l que, a SII del teorsma ;),8" I)qlllvUJCIlLo 01 ccnjunto de sist.em as:
xyz = 6 yz = ~ 3 .
XYZ=

en virt iul

1 X!/

TZ=

-6 '

=2

yz= xy=2_

-G -3

xz= -0

Hesoiv u nios ol pruner ~istum a do os l.c C{)!i j II lito, U idel ie ndo sucosi v ll~ men Ie 1a pri III pra IJCII ilC n del slstem a entre J a seguuda , tercora , c narl.n , o]JLenclllos: x = -2, Y = -'1, z = a. Do modo nrnilogo, del segundo sistema, 1I::l1laIl105: x = 2, y = "1, z= Do forma (1I1e ol con] unto de sistornns, ig IW I que 01 sistema ill i c lal

iu

-a,

equivalente a el, tlene las (2: 1: -:;l). 2. l\ICLoliO$ hlllda mentales ell tres Illc;ludDL-\ fllJ1dMllCIlLalos
1) translor-mncuin

sigutcntes
parn

solucioues:
sistemas.

(-2;

-'1; 3),

resul
(0

YCl'

pnrn resolve!'

sistemas

tit uc

It)]1;

:3) cam hio 110 vu rill hi os. EI


se

l i nonl

ilcl sist crun

ilion suma
lHr.'ilodo

de In tl'aJlsIOrlll<lcioll

Dctcng.irnouoa de ecuacioncs: nlgubraicn): 2) S11S'

lineal

del sistcm a

basn ell ol stguien

L(' teornma.

,,(X, y)=o { I~ y) =u (x,


son equicalentrs,
(11 sistornu

Udl(X,

y)

+a2f2

(x, y)=o
y)=O

bddx, y)+u2f2(X,

Eu port.icul a r,

:::11

al = I, a~ = 0.1)1 _.. I ,

±1, obtoncrnos

/dX' y)=o { /1 (.1:, l/) ± 12


que de acuer.Io

(.1:, y) = 0,

\;1)11 1<1 nf irrunc ion 1, es eqlliva!cnle u l inicial. Es le tcoremu 5(' d 1v ulga [11 caso cua ndo 01 lilt tIl era de ecu aciones es mayor qlle rlos, P.ej., para trus ecuacioues con tres incognitas
t ieno

lugar

01 siguionto

t coremn.

§ 10. Sisl(Hll;lS de ocuacinnes

raciona les

71

TEOnEl'I1A

,,'.

St 6.

al nz aJ VI li2 /;3
C! C2 CJ

0, los sistemas

{ f2(x,
f3(x,
son equiuolentes,
EI
III

I!

(x, y, z)

0 y

y, z)=O y, z)= 0

e Looo
5. X= {

'fEOHF.MA

de sustltu cl 1m so basa en e 1 Los sistemas de ecuaciones

sigll

ion Lp

tCOl"l!lllH.

F(y)
y)

=F (y)
(y), y)
=

I (x,

= g (x, y)

I (F

g (F (y),

to

son equioaleutes.

P.oj.,

sorrin equlvalontcs

los siguieutes x= 2y-5 {(2!1- fJ)2

sistemas:

x=2y-5 {x2 -I- y2 = 2x + y

+ y2=

2 (2y -!J)

+!/.

conor.xruo. Si la ecuacuin (F (.1:, y) = 0 es equiualeute x = F (y) (0 a La ecuacion y = F (;t)), el sistema (P (x, y) = 0 { / (z, Y) = If (x, y)

u /11 ecuncion

es equiualenie

a l Ilsfenla

{X=

E (y)
y)=g(F(y),

f(F{y),

y)

y =F (x) o bien al sistema { f(x, F(;r.»=g(r,


P .ej.,

F(x)).

III sist omn do ecuacio nils yz.+x= 2 (x-G) { ..lL-I- ~= x2-1- yZ


x !J

es cquivalcnto

al siguicuto

sistema:

X=y1;+ {
Para
pond icntc
!1"-\-1"
.iI

+ y~+111 !l
del

10

=(12+ !I ruudo

10)1.+1/,'"
-

un

Si5lcIDI.\
t eorerna

se

de tres ccuaclones
enu nc ia

con trcs vari;lbl(>:~ d corresslgu ient o.

72
1'F,OIlEiIl-\

Pr rrnorn
S'

parte.

Algebra.

Capitulo

II

Et sistema fl(:r.;,

de ecuacioncs

y, z)=cdx, !I, a) { 12 (x, y, a) = g-z (x, y, .z)


z=F(x,
y)

es equiualenie at stguieute

sistema:

II {
St El uietodo

t, {x, u. F
z=F(x,

(XI V,

F (~, V»~ = e, (x,


(.t, V).

V»~ =K« (x,

Y, ~ (x, y. F (x,
consiste
{('z{;l:,
(1'2

V»~ y»
en 10 siguiente.

de cambia

de varia ble

I FI (x, ( I r, (I,

y)
V)

= fd(rdX,
= 12 ('pdx,

V),

V)~ y

v),

(x,

v»~,

el sis/emil

FJ (x, Y) = 0 { F2 (x, y) = O.
mediante

escrtto en la forma

.'

las

IWellClS

uariables 'PI (.2:, { 11 (u, !J) = 0 12 (u, tI) = 0


, (u,,;

yJ

U,

<P2

(x, y) =

!)

puede

ser

Se<111 (ul; VI)' (llz; v2), •.. sistema, Entonces, el problema conjuuto de SlslCl11<1s:

se reduce

vn)

<1

las sol ucioues del ultimo Ia sol ucion del siguiente


{PI {

<PI
(1'2

[z, y) = uj (x, Y) = VI

[PI {

(x, y) = 112; rp2 [z, y) = V2

•••

(z, y) = Un • Y)
= Un

11'2 (X,

Las sol ucionos del sistema

de cste conj unto sertin,

si m uluineamente,

las

F, (x, y) { F'J, (x, y) Consideremos algunos siemplos resolver sistemas de ecuaciones.


EJE~lPLO J.

=0
=

O. al

tic cmpleo de estes mstodos


{ x'J.= 13x+
2.

y = x+ y. Sustraigauios In soguuda ecuaciou de 13 prunera. Entonces, segun el tcorama It, el sis lema
SOLUC10N.

Resolvamos

el sistema

13

4y

xZ-y2=(13x+4y)-(4x+13y) { y2=4x+13y

§ 10. Sistemas do scuaclones racionalcs


('5

73

equivalente nl inlcinl. Annl iccmos J<1 prunern ecuacrou 1.lel ~bLl'mfl obtcllido. Tenemos (x - y) (x y) = U (x -- y) y, i1 ro nt iuuaciun ,
(x y) (x y - 9) = O. Como result ado, llegnmos

teoremu

1, es equ ivalcnte

al

ill siguiente
iriic

sisll:'lJla

fpH_"

ial :

seglJJl el

(::r- y) (.1:+ y{ y'2.= 4;z;+ t3y.

9) = 0
es cquivalent e (II siguien-

De acuerdo con el teoremn


to conjunto de sistemas:

3 este sistema

X-y=o { y2= 4x+ 13y'

x+ y-IJ=O { y2= 4x+ 13y.


COil

Rssolvnmos cada uno de estes sistemas cion. EI primer sistema se translormn de rlondo hullnmos: EI segundo sistema
X, {

el melod!) de susl.itux=-={

a In Iormn
Y2 -

=0

del conjunto

Yt -

_ 0;

{'<:2

" , !r = 'llj

!J

I"

,"lv,

= 17 -17
.

se transfnrmn

a lu Iormn

.En total, homos hallado cuatro soluciones: (0, 0), (17; 17), (-3; 12), (12; -3). VI';RlFlc.ACION. Como durante Ia resolucinn del sistema prefijarlo se han real izndo solo t ranslormaciones eq llivnl entes, 1as soluciones
ohtenidas
EJEII1PLO
SOO,

= =

De l a ecuucron y"-:1 y, a continuacion. ·-:3, x. = 12.

X=9-Y { u'> 4 (9- y) + 13y. = 4 (8- y) + 13y hn llamos:

!h co", 12, 114 = dt; In l'ol::lciuu x = \)- Y oLtCIH!IIIIJS .1'3 =

aisimismo

, las

4.

Hesoivarnos

81 sistema de ecuaciones
2

del sistema

inicial.

x+ {
SOLUCION_ X2+

y+z=

2x+3y+z=

Adoplemos

(y+ 2P+ (z- J?=9. el meto do de StlSlitlicion,lC'Il!'IlIf):-;; 1 (z-1P'=

x=2-y-z { y, seguidamenta,
2 (Z-y-z)-I-3y+z= (2- y- z)2 (y 2)2

x=2-y-z y- z = - 3
y2+

++

!)

z2.+

yz- az= 0,

74

Primcra

p(\rte. Algebra.

Capitulo

II

A
1\!\

SU vez, I as dos till i mas (len ncio nos dol sistema sist em a d (1 rlos or,lind ones COil clos v aria hies.

uL tcuido for III a n Resol varuos este

sist em a scgun cl I!l(':;[udo de sust it uc io n. Tenemos:

{ ~Z~Z3)2: zz+ (z-.3) z- 3z= 0,


cs dccir,

Y=
Z2_

z-;~
4.z+ 3 ,~-o.

De la ult una ecuacion luil ln mus: Z, = 1, Z2 = ;;. Do Ie ocunciol\ ZJ o htcnemos: Yl = -2, Yz = 0, respecti vamenl.e, y de In ecuncion x = 2 - 11 - z , hul lamos XI = X2 = -1Asi , P II es, heruos 0 blcH ido las s igu iell Les sol ucio nos: (3; -2; 1),
y

~~t

(-1; 0; :1)_

2 E.I EIIII'LO" BlSO Iva mos 01 sis tetlin de ccuacioncs yz =2 { zx+y2=2. SOLUCION Sustituya mos lu pruncrn eCIIIH_;ion del problema por J a d ilorouc in .lo J a pri morn y sogurula ecuacto IIOS, 1a segu ndn , por In difereucin Je In »l·glluda y terceru Y no tocamos la tercera, Obtouemus

xy+z2.=

x2

ol sisl em a'
:r:y { uZ
1:Z

yz-

+ x2_y2

Z2 -

:1:2

=0
=.

zx+ y2~~ 2, (z-x)


(x-y)

('5

dccir,

d sistema

(z+.t:)-y (x-i-Y)-z

(z-x) =0 (x-y)=O

xz+y~=2
lj_,

que, e 1\
tenumos:

\ ..

i r lilll tid

Icoroma

es cqu ivide n to al inicial.

Despues,

(z-x)(z+x-y)=0 (x-g) (:r+ y- z) = 0 { xz+ y2= 2. De <IelIl'l'd u en II


siguicnto coujunto
()

I tcorcuta

:1, a oste sistuma

es eq n iV,t1CII ted

tic sistemas:

z-x=o
x-y=O ,

{'-X~O

xz+

11'1. -:- 2-

x+J/-z=O; XZ+y2= 2

{'+X-Y~O
x-y=O

xz+ y2= 2

f z+x-y=o x+y-z=O l xz-!-y2=2,

§ f O. Sistetnas

de ccuaciones

racioun les

R esol va mos 10$ sistemas de esto co nj II lito eon (,I IHr~t(l d I) d [' sust itucion. Del primer sistema hallarnos: (1; 1; 1), (-1; --1; -l};
del seg uud 0:

(V2; 0; V'2). (-V2; 0; -

V-zL

del tcrcero: del cuarto:


VERIFrCACIQN.

(VZ;
(0;

~(2;

0). (-liZ,
(0; -

'V2; y':2),

clones

En el proceso do In resol uci fueron equivalentes, por 10 que Ins ocho solucioncs hnlladns SOil, asim ismo , Ins dol sistema dado de ecuuc iouos. 3. Sistemas homogeneos. Un sistema de (1(1:; ccuacro nes e.011 des varIa hlos del tipo aox,r-l- arx"-I {
y

- ~r2,D}; V2: -V2), on lOllll,~ !J:-, tLH1SfofJl!;1-

~I-/.2X"-Zy2 (

+.
"

OoX>l+ Ulx"-Iy-l- U7X"-ZyZ+


In bru

,+ "n_
pri

,~l-0 n_l:cy"-t -1- 11nl!,l'


1 X!J1l-1 III

:""

-I- {),,!I"

::'';' (l
ilIIlIJil.';

roc ibe el no

eCIHlCiOl1e~ son

Los sistemas de este lipo se resuel veil mediante 1;1 COllI hlll<lcir>1I de dos melodos; trunslormacion l incal e mtrod ucciun do IIlleVH.'1 1';u·i:Jblos. mell1PLO G. Hallsmos las ruices reules del sistorua:

pol j no 10ios hornoge !lOOS de grade n. de do"; val' in blos).

de /tomogclI(Jo

(los

)lH)l'OS

ion!!,\! (),~ ..It'

3x.2+ X!/- 2y2 = a { 2x2 - 'jxy + ya = - 1,


SOI.UClUN. La prunora oClla(.iuiI os h011lug611CiI (n,(lldl'llh'~ !Jill) aSl so clencminan las ecuacioues dd tipo f (1', V) = 0, IfOJjf.l~', f (£, y) as U II pol ino m to homogenoo}. No t ornos q uo ,II hacer !I = 0, de 111 ecuacicu 3:c1l "-1- xY - 2y:: = 0 hal lamos :r. = o. P01'O cl par (0, 0) lIO SIl tisfuce In scgunda scu acton tiel sistorua por 10 q ue Y "1= U y, pN 10 tanto, ambos nrlembros de 1[1 ecuaciou hornogeuon ,k<" :xy ~ 2y'l = 0 so puedcn dividir por V'! (10 que no <ledI'n'H 1.1 penl ida de calces). 3X2 xy 2y2 f) " • ;) ( x ) a L O btenemos: -::z+-z --2-=-~ Y. a CQJllHIUClCLOII. ,-, T

+..:.. y
x= -y
do

2 = 0, de doude hal larnos que _:._ -1 =


0

!J'

if

hiou x=-yY'
IJIl

y,

Lien _::.."".~. .
-

sea,

AIlOr", el pro hl emn se reduce a resolver


eCU<lCiOHl'1S:

co njunl.o de

~iS!l'IIl;lS

x=-y { 2xz- 3xy

+ y2 =

X= 2_

-1'

l 2x

Y
3xy+
y2

= -I

7G

Prirnera

pnrtc, Algebra.

Capitulo

I1

EI prrmero lit) estes sistemas no CoS compatible y el segundo llano <los rafces: (2; 3), (-2; -3). Est as serrin las rakes del sistema dado. 3X2_ 8:ry+ 4y2 = 0 EJEMPI,O 7, Ilosolvnrnos e1 sistema 0 { 5x~-7xy-6y2 SOLUCION Ante todu hemus de !Oeiialar que en nuestro caso el pal' (0; 0) sot islace el sistema. Sea, lnllS adelnnte, Y =1= O. Drvidiendo

=.

por !J2 amlros

mismbros

de cada

uun

de ius ecuacicnes

de segundo grado , que form:m el sistema

preiijado , obtcnemos

nomogeneas

{ 5(~)
de don de hallumos
yy

3(;)'-8(;)+4~O
2_

7 ( : ) _ 6 = 0,
2

..:._ 2'
,

y=3

-=-=2.:r

'y

3 5

Es dcclr , ~
y

= 2.

Haciendu y = t, entunces .1: = 2l. Seiin lernos Ilue con t = 0 y x = 0, tam lnun y = O. Asi, pucs, los rnices del sistema dado es un par de la forma (2l; l), do nde t E R. EJElIIPLO 8 H eso J v amos cl sistema de ocuaciones
3X2_ {

2xy = 160

2_

3xy- 2y2= 0,

(13)

SOLUCION 1\1 lipliq uemos ambos m icmhros de In segundo II) por 2.0 Y sustraigainos Ia ecuaciun otn.enida de In primera del sistemn:

ecuacion
ecuncion

3x2 Zxy = 161) -2!1Xl_6Ilxy_ 4\Jy~= 100 -17x2+58xy+4I1y2=U •

Homos olJ[eniuo el siguienta

sistema

equivalente

al (13):

3X2- 2.xy = 100 { 17xZ- 58xy- 40y2 = O.


Exnnnnernos In ecuacion
17x2
-

(14)

liomogenca
58x!! I~O!l2

O.

(15)

Si y = 0, de esta ecuac ion hnllnrnos x = O. I'oro el par (0; 0) Do sauslace el sistema inicial , Es decir, y =F 0 y, por esta rezcn, di-

§ 1:0. Sistemas

de ecunclones

racionalcs

77 IIlIa

vidicndo ecuncion

ambos miembros de III ecuncton (15) por y2 ohu-nomos equ.valonte a ella:


t7 ( ; )
2~

58 ( ~ ) ~ 40

= O.
c uadr i tl C,I

Hagamos

rt

= -:r:
Y

l lcg aremo s a Ia ('ClIucion

cuyas

, 1'(l.1COS
'1

u1 = 4 ,

U2

111 = - 17.

E sto <
(1ti)

.. srgru

r'rca
:t

que

Ia

., CCIWCIOIl
- = --1'7
X.

15 )
v,

es

equi va ente

a.1

conjunto

d C ecuncrones: .

- ='1;
y

correspondien

temen to, 01 sistema

de sistemas:

os equi valen tp it! cou jun to

11'1

"

1
Aplica ndo hallamos
II

":'=<1
;;r:2-2XY=160;

(.!_=-~
'I

17

~ 3x2-2xy=

JGO ..
., IISU

cada u no de ostos sistemas (11metod» tI (I ralces: (8; 2); (-8; -2),

llici0

las sig uien tcs

". t7) ( -v'2


EJBl'r[PLO

("i' "'-2

17)

II,

'

Como al resol Vel" eJ sis! em a da do 5010 se rea Iizuro n tn nslorrn <\crones equivnl e ntes, las raicus lWUOlll.1S :';0[1 lamui(:n Ins clcl srstomn inicial ,

n. Hallemos

las raicus rcalos del sistema

{
SOLucrON.

X3+ y3 =1 xl Y

+ 2xy2 + ya = 2.
algcbraica de las
ecuUCIOIlt'S

(16)
del

Realicernos

la surna

sistema

(1G):
;&3+y3=1 - x'y+Zxy2+y3=2 2z3_:r:2y~2:r:y2+

.(-1) y3= O.

/.2

ob tune mos
cquivatente

el sistema:

{ Analicemos
al inicial.

2X3 - J:-2y - 2Iil. X3+y3=1, 2xJ

+ !/3 = a
2xy2

( 17)

la ecuacion

+xy
2

+ !l = O.

78

I'runura

parte.

Algebra

Capltulo

II

Como ('/I ol <llIlerior ejemplo, uqu i scrin posible div idir ambos ombros por y2. I'oro, en oste case l'!< 111[15 [{lcil de d CSCOiOponer el pr imer In iQlIIlJl'O en inc tOI'OS: XZ (2x _ y) _ yZ (2x - y) = 0
OJ j

y, seg til dame ntr , (21: Es dccrr , ('I sistema y) (,r; y) (x

+ y)

O. coujunto:

(17) es oquivalcntc

til siguicutc

2x-y =0 { xl + y3 = t ' Emplenndo cl rndtodo estes sistemas hal lamos

{
de sustttucion las srgulentcs
.

x+

Y= 0

;1;3-1-y3=

1.

( vg .' 2 I:-Il) (~i21. 3 3


1

. Vii)
2

respecto a carla UIIO de raicos del sistema (1G):


de ecuacio ues: (18)

EJIT.Ml'LO

10

II all emos I as raices ruulus rlel sistcmu

x~ { :z;Z_XY-I-y2=

x2y2 + y~ = U1
7.

SOLUCION Este sistema no es humogenoo, peco puedc ser reducido a el. Para clio es sulie iente clevar "I cuadrudo ambos mismbros de ill Sl'!:flllld" ecuncion. I.> Icncmos 01 ststcmn:

x~ xZyZ { (X2 _ xy

+ y~ = 81 + y2)Z = 4.fJ + 2x2yZ_ at

que cs ol cnrul ar iu del iuiciul.


X4 {

Dcspues, 2xJy

tcncmos: 2xyJ = 4!).


POI'

x~

+ :r,2yZ + y4_

xZy2

-I- .II' = !H

Sust it uyumos la scgundu ecuaciun do oste sistema cia de In prrmera y seguuda ccuacioues:
X4 x2yZ-l- y4 = { xJy- X2!1~+ x!i= 21.

In dilersn-

(1 D)

P <Ira resol

VOl'

oslo

sistema

humogiinco

a pl iq 11omos

01

de lil sum a algehraica:


.r4+.r~!I~-I-!I~=ClI _ { .T:O'l-x"'l~+1.1/"'=21

me tod 0
(20}

1.3

·1,~

10. Sis temas de

CCIHtciOIlCS

rae ionales
110

7\1

Si Y = 0, onto rrcesz = O. Pure c I par (0; 0) (l8). Si Y 0, 1<1division de ambos miemhros por y~ cnnd uce a Iu ecuacion

so tisl ,I ce (,I ~.i'1l ('1)1;1 de la 1'!lldCW!I (2IJ)

3(:
Haciendo

r- l : r +
t3
a la (20).
y
:~U4 -

iG { ~.
]a

-13 (;

)-1-

3", V,

quo es cqutvalento

U= _:_,

obtcnemos iQuJ
---1-

ccuncion

(ju2 -

D iv ida mos ambos

micmbros

1::lu ---1- :1 = u. de esta CCllHC iun por u ,III

3u2-13u+
y, a contluuneiou, Hagarnos

1G-~+~ It
~2

=0

(u +
2

)-13(u+--;-)+1Ii=U
nZ 0

o = u -I- _!_, uutonces 2) 130


O2

3 (02

+ 1G =
=3
1n

+~ u

= v2-

2y

ll'lIIll('nlO<;:

b ie n

3u2

au
U

-I lU = u.

de doudo
1 I')

VI

= 1,

.
de ccuacioucs:

n esol vamos
+-;;-=""3'

ahora

01 conjun Lo

+ _1_ = 1;
H

u-l-

La pl'i mera ccuaciou del COllj1T l! to no LiclIo sogu ada haltumos: Ut = 3, ul! = ~ . Asl , pues,
equi valente a I conj
1I

1',1 Cf!S

rcnlcs;

j,1 ('C-lIilciu II y o l'fJ II iva h'J)!c'

de la (20) ('I.' :11 con-

n to de ecuacioues:

__:_ :1; __:_ ~. = =


y

uueutras

q ue 01 sistema (H)) es, correspond j unto dn sistemas:

ien Lemen to,

{:=3
Esto conjunto
VERIFICAGION.

Xl+:c2yZ+y~=

01'

{~==t

Z~+XZy2+y!'

=tlL

(21)

Liene las falces: (:1; 1), (L; 3), (-;-:\, -1), (-1; -J). Al resol ver cl ejern plo , todas Ins t.ransformnclones, sal vo lu primera , conrl ucia 11 a. sistemas, eq uivnlen tos, H cal I zn mlo I H susut tieton do J as raices hail" IIns en cl s i:-;LI)Jlln (HI), lIO:; core i urn mus do q u C J us cuatro .1'nlcGS del coni unto (21) son La III bill u l ns de iiI. I,. Sistemas simCtricos. ncconlcmos los <latos I ((II!!/!. mcut.alcs ncerea de las expresio nes simdtricas. La oxprcsie n F (.r, y) rvcihe cl nornLrc de simetrica si f1.1sust ituir 111 variable x por u, y por x ell a no va ria. P.oj., Ius sigu ior tcs exprosioncs SOli si mclricns:

F (x, y) =xz

3xy+ y2, F(x,

y)

Vx+V+2xy-l-..!...+

_!_,
!I

80 Los Iuudameutales

Primera parte. Algebr«. Capitulo II

couslderan X Y Y l·Y· To dos los demas pollnomtos slmetr icos do dos vnrlnblos pucdou ser exprssados COli los Iundamentales, Hac iondo, pnra (liJrevinr, U =x lj, u = ;x;y, obtenemos, p.ej.:
Xl
X4

pol iuornios

sim{.tdcos

de

dos

var iab les

S8

+ y3
+
y1

(r

= {x~
= =

.re

= y) (x!l _ xy y2)"Z _ 2x~y2 (xJ

+ y~

+ (x + y)2
=

+ y2)
(ul

_
=
_

2:ty = u2 - 2v, U (u2 - 0V) = US


Lou)"! _

3uv,

Xli

+ yS
x'l.

+ xy + !/ = (X2 + 2xy + y2)


en el q LtO lod
115

(Ii (u~ -

+ yO!)

2u) (11:1 -

= u"' - 4u2v 21)2, y3) _ x:!.y2. (x y) = 3uv) - u"Zu= u6 - 5u3v

2v1

+ 5uv
V,

2,

xy = u2 -

etc.
l lev a

el nombre de simelrico. Pucde ser resuelto segun 01 metodo de cumblo


de varlahles, tomando como IItlOVQS vartablcs los pol iuomios trtcos Iundamsntales. EJEMPLO It nasal varnos 01 sistorna de ecuacio nes
;);3 {

Un slstamn,

I as ec uuciones

son si

me tr icns,

sirnri-

XJ 3

y3

= 17

x+xy+y=5.

SOLUCION.

Hagamos
0

{X
U CD

+y = u
xy = u,

tema p refi jad

so red

Como aI S igu ien te: u3-3uv+v3=

:1.,3

+ y3 = u3 -3uv,

el

Si5~

u+v=5. De ests sts tern a, 0 btenemos:

17

Ahora

nos

quecla

resol ver el siguieute

conj un to de sistemas:

{
Las solucloues son las siguiontcs:
oeSSF!V.A.CtON.

x+Y=3;
XY= 2

{X+Y=2
XY=

3.

1+tV2).

de esto conjunto y, con el, del sistema iniclal, (1; 2), (2; 1), (1 +CV2; 1-iVZ), (1- i V2;
sistema

(veaso la p6g 78):

Rctorncmos de nuevo til

estudiado

CD

el cjempio to

:z4 X2.112 +!/4 { ",:1-%.11+.113=7.

= 01

§ 10. Slstcmns

de ecuecionos

raclona lus

!::II

Este sistema es simetrico y, por 10 tanto, como el a nterror jJlII.'de ser transIormado a una forma mas sene il ln Int rod ucicndo nucv as Y<lriahl(,:r { x+y=!t
:t.'1

=v.

y,

a eontinuacien

(ll~-2.{!)t_~2=(1l {

ut-3v=

7.

))0 la asgunda ecuaci6n de estc sistema. hal lamos: I~~ = 3u i C'1n ayuda de esta sustiLuci6n In primera ecunci6n ([,,1 sistema sc transforrua a In Iorma: (3t! 7 ~ 2!J)2 - !J'l = 91, de dondo ohtenernos: IJ = 3_ l = 30 De Ia ecuac itin u 7 ha llamos UI.~ = ± AsL. pucs, ...\ sistetua tiBnc do:'! solucloues:

'I.

Por 10 tanto,

el sistema

inicial

es equivalente '

al con] unto de slstomneXy=.'1.

{,r+y=I,. xy=3
plo 10.
Este conjunto conduce

{:r+y=;-4
soluciones quo obtu
V .mos

a las rnismns

on (,I

{'j','I11-

EJi':HCICIOS

427.

x2+3!/2 -xz=u 2x-y+3z=11 !t2u. { x+2y-2z=1. _T.2+y~-2x+3y-!J=() { 2x2+2y2+x-5y-l =11.

BIZ {

+ III = [3

Iy=2.

x2_xy_y2+X_2y=_2 li2S. { 3z/l-5y~+3x-Gy=

-5.

421) .

x+yt=2 !I 1X = 2

z+zy=2.

430.
0-020~

x~y+I~!I=2 { ~3_+_4.~=7.

x+u

3."-'1

8"

'"

Primcr«

parte. AlgeLra.

Capitulo

]I

43G.
437.

439.
411.

453. xy{x+V)=20
451j.

"55.

7+y=T'

45G.

MiS.

4GO.

§ i1. Problornas

para

ecuacicncs

y sistornns dt, eCII;Il~IQru)~

8:t

§ 11. Problemas para In composlcton y sistemas de eeuaeloncs

de ecuacloncs

La resoluciun de prnhlemns escritos que se resueiven coulccclonando cell aciones, por regl a, se [leva a cabo en cu atro eLn pas: 1) designnudo eon Ins Ietras z, y, z. ... las variahles incugnitus, lie las que se trata en el problema; 2) Con ayuda de las variables introducidas y las variables conocidas del planteamiento del prohlema , COJlIeccionar el sistema de ecuaciones
(0

de una sola ecnac iun):

3)

1'1:50-

PflJ1I('f:\ parte.

,\ IgrbI'il. Cap!l.ulo

II

luci6n dol slbtema cbtnnido de ecuaciones (0 do una ecuncion); 4)elccci on dt! Ias soluctonos de ucucrdo COil cl son lido del proh Ie rna. 1. Proulcm,ls sohre dCllcndcncias 1111 mericns. Para resol ver Lalcs
pro hlsmus so utili 7.;1 II los sigu ien Ies [nc t ores: 1. Si at nuruero (1[1 lura! x se le ad j u nta a 1a derecha 01 de n cilras y, C-OIllO resultud 0 0 ilL onemos el mi mero 1{}"x 2. Si a y b son numoros naturnlcs, COli la particuiaridad a> IJ y que a no es mul t iplo de b, ex istc un par, Y s610 tales nurneros twLufllles q y r que a = IJq r, doude r < b divisor, 0, el dividendo, q, 01 cocieute, r, 01 resto) .

+ y.

1\ lllllero

de que

uno , de
(a es cl

. mMPLO L Hallomos E un numero de rlos cilras si sabernos que la cifr a de sus unidudes es 2 veces mayor que 101cifra do las decellas r q lie cl prod ucto d cI II Umora buscado por Ia suma do sus cif ras es igual n 1!t,)-

Sf:!<1 x 1.1 ciira de Ins deceuas, y, la de las unidcdes del gil lonccs, el propio numero l iene Ia forma lOx -1- !J. Del plnnteanueutu dol problema se deduce: prirnero , que y - x = 2 )" segundo, que (IO.T y) (.r y) = H4. Como result.nto Ill!ga11l0S al sistema de ecuaclones
SOLUClON.
IIum

ern buscado

V-X=2 { ("1Ox+y) (x+v)=


Este sistema

'144.
2 )• 11 del problema. Por 1.

tieuo

dos sol ucioucs:

(2; 4) Y ( - 3 t~ ; -

10 tanto,

obtanl do S8 (lJvid~ por el cuadrado del numoro B, en el cocionro ohtendrsmos au Y (In el rssto , 575. Si al numero A so adjunl(\ a l a dereclia el 1) .... unero lJ y del mnnero de cuatro ciiras obteuido sa resta 01 II urnero de cua t 1"0 cil ras que se forlllll si ol n uillcro JJ se a djunta

EI segundo pa r 110 sa lisfacc el pJ II n lea til [onto cl numero buscudo es ig nnl a 2/i, EJEMPLO 2 lL,Uemos dos numeros de dos cifras A y B sobre los (j \Ie snberuos 10 sigu i eute, S i al IlU utero A so ,HI j un t a a In derech a el IlUIllCrO D r, 11 cout.inuac ion, In citra 0, 01 numero de cinco cirrus

:\1

JH'lIllCrO

A, ohlClldrcmos
A I ntlju
ntu

SOLUCH}N

rOn

1287.

In .1erechn del
IlUIllCrO

numoro

tOB.

AilatllClulo

el plantounucnto delproblernu cl numero de cinco cilras 10DU...-l -to B es 81 di v identin, HZ, cl divisor, 39, el cocien te, 575, ol resto, es decir, iOOOA lOll = 39B2 575. A cont iuuocion, si [I In dereclia del mimore A pouemos el numero B (de dos cilras), obtenemos 100A 8. Si £I In dcrecha del rurmero IJ adjunta mos el numero A (de dos cilras), hallamos 100B .4. Scgun el pl anteamionto (100A B) - (100B A) = 1287. De ucuerdo

obtencmos

tOOOI1

+ 108.

este

Il iunero 11, (J btcuomos tic l res cirrus al uumero

A,

el

COil

-+

-+

§ 11. Problemas

para

ecuaciones

y sistemas

de ecuactoues

85

Como resul tado

Ilega mos at sistema

de ecuac to nes

1000A+ 10B= :,!)B"+575 { (iOOA -I- B) - (100B + A) = 1287,


111.10,

des pues
~~2

de

resol verlo,

nos

perm ito

!lalla r

AI = <18.

B, = 35'

A2 = {

B2= - ~~.
del

Est:\ clare que cl segundo par 110 sntislncc las r ond icioues problema, Los numsros buscados son i'J8 y :35.

2. Problemas sobre progrestoucs. La snccsion Ilumerica (a'l) rec ihe c1 Hombre de progresioii art tnietica si ex is to III' [I ii rnoro lal d que para torla n EN so cumplc la igun l.lnd all +1 = a" -+- d; el 1111111('1'0 d lleva el nombro dc raz6n (diferel1cia) de la J!fogrew)u arilnietica. La SlICCSiUIl (II,,) CII 1a q IIC b1 =1= U '<:0 d euo III i ll<l rlragn~,'IUIt K!'olll/;Irica SI II a y llIl 11 urn era q =1= U tal que para toda n E N SC CIIIII plo la I gualdad &11 +1 = b ; .q; cl nuniero q rec ibe cl nomine de razdrt de la (JFOgresuu: gcomclrica. I'ropicdades Iundaruentales do Ia progresion aru.unit ica: 1) arl a, d (It -1). 2) S = at +all . n, (on d e Sn = (11 (l2 l 2 an·

Jl

+ + ... +

~~) La (propieclad

suasion

(all) es una

progresion.

etlan d 0 para

to d (1 n

E N se cum ple

. la igualc 1III l
ariimetlcas

aritmeiica

cuatulo
art+l"""
fin

.+l",..

y ,wilo
2

caracterisiica

de la progresuin

Propledades Iundamentales 1) bn =b,qu-I. )Srl=


bdl-'1")

de 13 progresion

..

goometnca:
.

1-1] ,dondeSu=b1+bz+···+b",q-=t=1. 3) La sucesion. (bll) es una progresion geometrica

cuando para toda n

Ell In pnicticn, en Iugnr do lu Jgualdad I b '+I I = ~! ; -b ; +:: vs b mas comedo hacor uso de In igual dad b!+1 = btl ·b +2• eqlliv,1ionte a In prirnera, 4) Sf La progresion geomeirica es uiitnuamen!« decrecienie, es
ll ll

la igualdad I bn+1 tpropiedod. caracterlstlca de la progresion geometrica}.


SI!

EN

cumple

I = Vb"

malitia

y solo ·bn+l

eo

dccir,

I q I < 1. entonces
sobrc
por regia,

S=

-fbi '_q

,dolUle

S = L..s bTl' "


J>=I

Los problemas
grcsiones, clones.

dcpendencias

numericus

ligCHios

rOB

pro-

se retlucon

a Ia solucion

de sisicmus

de ecuu-

l'lG
EJEIIIPI.O 1

U a l lomns

cl

(('1'(,(,1'

(enni]\o

de una progrosion

g('o~

mel rica

ill fin if ,111 I 0 dec rec icnte ai sa bema.'; q ue su su !lUI es igllui It'!! a 9, y ia sunra \I~ los cuudrndos de to.tos sus (t;l'm(nos es igunl (t 40,5.
SOLUGION,

St'gtlll

(>1 pl,l!lkamiento

A nul icemos [n scrio u~, u~" ... . De acuerdo con el pIIHI tea JJ\ il'TI to :';!.J su m<l es igual a 110,5, Sona lcmos fl lie los terminos de 1<1~L'{'io Iormnn una progroaion gcornetrlce con 01 pr lmor termino y In l'ilZOIl qZ, 10 quo signif ica quo In sumn de dicha

b;, b;. . ...

S =~l,

('5

dedI',

_l

ul

-q

=9.

b:

progresion
el sistema

es rgunl

-1

-q {

fIt •.
t~2q

Como resultado,
.=

os posihle
nl

oser iliir

de ccuncionss

'"7""---"--=-

l-q~

Q = 40 ,5

que

1'(,50\\'01"10

!lOS

propcrciona:
Si Jet

/;I=G,

s= ~.

Asi , pues,

uS=bl·ql'=G·({-f=

:7'

.&JEMPJ.(l" Trcs numcl'os lorman una prugrusidn g·colllct.rica. tercer numoro l'l'sla mos 4. los n urn eros Ioruuui lHIIl progrcsion 'll'iLriu;[jca. S1 de los tel'luilloS segundo y torcero de In progrosion <It'il-meLie;} obt euido rsstamos do cndu lIlIO do ellus Ia unidad , do nuevo ohtenemos una pl'ogrcsion geouuitrica. Hallemos esos numeros.
Scan .1.", 1/, z los numerus buscados, Como elIos Inrrn a n progl'csi'(m geOJlHilrica (con mayor precision, son tel'minos sucesivns dt' nnu progrcsion geonuitricn), liaclendo lISO do 51.1 pJ'Opr iedad ca rae I('ds(. icu, 0 b l enemos V'! = :l_:Z. Sogui dumont e, COUlD los uumeros x, !II (z - I,) forman una progreaion nritmetica, apl icuud o
SOLUC{ON

una

su

Pl'OPIO(

. I MI

'. caractensllca,
.Y,

lin 1lamos

!I = .1:+(2:-4) 2
UII<\

. P or

rIII,

ya que los nurueros

metrica,

entouces

W-

(y -

1), (z - G) Iormnn 1)~ = x (z - 5). al sistema

progresuin

geo-

Como

resultado,
'2.1/

Ilcgarnos

do ocuaclones

{
(

~2;zx~4 = gOg;

q \1 e al ser resuelto

1105

proporciona:

(1; 3; 9),

(y- 1)2 = x (z- 5), l 7 4a )

y.
los

Estes
1\:-;i,
!) •

volutes

de a , y, z satisfacon
!lUlIlC'I'OS

las cond lc ioues del problouin.


1,

puos,

Luscados

5011:

3· y

n'

u0

1.'

lJ1Cn

1 9"

7 9

EJEIIIPLO ~

mall una progresruu

ILdlemo::; uu numero lle Ires c ilrns, cuyas ad! iuetr ic a )' que se d iv ide pur 115.

c ilrns

[or-

11. Problemas

para ecuacrones de

y sistemas

de eCU3.ci')II\);1

81

SOLUCION,

de las unidades
., una progresron

Sea x 10. dim del mimero

[H:S C ent

bUSCHdo,

enns, !J, d o las tI L'( ('fl;'~ ) z, Como los n(unet'()s.T, Ij, z Iurmnn
!I;;' 1'5

'I' an 't raet.tca , Y

;r+z

2~'

Segiin el pla ntea miento el numero busc ado se d I\' i1(') pur .decir, pnr 5 Y por g. 0 sea, el IItlll1cro acabo CUll 1,1 elfl,) U

rnientras

que Ia sumn
resultado

de las ci lras

del

II

umcro

huscarlo

Ia Sl, ::-c d 1\ ide


(j

pOI' 9. Como

, Hegamos

al conjunlo

de .los slsleljj:ls:

Del primer sistema hallamos

X= {

x+y=

2y

9'

IC.

'I'ornando para y lodos los vnlores POSt hie.') Ii csdc 1 !w.~1;\ ~I, 110-5 cercioramos que el lilt imo sis lema 5610 sa sat lslace COli pi pn!' (Ij; :~). 21/=x+5 Del segundo sistema hallamos r.: { x "+y +;;> = 9" I~. De forma analoga, tomando para y lodos los vulorcs posihlos desds 1 hasta 9, nos csrcioramos que dicho sistema s(11) ~B ~(Iti~[ace con los pares (1; 3) y (7; 6). Asi, p ues, las cond icioucs tiel pro blema 1:iC sa t islac ('11 (0 II 1. \I~S mimaros: 630, 135, 765. ado plan las siguientes suposici ones: 1. Si no hay intlicacioues especiales, el lito Y j uueuto se (' 1I usid ern unilorme. 2, La velocidad se considera maguitud posit ivu. 3. Los giros de los cuerpos en mo vim ient 0, 1.15 t runstc io ues ~ . nusvos regimcnos de mov im iento so consldern q ue ll'l\ nsc urren iustan t,9.neamen t e. Ii .. S i el cuerpo a ln velocidad protJ iii :t: sec despln Z(\ jH)i' lUI tin, 1<1 velocidud de cuya corriento es y, la velocidad del cuerpo a Iavur Lie
S9

:3.

Problcmas

sohre

el movimleutu,

Al resolver

t.ules problcmus

10.corriente

te, (x -

l\ n aft uonte. U na Inucha snle va por Ia corricnteI:)O lun haste Ia deseuibocadura del alluento al do en 01 pun to By, " cout iuuar i,') 11, va corricnte arriba por al rio hast« 01 puul o C. £1 rocorrido desdl:! i1 hasl,a C Iue cubierto en e1 transcurso de "13 11, 01 recorrh!o ill verso , en 15 h. Hallemos Ia d lstancia dssd o el pu Jl to A h astuel C ~i :;a homos que 10. velocidad de in corriente del rio (>:3 ::I km/h r 11\ pro IIin velo(' i(lad de la lancha, 18 km/h.

EJEMPLO 6. A un rio descmboca del punta A, si Lunda en 01 aIlucnte,

y).

se co nsidera

igual a (x

+ y),

rn ient ras que eo at ra r onieu-

88
SOL ucion

I'ruucru

p"I'IC. Algehra.

Calli! lila II

el puuto mieutrns

(it! -

Sp(1 .z j..III ill Ia velocidad del <If J uen te. Eutonces, desdo A liM.la 1:1 pun to B In Iancha va a una vclocidnd (18+;;) krn/h, (11W de:c;de el puuto B hnstn cl punto A, a una veloctd ad ;I.:) km/h , co us II III ie lido en el recorrtdo d esdo A hastu B 18~ z h

Y ell el rccorrt

do desde

B hastn A,

8') 18-:r 11.

Spa y km 1<1d istnncin !tesde B hasta C. Ucspl az.iudose del punto B <11punto C 1<1 laucha sc mucvc a una vcloc idurl de '15 krn/h , ell tanto quo del punto C 01 puuto }J a una velocidud de 21 kru/h, cubrteudo

cl carni rIO de fJ a C en
ill verso dee a B,

tit. I' arn


'ille

l' [

transc urso de lod


001

t5

h, rni en tras quae!

reeorr ido

recorr ido de A a C la lanch a consume el plouLcalIlicnto


del pro hl erna

( 1~+X

+ J%)

h,lv

segull

ccnsti t uyc

13 h Y p<lJ"a c t rccorr ida iH verso, (13&.1:t" que seg\IH eI pl.iu ten m ieuto del problema co nslitu ye 15 It.
Escl"l un 11105 ('I
,S1St

+ il)

10

em a de ec unciones

{
es POS! LI e ex prosnr y
II ue se resoch

80
18+.1:'
8<1 18-x

i5

= 18

u +~.= 21

15 '

'

Como In (hsL/lIIcia de A a C es ignal a I a suma de las d Istancias de A a JJ (80 km) Y de JJ a C (210 km), Lodo cl rccorritlo de A aCes igual a 2UO Jon. EJEMPLO 1 Del rillito A "I punt o 13 sal io u n cnmicu. Una horn despues de A t\ if sa Ii0 un turismo. Al ptt n to B los dos all (0 movil es II cgaron III mismo u em po. Si de los puntas A y B ellos It uliiernn snl ido 51III til tfw.C'umonte a1 CIlC uentro, so encon Irari (Ill dcspucs del h 12 III iII de Sit part.i tl a. (,Cuan Lo t ieru pu lard H en c uhr ir-el calli ion Ia tl istnnc in entre A v 1J? SOLlJClON Selln~: j..Ill, 1I Ia v eloc it! ad rtel ca miun e y km/h , la del tunsmo, cubre
x y

Y= 210.

o si II d if ic ultn d COli el mCl.()do do susti tuc iun (p. ej., COli s: de Ia primcro cc II<.IC ion). Hall amos x = 2,

z kill, l a chstancin

do A a D. Eutoncos,

ct camiou

en

~ 11

ell UfC cl rccor r itlo (' n tJ'c A y fJ y el tnrtsmo, en _:_h. Del pi a Jl team iento
y

CS<lm isma dista ncia,

In.
que

rl el proble ma S8 desprcnde

.:... -...:.. 1" Los a u to w6v il cs, = ell ca. m ino htlsl<\ uncu ntrarse

d espl a :t<I.ndose 111 enc uantro , estan _:__

:t'+u' h 10

q lie! de acuerdo

con las

§ 1 f. Problemas

par a ecuaciones

y sistemas

do

(>cllilci(1II~5

8')

com Iiciones

de I pro bI ema,

const ituye
Z :

'I I1

1')'nu ...

n,

, 0 ~('.I,

ti T
,)

I Como I.

result ado , escribirnus

un sistema

de des ocuccioncs

con t rcs \ :1\'i:II]les~

x-y=J
{

x+v =5'
es mayor quo (11de ccuariuncs,

AUH!}110 el mimero

de illcogllitns

eJ problema puede ser rosuelto , ya que 110 es precise hallur los vnlnres de cada nun de las variables X, g, z , sino In rnzou !_ (I iempn de x
dcsplazamiento del camio n].

del sistema

a Ia forma:
Z

5z

Gx

Trauslormemos la segundo ecuacion Gy y, seguidamcntc, cscrrhunos:

5=(3,":"+(j,Y.
r.

Haciendo

It

= !_ ,
x

: = -,

rcescribimos

cl sis t.()Jll:l

(.1):

U-V=1, { 2+..£..=5
!l II '

(jue, al resolverlo, nos proporcio na: u = 3, f) = 2. Es docu, l'I c<lmiun cubre 01 recorr ido de A a B ell J h. EJEi\lPLO 8. E[ recorrido pOI' el q ue se rlespaza u n cidl~'!(1 cuust a es G voces mayor que In del tercero , cCu:lJ sent In vo loc ul,ul med iu rl e mov imien to del ciclista por todo el recorr irlu, si 811 Lemos lJ \I e es iguai a In velocidad en el .segundo sector, 2 km/h mellor (Llll' hi vc]oci cia d en el pri mer sector y 10 k m/h ma yor que I n III itn d de Ia vcloc ida d de despla zam ien to en eJ tercer sec Lor? SOLUClCiN. Sea x km/h la vel oci dnd med ia del cicl ista , !J kill, la longitud del tercer sector, Z km , Ia longil.ud d~'1 SCglllldu S!'nOL
6ykm [x
-I-

de

tres sectoros,

COli

In

pnrt

icul

ar idml

tle

quo

In

IOllg'ILnd Ill-I pl'llllCI'(}

2) km/II

,
x I.mjh (b

J;m

J!l

- ;:U IA 'Ii/II

Fig. 1 Entonces, segun Ia vclocldad del cicl istu


ell

01 primer

sector

,5('[,\

(':l~t-2) kru/h ,

ell el segundo, esta

x km/h

y ell oj tercero,

(2.t:- 20) J..m/It


ell el

(p

1111e.

eJ planteamiento,

=f + 10) . En

ligada

con

In vclocid.ul v del cicl istn In velocidad media x mediante


ofrece cl esquema

tercer sector In formllia x =


del

la fig, 1 se

do movunieuto

Pruners cicl istn.


ex presado

parte,

A!goura. Capitulo II del c icIisl n de A a IJ puede cou a yud a de clos proced im ser
ien-

m (icn.po
iu

l ruduc

ie

de rnovlmicnto nrlo Ins variables

lo~:
11) sumru l'l l ie iu po de ruo v iru ieu I.o ell cad a 11110 rie los t I'I'S secGy ( x+2
,t

lores:

-1"'-;-

2x-21)

y).

h, media del ciclista:

h) dividir 7y+z h.
x
COIllU

todo

01 rscorrido
escribirnos x+2

por

10 velocidad

rc-ultado,

la ecuncion: y
2x-21)

uy

+-;-+
y

(2)

Trnuslormumos

lu ccuac iou

(2) a In Iorma

_Q,g__L
x+2

r 2x-21l
y

7(I+z x
7y

_..!..
:t

~+ x+2

2x-21)

Dlvid iendo .uubos m iernbros de in ultima ccuacion por y (10 que no ronductr.i .1 la p6n!ida de soluciones, ya que sa comprenrle que JI =P 0), oht ~'l It'lIlCJ~
__ I _

x+2

1 ~ =_ + -;:--';_' 7

2x-20

de doude hallnuros
Ius coudiciunes

J.'t

= H.,

es de 14k mill.

del problema.

XZ

= -20. La seguuda raiz no satisfuce 0 sea, Ia velocidad media del ciclista

OBSERVACION. La ecuacion (2) coutiano tres variables, pero durante las (,raus[ormaciones dos de ellas, 1/ y z (el valor de las euales no era neceearto hallar) sa eltmlcaron. Semejantes varlables reciben el norabre de au:r!llares.

Autos de pasar (II siguiente ejernplo sciialcmos que, co nvencionalmente, podeinos considerar COUlO problemas sobrc el movimiento aq uel lns ('II 100:; que 1'(' rnnl iza cierto trn hajo (p.ej., se rectif ica cierta runl.irhul de [nezns, ~e Hena un deposito, cLC.). En los problemas de esle t ipo el valor de lodo el trabajo (numero de piezus, volumen del dcpcslto , otc ) desempelia el pnpel de distanciu, mienlras que cl reurlimieuto del t rn ba]o [es rleclr, 01 valor del 1.1'avajo realizudo par uu id lHI de [.18)1'1 po) j uega el pa pel de velocldad. r;:.!EJllPLO 9. POI' dOl; I ubos de diferente duimetro llegn el ngun a ~11I d('pu~,d l.o. EI jn-imer dia, los dos t uhos, Iunciouaudo sirnulttinealu, imout 14 1Il:1 solo
ruen a l aro n de ngun. El segundo dia estuvo co nee-

:11. Problemns

p;lfa

ecuaclonss

y sistemas

do

OCII(I(:.1')1I0>;

!) Ii

tado ei tu 1.10 pequeiio. EI ,d ime 11[6 t4 m'' do aguu Iu ur rou.uul» ;1 It nuis que 01 primer dia. El tercer d ia ('I {rabiJjo conti nuo i.l!_llUi t ieinpo que ol .':legull do, pero , in ic ial me nil' Iunc in I tn ban .\lIlIJO,s '1.1 ])lJ;5 )' til imoniuron 21 m" do agua y, a t'.ollli uuac ion, ~ol u f unci I)JH).IH\ ol L u 1>0 grand o que ali menLo 20 HI:I de flg\1 a JIl:·IS.~Qtlt1 Cit !1(ld:1! I d l' llgnil alirncnta cada l.ubo durante 1 It? SOLUGION. Son x m3/h ul reiuliruiento dd 11ILo gl<l rule , 1/ m:'/h, el reud irnian Lo do! til bo pcq ucfio , I II, el li!'111 pod mall to cl IiHe Iunc io nan ambos tulios ol primer di a. Eutonccs, ol prnner tlia los hr hos altmcntaron (x y) t m3 de ngun, 10 q HO, SOgt"1I1 tIl pIn nteumien to, consl.i Iuyo H m'', Obtcnomos la prl mera oc U C ion (1' -I- y) I. = = 14. En pl transcurso del segundo dia ('1 tubo pequeno (ral!tlju {t -\- !)) It y al imento y (l 5) m" de aguu, lu que correspond» a 14 rn". 0 Ltenomos In segu nda eeu ncion y (t 5) = H. E I If'n.:('r

,I

a tl'n1.Jajar los dostu bus I1li meutun.to 21 lH~ \10 ngua, 1. • . tl' 2t . , o son, su lral!<I]O coujunto . uro -+.-y- II.' \ cOHlUlllfl(:l{lll,'I· Ira 10110 or solo el ttl bo gra lido que ali men Lv 20 m3 £1(' nguil, Io q lie srgtul ic a 'I ue su trubajo uuru20 h. Como 01 tro Lnio del tercer ilin dun:, el mismo

dio. comcnznron

-+. y + ~ = t + rl. s: x
21 20

uempo

q He d u rnuto

III'InOS

el sogund 0,

obtoueiuos
HI sist erna

1o tercer a cc IllICi{Ht: de
('elli\(

I
CIII (\

J I ega do .

iuJi(':'I:

y (t+ 5) = 14

(X-I-.: Y.') t

.14.

~ x+u +.12.. = l -Hi, .x

Dc la sogunda
touccs,

ccuacion

Corma:-+ <1:-

I:\. prirnora ill 14


1)

del sistema hnllnmos: t 5 =- ~, !I ecuacion del sistema so pnorlo recscri lur y la tcrcsrn:
de dos
.. _ __

+
11

eu('\I

=--5
11

-j--+_._=-'
z--y x

21

.:!Il

1:\

ttl

o h (Ullom

as

el sist

CG

uae louos
t:

14

.=--;)

14
11

x-l-y

{ _. 21 ElilIl irian do en ambits

%+11

+_!!_=~. Y
a:

ecuae io nos los dcuomi lin d 0 res, (1)1(,llClHOS;

f 5xy

+ 5y'Z.= 14x
=0.

14x'Z.-27xy-20y2

!l2

Prirneru La scgun d 1\
cell

parte. AJgelJriI. Capitulo cs htlillogellen.

If

(11.,16!I del sistemu termino


~.

Di v idlcudo

£lllll,OS 'l =

miombros obtenemos son


ZI

at'

ella,
Zz = -

por Lennillo,
Lien

pur !Jz y haciendo

In rcuucion

cU:lt!ru

14zz- 27z -20


ruiz

= 0,

cuyas
x

raices

-=- • y
s

= ~•

La seguuda 5

nes del prohloma,

us rlecir ,
Vt'I'

z=z{'

no sat isfacc

las cond iclo-

.,10 q~((} significa

que

Y-=T= 5.

Q ucda

por resol

t'l sis tcma

Y= 2, 0 sea, d reudimicnto y del 110quefio, 2m3fh. 4. Prohleums sobre 01 traba]o conjuuto, Por regla, el coutouid o de semcjan res pro hl CIIlIlS so reduce a 10 sig« ion teo Cierto tra Lu]o , cuyo volumen no so rudico y 110 es ln mngniturl buscuda (p.ej., Ia
impresion de \!!L manuscrit o , la excnvacion de una Iosa, cl Ilenado de un dopusito, otc.}, os r oal izudc por vurias personas o mncunismos (Iua IIIHCionoJl nnif or morncute (os rlecir , con rcnd imlunto coustuuto de elida \1110 de cllos). En tnlos problemas 01 volumen do lodo cl tralra jo ,q lie ltil de SCI' real izn do, so tom a como Ia unid ad (por unidud de modida). Si el rcudnnicnto del trabajo , 0 SC.I, eI valor de Ia labor rcal lzud ... por unidad de ticmpo, so dcsigna pOI' v y cl l.iempo necesarlo para, real izur Ludo el trabajo
HI

5xy fly2 = 14x de) tubo grande cs 5 rn3/1i

Y=2

,de

dondo x

, por

l, entouces

"=

Pilla arar toda l a parccla el priruor tractor COJ)SIIJllC 2 JJ menus que el tererro y 1 h uuis que el Sl'l{lI11do. Al trubajar sirnult,IUCUIIlCll(C JOI'l tractores pri moro y segundo 1<1 pnrculn pucdc SOl" aralia durante 1 It 12 min. t:CutluLu Iicmpo so cousumini PUI'U arnr la parcela al trahajar ell conjunto los tres tractores? SOLUGJON Sen .r: It cl tiempu ncccsurio pant <l1'<H' l a parcela con cl primal' tractor, y II, con cl segundo y z II, eon 01 torccro, EI volumen del trnhn]o (en IlIJO.'Sll'O cuso este l'f! ul :ll'e" de In pcrcela) so lOIlH\
EJEMpLO

+.

+,

iguul

11

1.

En tonces, Y

dol segundo

!' del

..!._
;to

('·3

01 tcudim ien to del primer

tractor,

tercero.

Scgu II cl pla II tC<lIuicn to del pro-

blema z -;; =- 2 y x _ y = 1. Ademas, se ha t1 kilo f] uc durun to cl trabajo coujuuto de los tractoros primcro y segundo la parcela puedo ser ;)l'nd,l 011 el trnuscurso do 1 h 12 min, OS decir, dUl'UIILO
,6 LH'lI\jlO,

6 5 II. I'era, ell este

' 5' I1, o1 pruner -56. .!.. frnccion


y

tractor

rea j'rza

G1 S·;-

pa rto orl Lr,j il<l jo y d seg undo, . SlgUI'f'lea q ue


-r;.ICC

do esc

~rabajo,

ESLO

13!j

+ -5!J = r1,

§ 11, Problemas

para
0

ecuacio nes

y slstcmas

'!o ecuocroncs
CCI!il(".jI) II {'S

!U

tres

Como resul tndo , variuhles

bte nernos

u n sistc rna de tros

co II

que :,} rcsol vcr!o , obtencrnos: (a; 2; S). (-OA; -O.ll; 2/1) La condicion del pro blema solo se sa l is] ace co 1\ Ia pri mern sol uc rou. Ahora, demos respuestn a l a pregunta plantenda 011 el problema. Al t.rn lmjar los tres trac tores 011 coujunto cl reud i uiicnto del l['1\ ha]o
" const.i . tuira 1 1 "3 + '2

-x=2

x-y=1 6 G - ·-=1 5:>: 5y

+ ~' i

sea,

31 37T'

r.>

L'..5 l

I' eCII', e I'

[ICIUIIO

nocosnno.

para

arar

I a puree I a con 1as tres tractores

3) sera" IgUU1 n 3f II.

EJE1'dPLO I!. C nand 0 Ias cosoch ad oras dol soy jos *) t rnlrn jn II si III V 1lallcnnH'lIle pucrlou rcco lectar 11'1cosecha ell ul trunscurso de u n di,l. Pero , de acuerdo CUll 01 pl nu, ins cosochad orus COIIHmznrOIl (L t ruliujar conaccutivamoute: durante In prlrnern hora s610 tra linjohn IIlIn c.o:-;eclmdora, en la segundu, dos y en lu tercora, tres, etc., Y asi, hastu

q lie ernprezaron conjuntamente

a tra bajar lodes Ins cosechedorus varias horas hasta Ia rscolecciou

q He I rn bajnrou cnrnplata de la

cosecliu, El liempo de trn hajo , prev lsto por el pion, potlriil rcd ucirse <lit G h si desda el principio de In recolectn trubajnsen C(lnllllltallll'lIle torlas Ins coseclradoras salvo cinco lie elias. lCu5nlas (.()~~(hud{)ras habrti ell el sovjos? SOLUCION. Tumomns el valor todo 01 trabnjo ig-ll.Jl II 1 e intrnd uzca III os tres vu ria Ld os: It, el 11 (oHeru d I} cosech ndorus L'I\ l'l so \ jns. .r, 01 reudim ien to del tra bajo de 11 ua coscclmdoru Ill)r 1 It, t II, 01 t iempo de trahaio coniunto de las cosecha doras 5(,~UH 01 plan. Dl' acuunlo COli el pluntoruniouto, /1 cosechudurus , COil r ru d uu icutu .r cadu uuu de el las, pucileu ronl iznr In recoloctu ('II el tr.inscurso de 24 horus, es decic, 21nz = L Segun 01 piau, dura II te In jn-i mora horn tra ha]n ba una cosochu dora. El vol nmen de t raba]o real izado en esta hera as igual a I. E" lit soguuda hora trabaiaban dos cosecharlcras Y Oil el transcurso do lli!n rcal iznron till vol umen do t raha]o lguul (I 2x, Ell 1u ierceca horn, tros

no

Ell (n - 1) ho ra (n (II -

coseohadorcs

n un vol umen de truhujo igual a :tl', etc, 1) eosechadora IIlcieron 1111 "t fa b:ljo igllfll a 1) z, Despues de esto, en el \J'<lnsCIU'SO de t II. tea bnJH ron todns olectuaro

las ti cosechadoras y el volumen de trabujo rcal izndo por elias 1:'$ igual a nix, Como resultado, el truba]o plauificndo de Ins t'oserita{lo-

Sovjoz, abravlatura

do granja agricola estutnl

(11'. dd T.)

04
ras so

1'I'illlCHI

parte, i\JgcLI'I, COIl'lIulo If cell ac iun:


-

d cscr

Ll emos de ,1: (n - 1) lenuillos de I., progrcsio d = .r. Esto (l"iCI'C dccir que
,'I:

;;.' 2x + ... + (It + srfialur 'lite + 2.L -to


.. ' _L( r
Il

dJC co n In

s ig u ien to

1) J:

•,.

n arltnict 2

+ (n n

+ nix

= 1.
t)
x es lu suma do en III que a. = :c,

ica (an)

+2x +

-1) x =

x+{u-l)x

( _1)=n(n-1}x

y la ccuaclon rau

(3) torna la. forma

ns: ( 11 2 1 + t ) = 1.

Pur fill, del pl.uuearuionto siguo que si dcsde el coruieuzo hub ictrabajado (IL - 5) cosccha doras, In l ahor hubicra durnrlo IIG (IL - 1 I) 11, co m« cs l.a bn prev isto ell cl pl n n , si no G 11 mcnos , cs dccir, «(Jl - 1) I - 0) It y. cntonccs, (n t - 7) (n -:J) s = L COIllO resultnd o obtcnemos 11 n sistema de tres ecuaciones rospecto do t res varinbles n, a;, t:

24nx= t
nx ( ,1-1

-y-+

t =1
(n.1.'-5x)=

(n+t-7)

1.

., 1 . D c J.11 pruner» ccuacron 1 11 HI amos I/X= 24' P omen d0 osta oxpresitin C'11 Ill. scguudn y torcora ccuucioncs dol si:;lCIUII, obtcnemus:
/IX=24
1

1I;[+t=24

t (n -]- t A continuncron, el sistema


119-n

7) (
50

;4 -

5x ) = 1. dif icultnd
.

rosul vo sin

sogun

ul
'

nuHoclo de sus l.i tueion. de In seguuda, t = -2-' ccuncion , obtcneiuos:

Do Ia pr imera ec llilCiulI hallumos Poniendo estus


CXIJl'CSlOnCS ell

x= 1

1 24n

a torccru

(,,-1- 35)
do do rulohnl l.uuos
teurnicntu). Es rlor It ir,

(rt - 5) -1 48n -,

~('glilldil scl uciou JlO sutislacc 01 plunsovios habia 25 cosech adorus. 5. Preble mas sobre aleaeiones Y mezclas. En los problemas de I'll el

= 25 (In

este l.i po sc 11';\ tn de b crcac ion de m ezclas, III onciones, d isol ucio ill'S, etc. La rcsol uc iun de seine jantcs pro bl em as esui rei acionada co n 10s

§ 11. Problemas COl)C(\plos

para ~cuaciones y sistemas tlo ecuocroues

~):>

ctc.,

de «cone ell !.rnei6n'), «porcentaje», (,lIIm~,~lJ'1I (,Ii L IIII ~\I.ul», '), y sc hnsn en las sigu ien Les S1.!POSicIOIICS:
1[IS rnezclas

1. Todns

ohton id <IS (aleaclonos,

d ISO!

IIUO

nos]

5()1I

homogcneas. 2, No se hace dilereuc ia sntre ('1 litro como uuiilad de cap,widul! y (>1Iitro como unidad de masa. Si tina mezcla (aleacion, disol ucidn) de ill asa m co Wit a .le Ius siistanclas A, 8, C (que t ienen masas 1/11' 1n2' /Ill' respec [,t \ o mcn te),

In magnitud

m m traciiin. de In sustancia

!!:!. ( m~ , 2::.! , tcSpectivamento)


A (E,
m

se denomtna

concenrccibe de Ia

C,

respect ivamente)
m

en la mezcln.

La magnitud

.!!!!.100% (~
m

10m&, ~
que !!:l_
1ft

100%, respectivamente)
es dccrr,

e1 nombre

de porcentaje

de la sustancia

en Ia mazcla.

Esta elaro,
Tenemos

COlleen tracion

tercera.

do dos sustancias
12,

+ ~ + ~ = 1, m
nt

A (B, C, respecuvnmcnte)

depende

In COlleen tracI!) 11 de Ill. de coure v estano

EJEMPLO

de una masa de 12 JIg que contieno el 45 % de cobro. dQw! cl1ntld,Hl de estaiio puro hay que afiad it' a esta aleacion 1)11 ra fJ ue 1<1 nueva contenga e1 40 % de cobre?

un

[,1'02.0

de una alenclun

SOLucrON. Sea que Ia masa de estafio que hay que ml,Hlil' 11 11\ alene ion es igual Il x kg. Entouces, S8 obtendra una aleaciou de mnsa (12 x) kg con un contenido del 4.0% de coke. 0 sea, ell L\ nueva

aleacion
de
III asa

hay

1: t
l

40 kg de cohrc.

La alcaclon

iuiclnl

tk~ J2 J\~
I!O~ '{l3 kg. I
011 1;1

contenia

el 45

% <1 e cubre, es dscir, en ell n habiu


ill

Como hi mnsa (Io co bre, tunto (' II In al eacion


es Ia misrn a , podemos escrihir 1a siguienLe
(12+x) 40 = ~.45 WI] lUll'

icial como cell aciun:

uu ov a

Despues de resol verla, oh teuemos z = '1,5. Asi, piles, inicial hay que aiJadil' 1,5 J{g de estaiio.
EJElI1PLO

CI

In nl cue iun

del 5 tnmar
COil

% y 40 %. <::Que cantrdad
para despues

13.

Tenemos

Sea In masa del acero de In pri mera marc» i~lIld x t., cntonccs hay (Jill" tomar (140 - .l) t de acero de III s{lgumla runrra. El contcnido de niquel en 81 acero de In ptimern mnrcn constituvo 01 5%, por 10 tanto en x t de acero de dicha mnrca habra ;t' ,0.05 ~ do niquel, El contanido <.1(' este metal ell e1 acero de Ia seguuda 1l1[11'('U es el 40 %, por 10 que en (140 - x) L de. acero de l a segurul a 111<11,',\1 l'l
SOI ..UCION.

contenido

do niquel

acero tie dos closes eon conLenidu de III(jUel de acero de una y otra 11l(\[(,H hay do do In refundiciou producir J 40 t de acero

del 30%?

Prrmera

parte. Algebra.

Capitulo

II

contenido

de uique l constit uye ("l40 - x) 0,4 L. Segun el pl anteam innt.o , despllt!s de u n ir Ins dos marcas de acero toruuilas ha de obtencrso 1(10 t tie ucuro COil un con l.en iclo de niquol ig unl ,,1 30%" 'es decic, al aea ha r Ia rc[ulldicion en et acero pro duc ido deben habet 140 ·0,3 L tie n iq uel. Pcro esta ca nt idad de niquel se forma de ,;t·O,05 t, coutenidas en e1 acero de la primera rnarca y de (l40-x)X xO,4 t, en el d(' III segunda. Asi , put'S, escribaruos la ecuacion

X·O.O:>

+ (H.O -

x)

O,lt = 1110 ·0,3,

-de dunde hallaroos x = 110. Pur 10 tnnto , !tllY do t.omar liO L de accra con co ntcnulo del G % de niq uel y 100 t, de acero con e140 % de niquel , EJEl\jPI,O 14 De IlIl rccip iento de 51}1 de capacirlad , Ileuo de aeitio. ::5(' han vert ulo vnr ios lit ros y se agrega agua y. a conttnuacion, de II uevo , S8 v t erto 1tl 1lI isiua en nLi dnd de 1itros de J a mezcl a , Entonces,

-eu la mczcla rostante quctlau 24 I de licitlo pilro. ~Ql1e cant idad de -ac ido flit' vert ida la primcra VC1.? SOJ.UCl(JN Sea que la pr imera vez se v irl.io z I de acido. Entonces eu el recipient o quedaron (!){! .:r) I de ac ido , Al niladic agua al reci p icut c obt uvimos 541 de In m eznl " en In que se dil uyero n (S4 - x) I
t

,de acido. acido


.-:1.

Est»

slguif ica que

1 1 de In

1(1

rnezcla En

contiene e I.

5~tl x 1 de

(cuncentrac

del rec ipi en te se vertierotI x 1 de In ruszcla y esta call t.idad cou tenia :'Hr:-: x .;:. 1 de {wi do. Asi, pues, In p ri mera vez so vertieron x 1 de
;)'1

io n

de

rnezc la).

segun do

easo ,

.;icido,

Ia segu udu,

5,\-1.
,)I

,'t.

z , Duran to las

dos veces

Iueron

vcr lidos

.54 - 20 = 30 I do acitlo.

Como

resul tudo , eJaboramos 30 .

le ecuacion

54-x x-!---x= 54 Despues


(\0 I'OS{)

Claro

Ivorl a ohtenornos (\os ra ices Xl = 90, x 2 = IS. XI = ~JOno sat isluce al problome. Por 10 tanto, In primera vcz Iuerou vert idos 18 1 do iicido, rmmrLo 15 UH recept iiC\110 <10 8 I de capaculad esni relleno de u on mczcl a do ox igeno y ni tr6gclIo, co n In pnrticu lnridatl de q ue e1 oxigcno OCIIV" ol l() % de la capacirlad del recoptaculo. De el sa evncuu ciI.W[H caut irlud de Ia mezcla, so rel1ena al receptaculo de Jlitorgcuo y, de IllICVO, se evacua unn misma cant idad de 10 mezcl a , daspues de 10 tllHl al recept acul o 50 aiindc nitrogeno. Como result ado en el rec ipicute el co ntcnido de ox igeno 8S igual al U%. ~CllunLos Iitros de 1<1meld n so cvacuo del rcceptaculo cada vez? SOL!JC[ON Supougnmos que c ada vez se avncua han z I de Ia rnezcla
esLii quo el vulur y se al imentahan z I de nitrogeno. Despues de Ia prirnera evucuaciun

§ 11. Problemas

para

ncuacicnos

y sistemas

de

('(lIdt·lunc'S

\17

un nl rocepuiculo querl a hun (S - .r) ·U,lli I de ox iuu no (jIlt' ',e .11111) ('I".lll ell 8 I do I a mezcl a (Iuego tlo ail a d ir pOI' pri mcru \' ez II iI ]'(Jg" 110J.

E II es t a etnpa,
es dcci r , (8 -x) una conccutracion

1 cuncen 3
0,02.

l.rncion

.,

ue oxrgcno cvacuado

l'

es 19l1a

liS-x)
<l.

11,113

'

Al haber

pOI' ~Pglllldil

'fez x 1 do
\'011

13 mezcla , en ol reccpt acu lo qucdaron


de oxigeuo
.,

quctlnhnu (8-.x) (8- x) 0,02 1 de oxigono, que so diluyorou en 8 I dp lit mezcla (despuds de alimoutar nitrogcno por scgunda vez). Ell l'slit

igual
,

a (8 - x) 0,02,
.

(8-x)

I de In mezoln
us dectr,

etapa,

1£1 conceutracion
(8-z)%

do oxrgono

es igunl
ob

(8-~)~·n,1l2

cl

porccn tnje,

0 ,02 ·10016. 0
8

A'si , plies, 100


= 9,

lCIIClllOS

la

, ec uacion

(8-.:)2.0,112.

de donde hal lamos Xl = 2, X2 =Ui. Como VClllQS, C5 im peeiblo evacuar H 1 do un rscepuicu 10 en 01 que hnhia 8 1. De forma que CI1,J(I vez sa evacunban del t'cccpt,iClllo 2 1 do 10 mezcl a. EJEMPLO l6. Tenemos dos aleaciones cle mesa a kg y b li.g COIl diferente porcenlaje de cobre. De cada 11M de 1113 nlencionos so COl'to

(lJ - :c)kg (a - :X}kj( y%

z%

Fig. 2 un trow de igual masa, los cambinron de Jugal' y Iundierou junto con

los restnntcs trows ric las aleaciones iniciales. En las nuevns nl enCLones el porcentaje de cobre so hizo igual. lenal es In masa de cada uno de los trozos cortados? SOLUCI6N. Sea x kg In masa de carla 11110 de IDS trows eorta.los. y %. C I porcen taje de co bre en 13 pr iruora aleacion, z I!!). ('1 porr e 11taje de cohre en Ia segundn. Dospues de cambial' de Jugal' las partes de masa x l,g on la Pl'illll'I'H aloacieri obtenida (fig. 2) 1£1 t idad de cobrn ~pd. can
1-02!)I.

'\~;T'!I + I~IJ'::'

08

prj

til ern jHH

tu, Algcuro.

Ca pitn 1(I r r

r pi porccu
{a. :rl 'I
U.

tiljl'

do cobre

SCnl

igua!

,I

-+

VIIT·y • tlJlI'z ------100%,

(I.-:r

:r1.

"

es decir, u nu cun tidHd

.E 1\ lu Sl'g l.IIIdli ali~ilci6!l


d e co ure """"TiIi"l""z
b=:«
tlill
L

I>-x

+ 1I1U'Y'
s:

0 btt.u idu
que

(fig.

3) hnl>ru

m iontras

81 porcentaje
(Ii -Il/,

de

cobre

sera

--.z-I--·u
h
111il •

:f

100%, es dccir , igual a

z+ :ry•

De ncuordo con

el p ];1I1lC;llllHHllo, I:1Jl Ias .11 oacionos oil ~on idns 01 porcon taio do co bro os ul T11i.'\J1 10. 1\:-;). p u es, 1Iegomos a In cc uuc i6 n
((I-.r) y+xz b
SIICl, .

(b-.r) z+xy
/,

..,i V iun ('III


(ahy -

e (.OJIClllUS;

oby -

uxy

+ vu = alrz
bxz) z) ux (y -

u;rz
(a:t·y -

+ a.c.ll,
axz) = 0,
z) =

(tln) (y -

(I;xy z) ((lU -

ab (y -- z) el pl<Wll:'Hlllll'llto

ax (.II -

ax -

"x)

o.

0,
ax (;X

Segull

!J '4= Z, tic forma

que ab -

= 0,

de do ndo b~d!nUlOs ;1: = !t~b . Las vuriublos y, z so el iminaeon J 111"1!J! l o ln 1"050 l ur; 1I;!l de 1<1 cc lJ3C it)!) o b lcnidn (nux illnres), Asi , pues, 1.1 masa de carla uno de 10$ l.rozos cor tados es ig ual a

"-r-b kg.
lm:mC.ICIOS

0,,/)

480. La (ill rna d () 10:; cua d ra dos do las cifrM do un mero de dos cllra s ea igual n 10. S I dol nuruoro buscado sustrncrnos 18, o htcnomos un mnuero escrlto con osas nusmas cifras, pero en orden inverse. Hallen cl nurnero buscado. 1i81. t Q ue au I J IC!~ de dos c ir rus es II vcces ma yo. r que la suma do sus cif rc s y 3 voces mayor quo. cl pr,!(lil~ to de elIas? lj82. Ilnllcn dos uurneros CJ\lOfl)S, ellyn suma E'S (gual a 1244. Si al primer

uu

nu mero S" Insr ribe n lfl dorcchu In ctlrn a y del segundo sa elimlna Ill. (dLilllf1. citra 2, los nrnnoros oiltl'ni(!o5 l'-('T;in igunlNI. ,'18:), \J 11 11tun r rn de tros Co if r<1Sturrn inil C'JIl 111 rn 3. S i csla so lruspnsa n I coeif mivuzo d('lllUIllI'TU, ('I nuevo senl mayoron 1 que cI nurnero inicial tripl icntlo, II ullon l,l ruunor» inic ial. IiBtj. Un !lIJJlICro de s,'L~cirras (~nmien7;a POL' Ia cilra 2. Si truspasamos esta del flL\~uid(J sl.'r;i tn-s vt-oes mayor quu III inicial. Halleu ests. 1t8::;. La 511!l1U d (' tod os lo S J\ (lin eros pal"f'S r un rl iv 1d ida sin resto pi! r 11J1 0 de ol I()s. rl n lien (OJ r] iv isor st aahomos que In sum" do sus cltras es igual (\ tJ Y quo 01 ~"cicnlc; gr) dif crcnc ln del d ivtsor s6!0 I)tH' I,lJ orden de Ias cllrns.
primer pucstn atii! limo, ccnservando el ordan dB las tlcmas, ul numero

You might also like